Sei sulla pagina 1di 119

Copyright 2014 Delhi Academy of Medical Sciences, All Rights Reserved.

1/119

Test Information
Test Name

GRAND TEST 134

Total Questions

300

Test Type

Examination

Difficulty Level

Difficult

Total Marks

1500

Duration

180minutes

Test Question Language:- ENGLISH


(1).

A bacterial culture with a starting density of 103cells/ml is incubated in liquid nutrient broth. If the bacteria have both a lag time &
generation time of 10 minutes, what will the cell density be at 30 minutes
a. 1.0 x 103
b. 2.0 x 103
c. 3.0 x 103
d. 4.0 x 103
Solution. (d) 4.0 x 103
Ref:Read the text below
Sol :

Bacteria divides by binary fission in logarithmic fashion.

The gene when triggered cannot be inhibited.

Therefore after lag phase of 10 minutes two divisions will be possible in the next 20 minutes.
Your Answer. b
Correct Answer. d

(2).

Positive cold agglutinationtest is seen in infections with:


a. Mycoplasma
b. Chlamydia
c. Infectious mononucleosis
d. Varicella
Solution. (a) Mycoplasma
Ref.:Read the text below
Sol :
Agglutinins are antibodies that cause the red blood cells to gather together.
Cold agglutinins are active at cold temperatures.
Febrile (warm) agglutininsare active at normal body temperature.
The presence of warm agglutinins may occur with :
Infections, including brucellosis, rickettsial disease, salmonella infection and tularemia.
Lymphoma.
Systemic lupus erythermatosus.
Use of certain medicines, including methyldopa, penicillin and qinidine.
The presence of cold agglutinins may occur with :
Infections, especially mycoplasma pneumonia.
Previous viral, staphylococcal, or malaria infection.
Cancer, including lymphoma and multiple myeloma.
Systemic lupus erythematosus.
Your Answer. a
Correct Answer. a

Copyright 2014 Delhi Academy of Medical Sciences, All Rights Reserved.

2/119

(3).

The necrotising enteritis is caused by:


a. Clostridium perfringens
b. Clostridium botulinum
c. Clostridium septicum
d. Clostridium histolyticum
Solution. (a) Clostridium perfringens
Ref:Read the text below
Sol :

Your Answer. a
Correct Answer. a

(4).

Causes of antigenic drift in influenza viral infections :


a. Small mutation in neuraminidase and hemagglutinin
b. .Large mutations in hemagglutinin only
c. Step mutations in viral genome
d. None of the above.
Solution. (a) Small mutation in neuraminidase and hemagglutinin
Ref.:Read the text below
Sol :
Influenza virus type A strains can be classified into subtypes based on variations in their surface antigens.
Originally only variations in the hemagglutinin were studied, and subtyping depended only on antigenic shifts occurring in
the hemagglutinin.
The earliest isolates (WS, PR8 and related strains) were designated A0. In 1946, the hemagglutinin underwent a major change and
these strains (CAM, FM1 and others) were named A1 or A (A prime). FM1 and others).
In 1957, new pandemic strains originated in Asia. These were called the A2 (Asian) strains.
The next major change occurred in 1968 with the emergence of the A2 (Hong Kong) subtype.
Within each of these subtypes, he strains have shown a gradual antigenic drift.
With the recognition that viral neuramindase also undergoes independent antigenic variation, a new system of classification was
proposed by the WHO in 1971, which took into account the nature of both the surface antigens.
According to this, the hemagglutinins of AU, A1, A2 (Asian) and A2 (Hong Kong) were named H0, H1, H2 and H3, respectively.
This classification was again modified by the WHO by grouping together H0, H1 and Hsw under the designation H1.
The neuraminidases of human influenza type A viruses belong to two subtypes the subtype N1 occuring in AU and A1 strains, and
subtype N2 in A2 (Asian) and A2 (Hong Kong) strains.
The complete designation of a strain will include the type, place of original, serial number and year of isolation followed by the antigenic
subtypes of the hemagglutinin and neuraminidase in parenthesis, for example A/Hong Kong/1/68 (H3N2).
Your Answer. a
Correct Answer. a

(5).

All of the following statements concerning transduction are true except:


a. Transduction may be generalized
b. Transduction may be specialized
c. Temperate phages are the preferred vehicles for gene transfer
d. The phages make slow replicas of their DNA
Solution. The phages make slow replicas of their DNA
Ref.:Read the text below
Sol :
In fact, the capacity of phages to make rapid replicas of their DNA has made them valuable in genetic engineering.
Your Answer. a
Correct Answer. d

Copyright 2014 Delhi Academy of Medical Sciences, All Rights Reserved.

3/119

(6).

Which of the following statement is not true regarding transduction?


a. It involves the transfer of bits of genetic material from bacterium carried by a bacteriophage
b. Some bacteriophages mediate generalized transduction
c. It is less reproducible than transformation
d. It is more reproducible than transformation
Solution. (c) It is less reproducible than transformation
Ref.:Read the text below
Sol :
Transduction is a process of bacterial gene transfermediated by bacteriophage particles.
Some bacteriophages mediate generalized transduction, which may transfer any bacterial genes; others mediate specialized
transduction, in which a particular phage strain can transfer only certain genes.
Transduction is more reproduciblethan transformation because the DNA is protected from damaged by the surrounding phage
coat.
Your Answer. c
Correct Answer. c

(7).

Regarding dengue fever:


a. The usual incubation period is 2 to 3 weeks
b. The fever may have a diphasic course (saddle back form)
c. Absolute bradycardia is seen early
d. The backache and pains in the muscles and joints are mild.
Solution. (b) The fever may have a diphasic course (saddle back form)
Ref.: Readthe text below
Sol :
The onset of dengue may be sudden with a sharp rise in temperature.
The temperature returns to normal after 5 to 6 days, or subsides on the third day and rises again 5 to 8 days after onset [i.e., a
diphasic saddleback course].
Your Answer. b
Correct Answer. b

(8).

The distinguishing characteristic of a positive delayed type hypersensitivity skin test is :


a. Erythema
b. Necrosis
c. Induration
d. Vasculitis
Solution. c) Induration
Ref.:Read the text below
Sol :
Delayed type hypersensitivity (DTH) skin test are useful as epidemiologic screen for certain infectious diseases, especially
tuberculosis and various fungal infections.
The minimal indication of a positive DTH skin test is the presence of induration at the site of the intradermal injection of
antigen.
The induration usually occurs within 24 to 48 hours and is primarily the result of the infiltration of lymphocytes.
Erythema frequently accompanies induration, but it is not indicative of a positive test.
Individuals who display a strong positive reaction to the skin test may develop a necrotic lesion at the site of injection, and secondary
bacterial infection is a potentially serious side effect.
Vasculitis and neuritis are not associated with a positive DTH skin test.
Your Answer. c
Correct Answer. c

Copyright 2014 Delhi Academy of Medical Sciences, All Rights Reserved.

4/119

(9).

Delta hepatitis only occurs in patients who also have either acute or chronic infection with hepatitis B virus. The delta agent is :
a. An incomplete hepatitis B virus
b. Related to hepatitis A virus
c. A hepatitis B mutant
d. An incomplete RNA virus
Solution. (d) An incomplete RNA virus
Ref.:Read the text below
Sol :
The delta agent was first described in 1977 and has recently been shown to be an incomplete RNA virus that requires HBsAg for
replication.
It is found most often in persons who have multiple parenteral exposures (e.g. intravenous (IV) drug abusers, hemophiliacs, and
multiply transfused patients.
Your Answer. d
Correct Answer. d

(10).

Cell envelopes of both gram-positive and gram-negative bacteria are composed of complex macromolecules. Which of the
following statements describes both gram-positive and gram-negative cell envelops?
a. They contain significant amount of teichoic acid
b. They are diffusion barrier to large charged molecules
c. Their antigenic specificity is determined by the polysaccharide O antigen
d. They act as a barrier ot the extraction of crystal violet iodine by alcohol
Solution. (b) They are diffusion barrier to large charged molecules
Ref.:Read the text below
Sol :
Bacterial cell envelopesconsist of both the cell wall and cell membrane; in gram positive and gram negative bacteria, this
cytoplasmic membrane acts as a diffusion barrier to large charged molecules.
The cell envelope of gram negative bacteria contains lipoprotein, lipopolysaccharide, and peptidoglycan, molecules; the
polysaccharide component of the lipopolysaccharide is the O antigen, the chief surface antigen of these bacteria.
On the other hand, gram positive organisms contain large amounts of teichoic acids, which are important surface antigens for these
bacteria.
The cell wall of gram positive bacteria acts as a barrier to the extraction of crystal violet iodine complex by alcohol; this property
is the basis of the Grams stain.
Your Answer. b
Correct Answer. b

Copyright 2014 Delhi Academy of Medical Sciences, All Rights Reserved.

5/119

(11).

Mechanism of action of polymerase chain reaction :


a. DNA hybridization
b. DNA amplification
c. RNA amplification
d. RNA hybridization
Solution. (b) DNA amplification
Ref.:Read the text below
Sol :
The polymerase chain reaction is a test tube system for DNA replication that allows a target DNA sequence to be selectively
amplified, or enriched, several million fold in just a few hours.
Within a dividing cell, DNA replication involves a series of enzyme mediated reactions, whose end result is a faithful copy of the
entire genome.
Within a test tube, PCR uses just one indispensable enzyme DNA polymerase to amplify a specific fraction of the genome.
In one application of the technology, small samples of DNA, such as those found in a strand of hair at a crime scene, can produce
sufficient copies to carry out forensic tests.
Your Answer. b
Correct Answer. b

(12).

The 'safety pin' appearance is given by all except :a. Burkholderia mallei
b. Burkholderia pseudomallei
c. Yersinia
d. Calymmatobacterium
Solution. (a) Burkholderia mallei
Ref:Read the text below
Sol :

Burkholderia mallei gives a beaded appearance

Burkholderia pseudomallei shows typical bipolar 'safety pin' appearance.

Yersinia show bipolar staining (safety pin appearance) with the two ends densely stained and the central area clear.

Calymmatobacterium show bipolar condensation of chromatin giving a closed 'safety pin appearance' in stained smears.
Your Answer. b
Correct Answer. a

(13).

Which is the most common pathogenic parasitic infection in humans?


a. Giardia
b. Entamoeba
c. Ascaris
d. Taenia solium
Solution. (a) Giardia
Ref:Read the text below
Sol :

Giardia lamblia is the most common pathogenic parasitic infection in humans.


Your Answer. b
Correct Answer. a

Copyright 2014 Delhi Academy of Medical Sciences, All Rights Reserved.

6/119

(14).

Rose-Waller test is a type of:


a. Side agglutination test
b. Tube agglutination test
c. Passive agglutination test
d. Antiglobulin test
Solution. (c) Passive agglutination test
Ref:Read the text below
Sol :

Rose-Waller test is a type of passive hemagglutination test, which is used to detect RA factor in patients of rheumatoid arthritis.
Your Answer. b
Correct Answer. c

(15).

Which of the following is not a complement inactivator:


a. S protein
b. Factor I
c. Factor H
d. C4 binding protein
Solution. (a) S protein
Ref:Read the text below
Sol :

S protein is an inhibitor of complement system not an inactivator.


Your Answer. a
Correct Answer. a

(16).

Thymus dependent area of spleen is:


a. Around the central arteriole
b. Perifollicular region
c. Germinal center
d. Mantle layer
Solution. (a) Around the central arteriole
Ref:Read the text below
Sol :

The lymphatic sheath immediately surrounding the central arteriole is the thymus dependent area of the spleen.

The perifollicular region, germinal center and mantle layer forms the bursa dependent areas.
Your Answer. b
Correct Answer. a

Copyright 2014 Delhi Academy of Medical Sciences, All Rights Reserved.

7/119

(17).

Panton - Valentine toxin affects:


a. Red blood cells
b. White blood cells
c. Platelets
d. Liver
Solution. (b) White blood cells
Ref:Read the text below
Sol :

Panton - Valentine toxin is cytolytic to polymorphonuclear cells, macrophages, and monocytes.

It is also called synergohymenotropic toxin.


Your Answer. c
Correct Answer. b

(18).

Most frequent site of colonization of Staphylococcus aureus is:


a. Anterior nares
b. Perineum
c. Vagina
d. External auditory canal
Solution. (a) Anterior nares
Ref:Read the text below
Sol :
Anterior nares is the most frequent site of human colonization of Staphylococcus aureus.
Your Answer. a
Correct Answer. a

(19).

Immunoglobulin on surface of basophils, associated with anaphylaxis is:


a. IgE
b. IgA
c. IgG
d. IgM
Solution. (a) IgE
Ref.:Read the text below
Sol :
IgE is central to human anaphylaxis by virtue of its binding to mast cells and basophils.
When these cells are activated by binding of antigen to the IgE, they release of an alphabet soup of active substances which may cause
hypotension, bronchospasm, urticaria and, in some cases, circulatory collapse and death.
Your Answer. a
Correct Answer. a

Copyright 2014 Delhi Academy of Medical Sciences, All Rights Reserved.

8/119

(20).

Which of the following is a powerful macrophage chemokines:


a. MCP-1
b. MCP-2
c. MCP-3
d. MCP-4
Solution. (a) MCP-1
Ref.:Read the text below
Sol :
Monocyte chemoattractant protein 1 (MCP-1) is one of the most powerful macrophage chemokines.
Your Answer. a
Correct Answer. a

(21).

Local anaestheticsolution used to produce extradural anaesthesia is:


a. Bupivacaine
b. Ropivacaine
c. Cocaine
d. Procaine
Solution. (a) Bupivacaine
Ref Read the text below
Sol:

Local anaesthetic solutionsare deposited in the epidural space between the dura mater and the periosteum lining the vertebral
canal. The epidural space contains adipose tissue, lymphatics and blood vessels. The injected local anaesthetic solution produces
analgesia by blocking conduction at the intradural spinal nerve roots.

The quality and extent of the blockade produced by each agent is determined by the volume as well as the total dose of the drug. The
spread of local anaesthetic solutions may be more extensive ipregnant women as the volume of the potential space is reduced by venous
engorgement in the epidural space. Enhanced effects may also be seen in the elderly and in patients with arteriosclerosis due
to impairment of vascular absorption from the epidural space.

Bupivacaine (0.5%) or lignocaine (1.5-2.0%) are usually used to produce extradural anaesthesia. Repeated administration of
lignocaine or mepivacaine into the epidural space may result in a diminished response with each subsequent dose (tachyphylaxis). This
may be due to local changes in pH due to the relative acidity of these solutions. The reduction in pH may reduce the amount of
free base available for diffusion across the neuronal membrane
Your Answer. a
Correct Answer. a

(22).

CNS toxicity of local anestheticsis thought to be due to


a. Direct stimulation of seizure foci in the cerebral cortex
b. Depression of the cardiovascular system
c. Direct stimulation of excitation neurons in the cerebral cortex
d. Depression of inhibitor fibers in the cerebral cortex, which leads to unopposed facilitory neurons
Solution. (d) Depression of inhibitor fibers in the cerebral cortex, which leads to unopposed facilitory neurons
Ref:Read the text below.
Sol:
There is no direct stimulation by local anesthetics of seizure foci or excitatory neurons. CNS toxic effects precede
cardiovascular depression.
Toxic effects in the CNS are excitatory in nature and are thought to be due to initial depression of inhibitory fibers.
Twitching, tremors, and seizuresare the signs of unopposed facilitory neurons at work.
At yet higher tissue concentrations, neurons are also inhibited, which leads to generalized CNS depression or coma.
Your Answer. d
Correct Answer. d

Copyright 2014 Delhi Academy of Medical Sciences, All Rights Reserved.

9/119

(23).

Local anesthetic depression of myocardial contractility is most closely related to the


a. Volume of drug injected
b. Calcium channel blockade
c. Potency of drug injected
d. Site of injection
Solution. (c) Potency of drug injected
Ref:Read the text below.
Sol:
Drug potency is directly related to impairment of cardiac contractility.
It takes only a small dose of a potent drug, such as bupivacaine, to cause myocardial depression compared with the less potent
drugs.however a high enough dose or large enough volume of any local anesthetic, regardless of potency, can reduce cardiac
contractility.
Your Answer. b
Correct Answer. c

(24).

Per kilogram of body weight, the highest dose requirement for thiopental at induction is in
a. The elderly
b. 30-to 50 year old adults
c. 6 to 12 months old infants
d. 1 to 6 month old infants
Solution. (d) 1 to 6 month old infants
Ref:Read the text below.
Sol:
The highest dose requirement for thiopental at induction is in 1- to 6-month-old infants.
The ED50 (the dose at which 50 percent of the children are induced) at this age was found to be close to 7 mg/kg.
In adults the induction dose range is 2.5 to 4.5 mg/kg, while in older children it is 5 to 6 mg/kg.
Your Answer. b
Correct Answer. d

(25).

Which of the following orders of speed of recovery and return of psychomotor function is correct?
a.
b
c.
d.
a. Propofol > midazolam > thiopental > methohexital
b. Propofol > thiopental > midazolam > methohexital
c. Propofol > midazolam > methohexital > thiopental
d. Propofol > methohexital > thiopental > midazolam
Solution. (d) Propofol > methohexital > thiopental > midazolam
Ref:Read the text below.
Sol:
Propofol has the most rapid recoveryof the four induction agents, irrespective of the maintenance agent.
Methohexitalhas a slightly faster recovery than thiopental.
Barbiturates have more rapid recovery than benzodiazepines such as midazolam.
Your Answer. c
Correct Answer. d

Copyright 2014 Delhi Academy of Medical Sciences, All Rights Reserved.

10/119

(26).

Which of the following drugs produce rapid induction of sleep with least influence on sleep architecture
a. Zaleplon
b. Diazepam
c. Chlorpromazine
d. Haloperidol
Solution. (a) Zaleplon
Ref.:Harrison-175
Sol :
Benzodiazepine-receptor agonists arethe most effective and well-tolerated class of medications for insomnia.
The broad range of half-lives allows flexibility in the duration of sedative action.
The most commonly prescribed agents in this family are :
Zaleplon (5-20 mg.) witha half-life of 1-2 h;
Zolpidem (5-10), and Triazolam(0, 125-0.25 mg), with half-lives of 2-3 h;
Eszopiclone (1-3 mg), with a half-life of 5.5-8 h; and
Temazepam (15-30 mg) and Lorazepam(0.5-2 mg), with half-lives of 6-12 h.
Your Answer. a
Correct Answer. a

(27).

Which of the following local anesthetics is an acetanilide derivative?


a. Tetracaine
b. Lidocaine
c. Cocaine
d. Procaine
Solution. (b) Lidocaine
Ref Read the text below
Sol:

Lidocaine belongs to derivatives of acetanilide, realizes local anesthetic and antiarrhythmic actions.

Is used for all kinds of local anesthesia: terminal, infiltration, conduction anesthesia. From mucous membranes it is soaked up
quickly enough and it can have a systemic effect.
Epinefrine is an adrenergic agonist, narrowingvessels and extending the action of lidocaine
Your Answer. b
Correct Answer. b

Copyright 2014 Delhi Academy of Medical Sciences, All Rights Reserved.

11/119

(28).

Indicate the anesthetic agent of choice in patient with a liver disease:


a. Lidocaine
b. Bupivacaine
c. Atracurium
d. Etidocaine
Solution. (c) Atracurium
Ref Read the text below
Sol:

A Liver dysfunction may result in prolonged duration of action of anesthetic and neuromuscular blocking agents because of
altered metabolism or clearance rates. Isoflurane is the preferred anesthetic agent for patients with cirrhosis, while
methoxyflurane, chloroform and halothane should be avoided if possible.

In addition, the actions of neuromuscular blocking agents may be prolonged due to increased biliary excretion and decreased
pseudocholinesterase activity.

Therefore, atracurium is the drug of choice in patients with liver disease or biliary obstruction, and doxacurium is
recommended for prolonged surgeries. Oxazepam and lorazepam are the most suitable anxiolytic sedatives, whereas fentanyl and
sufentanyl should be the first-line narcotics.

In contrast, morphine, meperidine and barbiturates can precipitate hepatic encephalopathy and should be avoided llergic
reactions to procaine are usually not in response to procaine itself, but to PABA.
Your Answer. d
Correct Answer. c

(29).

Which one of the following when given IV can cause intense pain:
a. Thiopentone
b. Ketamine
c. Fentanyl
d. Midazolam
Solution. (a) Thiopentone
Ref Read the text below
Sol:

Thiopentone, when givenIV, if extravasated, produces intense pain. Also, inadvertent intrarterial injection causes severe pain,
necrosis and gangrene.
Your Answer. a
Correct Answer. a

Copyright 2014 Delhi Academy of Medical Sciences, All Rights Reserved.

12/119

(30).

The following tendons pass under extensor retinaculum except:


a. Extensor digitorum brevis
b. Extensor hallucis
c. Tibialis anterior
d. Superficial peroneal nerve
Solution. (d) Superficial peroneal nerve
Ref: Read the text below
Sol :
Structures that pass anterior to the extensor retinacula on the anterior surface of the ankle (medial to lateral) :
Saphenous nerve and great saphenous vein (anterior to medial mallcolus)
Superficial peroneal nerve.
Structures that pass deep to the extensor retinacula (medial to lateral) :
Tibialis anterior tendon.
Extensor hallucis longus tendon.
Anterior tibial artery
Deep peroneal nerve.
Extensor digitorum longus tendon.
Peroneus tertius
Your Answer. d
Correct Answer. d

(31).

The structures that passes between middle cerebellar peduncle and pons is :
a. Trigeminal nerve
b. Vagus nerve
c. Olfactory nerve
d. Optic nerve
Solution. (a) Trigeminal nerve
Ref: Read the text below
Sol :
The pons is dominated by the massive, transversely oriented structure on its ventral surface.
This part of the pons is called the basal pons and looks like a bridge interconnecting the two cerebellar hemispheres.
It does not, however, interconnect them.
Rather, many of the fibres descending in a cerebral peduncle synapse in scattered nuclei of the ipsilateral half of the basal pons.
These nuclei in turn project their fibres across the midline.
They then funnel into the middle cerebellar peduncle and finally enter the cerebellum.
The entry and exit of Cranial Nerves around the pons.
The trigeminal nerve (CN V) enters the brain-stem at the midpons.
Your Answer. a
Correct Answer. a

(32).

Which of the following muscles is responsible for protruding the tongue ?


a. The styloglossus
b. The hyoglossus
c. The genioglossus
d. The palatoglossus
Solution. (c) The genioglossus
Ref: Read the text below
Sol :

Both genioglossus muscles pull the tongue forward. Remember that if only one of the muscle functions, the tip of the tongue points
to the side of the resting or paralysed muscle.
Your Answer. c
Correct Answer. c

Copyright 2014 Delhi Academy of Medical Sciences, All Rights Reserved.

13/119

(33).

Regarding the uterus the following statements are true except :


a. The nerve supply of the uterus is derived from the inferior hypogastric plexus
b. The posterior surface of the cervix is completely covered with the peritoneum
c. The long axis of the uterus is usually bent posteriorly on the long axis of the vagina.
d. The internal iliac veins drain the uterine veins
Solution. (c) The long axis of the uterus is usually bent posteriorly on the long axis of the vagina.
Ref: Read the text below
Sol :

The long axis of the uterus is normally bent anteriorly on the long axis of the vagina.

The angle so formed,usually about 90, is called the angle of anteversion. So the uterus is said to be anteverted
Your Answer. c
Correct Answer. c

(34).

Central sulcus is an example of :


a. Limiting sulcus
b. Complete sulcus
c. Axial sulcus
d. None of the above
Solution. (a) Limiting sulcus
Ref: Read the text below
Sol :

Central sulcus is an example of Limiting sulcus .

It separates the motor from sensory areas.

Calcarine & Collateral sulcus are examples of Complete sulcus.

The calcarine sulcus is also an eg of Axial sulcus ( grows in the axis of visual lobe ).
Your Answer. b
Correct Answer. a

(35).

Callosal sulcus contains one of the following :


a. Deep middle cerebral vein
b. Middle cerebral artery
c. Perficial middle cerebral vein
d. Anterior cerebral artery
Solution. (d) Anterior cerebral artery
Ref: Read the text below
Sol :

The callosal sulcus contains the anterior cerebral artery.

Middle cerebral artery is present in the lateral sulcus.


Your Answer. b
Correct Answer. d

Copyright 2014 Delhi Academy of Medical Sciences, All Rights Reserved.

14/119

(36).

The somesthetic association areas of brain include:


a. 3
b. 7
c. 6
d. 17
Solution. (b) 7
Ref: Read the text below
Sol :

The primary sensory areas are no 3,1,2.

The sensory association areas include areas no 5 & 7.


Your Answer. b
Correct Answer. b

(37).

Arcuate fasciculus connects :


a. Temporal & frontal lobes
b. Frontal & occipital lobes
c. Temporal & parietal lobes
d. Frontal & parietal lobes
Solution. (a) Temporal & frontal lobes
Ref: Read the text below
Sol :

White matter of CNS are classified as Association, Projection & Commissural fibres. Uncinate fasciculus is an example of
association fibers.

They connect frontal ( Brocas area no 44, 45 ) & temporal ( Wernikes area no 22 ) lobes.
Your Answer. a
Correct Answer. a

(38).

Tract present in the middle cerebellar peduncle is


a. Ant spinocerebellar
b. Pontocerebellar
c. Olivocerebellar
d. Reticulocerebellar
Solution. (b) Pontocerebellar
Ref: Read the text below
Sol :

The olivocerebellar fibres passes through the inferior cerebellar peduncle, pontocerebellar through MCP.

The anterior spinocerebellar & reticulocerebellar fibres pass theough SCP.


Your Answer. c
Correct Answer. b

Copyright 2014 Delhi Academy of Medical Sciences, All Rights Reserved.

15/119

(39).

A right-leg paralysis characterized as spastic may indicate damage to :a. The medial aspect of the left precentral gyrus
b. The lateral aspect of the right precentral gyrus
c. The anterior horn cell
d. The right fasciculus gracilis
Solution. (a) The medial aspect of the left precentral gyrus
Ref: Read the text below
Sol :

The precentral gyrus of the frontal lobe is the primary motor cortex giving rise to the descending corticospinal tract.

The homuncular organization localizes lower limb innervation to the medial aspect of the precentral gyrus. The descending
corticospinal tract has its cells of origin in the contralateral cerebral cortex.

Spasticity is one of the cardinal findingsin an upper motor neuron syndrome. If the anterior horn cells are involved, the
presentation has features of a lower motor neuron syndrome, specifically, a flaccid paralysis.

The fasciculus gracilis is an ascending sensory pathway. A lesion of this pathway may produce ataxia but not paralysis.
Your Answer. a
Correct Answer. a

(40).

An aneurysm near the bifurcation of the internal carotid artery into the anterior and middle cerebral arteries produces deficits.
With which of the following cranial nerves are they most likely to be associated ?
a. Oculomotor
b. Trigeminal
c. Trochlear
d. Optic
Solution. (d) Optic
Ref: Read the text below
Sol :

The optic nerve (and chiasma) is located lateral to the bifurcation of the internal carotid into its two main terminal branches.

An enlargement of the vessel may encroach upon the optic nerve and produce visual field deficits.
Your Answer. b
Correct Answer. d

(41).

The nucleus that gives rise to motor fibers that innervate muscles of larynx, pharynx, and palate
is the:a. Hypoglossal nucleus
b. Nucleus ambiguus
c. Nucleus solitarius
d. Dorsal motor nucleus of the vagus
Solution. (b) Nucleus ambiguus
Ref: Read the text below
Sol :

The nucleus ambiguus gives rise to the motor innervation of larynx, pharynx, and palate. The hypoglossal nucleus innervates the
tongue muscle.

The nucleus solitarius is innervated by visceral sensory neuronsfrom the viscera.

The dorsal motor nucleus of the vagus is comprised of parasympathetic preganglionicneurons that innervate the viscera.

The Edinger-Westphal nucleus is comprised of preganglionicparasympathetic neurons that innervate the smooth muscles of the
eye (sphincter pupillae and ciliary apparatus).
Your Answer. d
Correct Answer. b

Copyright 2014 Delhi Academy of Medical Sciences, All Rights Reserved.

16/119

(42).

All are functions of latissimus dorsi except:


a. External rotation of shoulder
b. Extension
c. Adduction
d. Medial rotation
Solution. (a) External rotation of shoulder
Ref: Read the text below
Sol :

The latissimus dorsi is responsible for extension, adduction, transverse extension also known as horizontal abduction,
flexion from an extended position, and (medial) internal rotationof the shoulder joint. It also has a synergisticrole in extension
and lateral flexion of the lumbar spine.

Most latissimus dorsi exercises concurrently recruit the teres major, posterior fibres of the deltoid, long head of the triceps
brachii, among numerous other stabilizing muscles.

Compound exercises for the 'lats' typically involve elbow flexion and tend to recruit the biceps brachii, brachialis, and
brachioradialisfor this function.
Depending on the line of pull, the trapeziusmuscles can be recruited as well; horizontal pulling motions such as rows recruit
both latissimus dorsi and trapezius heavily.
Your Answer. d
Correct Answer. a

(43).

Muscle causing Depression of mandible is


a. Medial pterygoid
b. Lateral pterygoid
c. Masseter
d. Buccinator
Solution. (b) Lateral pterygoid
Ref: Read the text below
Sol :
Actions of Lateral Pterygoid Muscle
Pulls the neck of the mandible forward with the articular disc during the process of opening the mouth.
Acting with the medial pterygoid of the same side, it pulls the neck of the mandible forward with the articular disc, causing the
jaw to rotate around the opposite condyle, as in the movement of chewing.
The posterior fibers of the temporalis retract the mandible, but as it is not given in the choice we shall go for lateral pterygoid.
Your Answer. b
Correct Answer. b

Copyright 2014 Delhi Academy of Medical Sciences, All Rights Reserved.

17/119

(44).

Muscle attached to the roof of the orbit is:


a. Superior oblique
b. Superior rectus
c. Lateral rectus
d. Superior Pterygoid
Solution. (a) Superior oblique
Ref: Read the text below
Sol :
Roof of orbit
It is concave from side to side.
It is formed :
Mainly by the orbital plate of the frontal bone; and
Is completed posteriorly by the lesser wing of the sphenoid.
Relations
It separates the orbit from the anterior cranial fossa.
The frontal air sinus may extend into its anteromedial part.
Named Features
The lacrimal fossa placed anterolaterally lodges the lacrimal gland.
The optic canal lies posteriorly, at the junction of the roof and medial wall.
The trochlear fossa lies antero-medially. It provides attachement to the fibrous pully (trochlea) for the tendon of the superior oblique
muscle
Your Answer. a
Correct Answer. a

(45).

Sudden standing evokes the baroreceptor reflex. Which one of the following will be greater after a person suddenly stands up than it
was before the person stood?
a. The end-diastolic volume
b. The renal blood flow
c. The venous return
d. The ejection fraction
Solution. (d) The ejection fraction
Ref: Read the text below
Sol :

When a person rises suddenly, blood pools in the dependent portions of the body, causing decreases in venous return, left
ventricular end-diastolic volume, stroke volume, and pulse pressure.

The reduced stroke volume leads to a drop in cardiac output and, therefore, a drop in blood pressure.

Decreased blood pressure produces the baroreceptor reflex, leading to an increase in sympathetic activity, which increases total
peripheral resistance, cardiac contractility,heart rate, and ejection fraction.

These changes in the cardiovascular system return blood pressure toward normal.
Your Answer. d
Correct Answer. d

Copyright 2014 Delhi Academy of Medical Sciences, All Rights Reserved.

18/119

(46).

The enzyme ultimately responsible for the formation of fibrin monomers is


a. Heparin
b. Plasminogen
c. Thrombin
d. Kininogen
Solution. (c) Thrombin
Ref: Read the text below
Sol :

Thrombin, the enzyme ultimately responsible for the formation of fibrin monomers, is generated from prothrombin by
activated factor X.

Activation of factor X occurs via both extrinsic and intrinsic pathways. Kininogens are enzymes responsible for the production
of peptides (kinins) associated with inflammation.

Plasminogen is the inactive precursor of plasmin, the proteolytic enzyme involved in clot dissolution. Heparin is an anticlotting
agent that is found on endothelial cell surfaces.
Your Answer. c
Correct Answer. c

(47).

Which one of the following is the best index of after load?


a. Left ventricular end-diastolic pressure
b. Left ventricular mean systolic pressure
c. Pulmonary capillary wedge pressure
d. Total peripheral resistance
Solution. (b) Left ventricular mean systolic pressure
Ref: Read the text below
Sol :

The afterload is the force that the sarcomeres must overcome in order to shorten during systole. According to the law of
Laplace, this force is proportional to the pressure (P) and radius (r) of the ventricle during ejection (force P r).

The mean left ventricular systolic pressure would therefore be the best index of afterload.

Although total peripheral resistance can influence afterload by causing changes in mean arterial blood pressure, these factors
can only influence afterload by causing a change in ventricular pressure.

Pulmonary capillary wedge pressure and left ventricular end-diastolic pressure are estimates of the volume of blood in the
ventricle during diastole and are indices of preload.
Your Answer. d
Correct Answer. b

(48).

The principal paracrine secretion involved in the inhibitory feedback regulation of gastric acid secretion is
a. Gastrin
b. Somatostatin
c. Histamine
d. Enterogastrone
Solution. (b) Somatostatin
Ref: Read the text below
Sol :

Somatostatin, located within the SS cells of thegastric antral mucosa, is the principal paracrine secretion involved in the
inhibitory feedback of gastric acid secretion.

Somatostatin is released in response to an increase in hydrogen ions.

In humans, gastric acid secretion by the parietal cell occurs in response to excitatory neural (acetylcholine), hormonal (gastrin), and
paracrine (histamine) stimuli. Inhibitory feedback regulation of acid output also involves neural (enterogastric reflex), hormonal
(enterogastrone), and paracrine (somatostatin) influences.
Your Answer. b
Correct Answer. b

Copyright 2014 Delhi Academy of Medical Sciences, All Rights Reserved.

19/119

(49).

Inactivation of the sodium-potassium pump will cause


a. An increase in the intracellular volume
b. An increase in the intracellular potassium concentration
c. Hyperpolarization of the membrane potential
d. An increase in the excitability of nerve cells
Solution. (a) An increase in the intracellular volume
Ref: Read the text below
Sol :

The sodium-potassium pump (Na+,K+-ATPase) maintains a low intracellular Na+ concentration. Decreasing the activity of
Na+,K+-ATPase leads to an accumulation of intracellular Na+.

The osmotic activity of the increased intracellular Na+ pulls water into the cell, causing the cell to swell.

As Na+ enters the cell and K+ leaves the cell, the concentration gradients across the cell membrane are dissipated and the
cell depolarizes.

Depolarization inactivates Na+ channels, leading to a decrease in membrane excitability.


Your Answer. d
Correct Answer. a

(50).

The amount of potassium excreted by the kidney will decrease if


a. Distal tubular flow increases
b. Circulating aldosterone levels increase
c. Dietary intake of potassium increases
d. Na+ reabsorption by the distal nephron decreases
Solution. (d) Na+ reabsorption by the distal nephron decreases
Ref: Read the text below
Sol :

The amount of potassium excreted is controlled by the amount of potassium secreted by the distal tubule.

Potassium secretion is a passiveprocess that depends on the electrochemical gradient between the distal tubular cells and the
tubular lumen and the permeability of the luminal cells to potassium.

By inhibiting Na+ reabsorption, the intraluminal potential becomes less negative and K+ secretion is reduced.

K+-sparing diuretics such as amiloride act in this fashion. Aldosterone increases the intracellular potassium concentration by
augmenting the activity of the Na-K pump and increasing the potassium permeability of the luminal membrane.
Increasing dietary intake increases the plasma potassium concentration, which in turn stimulates aldosterone production. Increasing the
rate of distal tubular flow increases secretion by maintaining a low potassium concentration within the filtrate and thus increasing
the electrochemical gradient for potassium. The presence of nonreabsorbable organic anions will increase the electrochemical
gradient for K+ and therefore increase K+ excretion.
Your Answer. d
Correct Answer. d

Copyright 2014 Delhi Academy of Medical Sciences, All Rights Reserved.

20/119

(51).

The glomerular filtration rate will increase if


a. Sympathetic nerve activity to the kidney increases
b. The afferent arteriolar resistance increases
c. The efferent arteriolar resistance decreases
d. The plasma protein concentration decreases
Solution. (d) The plasma protein concentration decreases
Ref: Read the text below
Sol :

The amount of fluid filtered by the glomerulus (the GFR) will increase if filtration pressure rises or plasma oncotic pressure
decreases.

Decreasing the plasma protein concentration decreases the capillary oncotic pressure, which leads to an increase in GFR.

If the afferent arteriolar resistance increases without an increasein perfusion pressure, the pressure within the glomerulus
will decrease, causing a decrease in GFR. A similar result will occur if the efferent arteriolar resistance falls.

Increasing sympathetic nerve activity to the kidney decreases renal blood flow and increases Na+ and water
reabsorption from the proximaltube, making it easier for the kidney to excrete concentrated urine.
Your Answer. a
Correct Answer. d

(52).

Phase-4 depolarization of SA nodal cells is caused by


a. An increase in the flow of sodium into the cell
b. A decrease in the flow of potassium out of the cell
c. An increase in the activity of the Na/Ca exchanger
d. A decrease in the flow of chloride out of the cell
Solution. (a) An increase in the flow of sodium into the cell
Ref: Read the text below
Sol :

Phase-4 depolarization is caused by the activationof a Na+ channel. The channel is called the funny channel because it is
activated when the membrane hyperpolarizes in contrast to the Na channel responsible for the action potential, which is activated
when the cell depolarizes.

Potassium conductance decreases during phase-4 depolarization and thus the flow of potassium out of the cell is diminished.
However, this change in potassium current is not responsible for phase-4 depolarization.

Chloride conductance does not change during phase 4. The Na/Ca exchanger maintains low intracellular calcium at rest and
may reverse its direction and pump calcium into the cell during phase 2 of the cardiac action potential.
However, neither the Na/Ca exchanger nor the Na-K pump is involved in phase-4 depolarization
Your Answer. a
Correct Answer. a

(53).

Cardiovascular changes that occur during inspirationinclude decreased


a. Right ventricular filling
b. Right ventricular output
c. Pressure gradient from extrathoracic veins to the right atrium
d. Systemic blood pressure
Solution. (d) Systemic blood pressure
Ref: Read the text below
Sol :

During inspiration, blood flow into the pulmonarycirculation increases.

The increased volume of blood in the right ventricle pushes the intraventricular septum toward the left ventricle, resulting in a
decreased filling of the left ventricle.
The decrease in left ventricular preload results in a decreased left ventricular stroke volume and a decrease in systemic blood
pressure
Your Answer. d
Correct Answer. d

Copyright 2014 Delhi Academy of Medical Sciences, All Rights Reserved.

21/119

(54).

Self-stimulation could be induced experimentally most effectively from which part of brain ?
a. Periaqueductal area (Area around the aqueduct of Sylvius)
b. Periventricular region of hypothalamus
c. Medial forebrain bundle
d. Mesencephalon
Solution. (a) Periaqueductal area (Area around the aqueduct of Sylvius)
Ref:Read the text below
Sol :
The options in the above question are all components of the endogenous pain suppression system (also called the descending pain
inhibiting pathway), therefore the term self stimulation here probably refers to this endogenous pain suppression or analgesia system in
the brain and spinal cord,
The analgesia system consists of three components:
(1) Periacqueductal gray and periventricular areas of the mesencephalon and upper pons that surround the acqueduct of
Sylvius and portions of the 3rd and 4th ventricles.
Neurons from these areas secrete enkephalin at their endings, sending signals to
(2)Raphe magnus nucleus in the lower pons and upper medulla, and the nucleus paragigantocellularis located laterally in the medulla.
From these nuclei, serotonergic neurons travel down the dorsolateral columns in the spinal cord to
(3) Enkephalinergic neurons in the dorsal horn of the spinal cord - enkephalin from these neurons causes both pre and post
synaptic inhibition of the pain pathway.

Electrical stimulus either in the periacqueductal gray or raphe magnus nucleus can suppress many strong pain signals
entering by way of the dorsal spinal roots.

Also stimulation of other areas, such as the periventricular area of hypothalamus and to a lesser extent, the medial forebrain bundle,
also in the hypothalamus, excite the PAG, thereby causing suppression of pain transmission.
Your Answer. a
Correct Answer. a

(55).

Ventrolateral cordotomy is helpful in pain relief in the right leg as it interrupts


a. Right corticospinal tract
b. Left ventral spinothalamic tract
c. Left lateral spinothalamic tract
d. Right lateral spinothalamic tract
Solution. (c) Left lateral spinothalamic tract
Ref:Read the text below
Sol :

Lateral spinothalamic tract carries pain and temperature from the opposite side.

To relieve pain in the right leg ventrolateral cordotomy will interrupt the left lateral spinothalamic tract.
Your Answer. b
Correct Answer. c

Copyright 2014 Delhi Academy of Medical Sciences, All Rights Reserved.

22/119

(56).

Intrapleural pressure is negative because


a. Chest wall and lungs recoil in opposite directions to each other.
b. Transpulmonary pressure is negative
c. Intrapulmonary pressure is always negative
d. Pulmonary collapse is prevented by surfactants
Solution. (a) Chest wall and lungs recoil in opposite directions to each other.
Ref:Ganongs Review of Medical Physiology; 23/e, pg 591-592
Sol :

The pressure in the space between the two layers of pleura is subatmospheric: -2.5 mms of Hg (w.r.t atmosphere) at the start of
inspiration, decreasing to -6 mms of Hg at the end of inspiration.

This is because the lungs and chest wall are elastic structures with a tendency to recoil in opposite directions- lungs tend
to recoil towards the inside and the chest wall tends to recoil towards the inside.

The lungs are stretched when they expand during inspiration, and at the end of quiet expiration (functional residual capacity) their
tendency to recoil from the chest wall is just balanced by the tendency of the chest wall to recoil in the opposite direction (at
FRC the lung recoil is equal to and is balanced by the chest wall recoil- FRC is also k/a relaxation volume of the lungs).

If the chest wall is opened, the lungs collapse; and if the lungs lose their elasticity, the chest expands and becomes barrel shaped.
Your Answer. a
Correct Answer. a

(57).

All of the following associations of hormones site of production and chemical structure are correct except
a. Androgens zona reticularis of adrenal cortex steroid
b. Vasopressin zona glomerulosa of adrenal cortex steroid
c. Cortisol zona fasiculata of adrenal cortex steroid
d. Norepinephrine adrenal medulla cate cholamine
Solution. (b) Vasopressin zona glomerulosa of adrenal cortex steroid
Ref:Read the text below
Sol :
Vasopressin and oxytocinare peptides that are synthesized in the hypothalamus and secreted from the posterior pituitary.
The adrenal cortex synthesizes three types of steroid hormones including glucocorticoids (e.g., cortisol) in the zona fasiculata,
mineralocorticoids (e.g., aldosterone) in the zona glomerulosa, and androgens in the zona reticularis.
The adrenal medulla synthesizes catecholamines including norepinephrine. The pancreatic lslet secrete the peptide hormones insulin
(-cells) and glucagon (-cells), as well as other hormones.
Your Answer. b
Correct Answer. b

(58).

All of the following statements regarding calcitonin are true except


a. Calcitonin is synthesized in C sells located primarily in the thyroid gland
b. Calcitonin increases serum phosphate concentrations
c. Calcitonin lowers serum calcium concentrations
d. Calcitonin is synthesized as a large precursor protein
Solution. (b) Calcitonin increases serum phosphate concentrations
Ref:Read the text below
Sol :
Calcitonins two major biologic effectsare to lower both serum calcium and serum phosphate concentrations.
Calcitonin is synthesized in Ccells of neuroendocrine origin, which are located primarily in the thyroid gland and, to a lesser
extent, in the thymus.
Your Answer. c
Correct Answer. b

Copyright 2014 Delhi Academy of Medical Sciences, All Rights Reserved.

23/119

(59).

All of the following statements regarding 1,25-(OH)2D are true except


a. Elevated parathyroid hormone (PTH) acts on renal tubules to increase the formation of 1,25-(OH)2D
b. 1,25-(OH)2D is the biologically active from of vitamin D
c. High levels of serum phosphate act on renal tubules to increase the formation of 1,25-(OH)2D
d. It increases the intestinal absorption of calcium and phosphate
Solution. (c) high levels of serum phosphate act on renal tubules to increase the formation of 1,25-(OH)2D
Ref:Read the text below
Sol :
Low serum phosphate levelsand elevated parathyroid hormone (PTH act on renal tubules to increase the formation of
1,25-(OH)2D (dihydroxy vitamin D).
Dihydroxyvitamin D isthe biologically active form of vitamin D, and the last enzymatic step in its formation is the principal site of
regulation.
This biologically active form of vitamin D also causes an increase in the intestinal absorption of calcium and phosphate so that
serum calcium concentrations return to normal.
Your Answer. c
Correct Answer. c

(60).

Fluoroquinolones (ciprofloxacin) act by inhibiting bacterial DNA gyrase. What is the function of DNA gyrase?
a. Unwind the DNA
b. Relieve torsional strain on DNA
c. Initiation of transcription
d. Synthesis of Okazaki Fragments
Solution. (b) Relieve torsional strain on DNA
Ref Read the text below
Sol:
TOPOISOMERASE
Relieves the torsional strain produced by unwinding of DNA

The enzyme causes nicks in the DNA by transesterification and not by hydrolysis of phosphodiester bond

Topoisomerase I:
-Makes single stranded nicks in the DNA
- Relieves positive and negative supercoils in eukaryotes
- Requires no ATP

Topoisomerase II:
-Relives positive and negative supercoiling
-Can introduce negative supercoiling
-Requires ATP
-DNA gyrase is a subset of Topoisomerase II.
Your Answer. b
Correct Answer. b

Copyright 2014 Delhi Academy of Medical Sciences, All Rights Reserved.

24/119

(61).

Which among following is not an inhibitor of complex I of electron transport chain?


a. Barbiturates
b. Piercidin A
c. Rotenone
d. Antimycin
Solution. (d) Antimycin
Ref Read the text below
Sol:
INHIBITORS OF RESPIRATORY CHAIN
Inhibitors of electron transport
Via complex-I
Via complex-II

Barbiturates
Percidin-A
Rotenone
Malonate
TTFA
CarboxinE

BAL
Via complex-III Antimycin-A
Cyanide
CO
Via complex -IV H2S
Sodium azide
Your Answer. c
Correct Answer. d

(62).

All of the given statements are true except:


.
a. Heterochromatin is the relatively uncondensed portions of DNA
b. Histones are positively charged because of the high content of basic amino acids in them
c. Methylation of DNA will increase the condensed portions
d. Euchromatin represents the transcriptionally active portions of the DNA
Solution. (a) Heterochromatin is the relatively uncondensed portions of DNA
Ref Read the text below
Sol:
HETEROCHROMATIN

Transcription ally inactive DNA


Condensed

Formed by histone deacetylation and DNA methylation

Has very high methyl cytosine content

2 types:
-Constitutive: always occurs in condensed form; seen in centromere and ends of telomere.
-Facultative: at times condensed and at other times actively transcribed.
Your Answer. a
Correct Answer. a

Copyright 2014 Delhi Academy of Medical Sciences, All Rights Reserved.

25/119

(63).

True statement about glucokinase is:


a. Km value is higher than normal blood sugar
b. Found in al tissues
c. Glucose-6-phosphate inhibits it
d. Has both glucose-6-phosphatase and kinase activity.
Solution. (a) Km value is higher than normal blood sugar
Ref Read the text below
Sol:

Glucokinase is the Type IV isoenzyme of hexokinase

Glucokinase differs from hexokinasein several important properties


It has a much Higher Km, requiring much higher glucose concentration for half saturation.
Features

Hexokinase (I-III)

Glucokinase (hexokinase 4)

Found in

All tissues including fetal and adult


liver

Adult liver, beta cella of pancreas

Km

Low (0.1 mmol/L)

High (10 mmol/L or 180 mg/dl ogf glucose

Affinity (for glucose)

High

Low

Stability

More

Less(labile)

Phosphorylate

Any hexose

Glucose specific

Allosteric inhibition

Glucose -6- phosphate

Fructose-6- phosphate

Glucose feeding

No change

Increased

Insulin

Induce

Induce

GH, glucocorticoids

Inhibit

Inhibit

Fasting/Diabetes Mellitus

Not much change

Decreased

Your Answer. a
Correct Answer. a

Copyright 2014 Delhi Academy of Medical Sciences, All Rights Reserved.

26/119

(64).

Enzyme marker for golgi apparatus is:


a. 5 nucleotidase
b. LDH
c. Galactosyl transferase
d. Glucokinase
Solution. (c) Galactosyl transferase
Ref Read the text below
Sol:
Organelle

Marker

Nucleus

DNA

Mitochondria

Glutamate dehydrogenase

Endoplasmic reticulum

Glucose-6-phosphatase

Lysosome

Acid phosphatase

Plasma membrane

5 nucleotidase
Na+ K+ ATPase
Adenylyl cyclase

Golgi Apparatus
Cis-golgi
Medial-golgi
Trans-golgi
TGN (Trans golgi network)

Galactosyl transferase I

Golgi mannosidase II

Galactosyl transferase

Sialyl transferase

Peroxisomes

Catalase

Cytosol

Lactate dehydrogenase

Intermitochondrial
membrane

ATP synthase

Your Answer. c
Correct Answer. c

(65).

Which arm binds the aminoacyl tRNA to the ribosomal surface?


a. Acceptor arm
b. Anticodon arm
c. DHU arm
d. Pseudouridine arm
Solution. (d) Pseudouridine arm
Ref Read the text below
Sol:
Pseudouridine arm (T C)
.(T C) Stands for ribothimine, pseudouridine, cytosine
Involved in the binding of aminoacyl tRNA to the ribosomal surface
Your Answer. b
Correct Answer. d

Copyright 2014 Delhi Academy of Medical Sciences, All Rights Reserved.

27/119

(66).

Uronic acid is formed by:


a. Oxidation of sugars at aldehyde group
b. Oxidation of sugars at last carbon atom
c. Oxidation of sugars at first and last carbon atom
d. Reduction of sugars
Solution. (b) Oxidation of sugars at last carbon atom
Ref Read the text below
Sol:
OXIDATION OF SUGARS

At aldehyde group: forms aldonic acid


At last carbon atom: to uronic acid

At last first and last carbon atom: to saccharic acid


Eg. In case of glucose,

Oxidation of aldehyde group produces gluconic acid

Oxidation of aldehyde group produces glucoronic acid

Oxidation of aldehyde and alcohol group produces gluccharic acid.


Your Answer. b
Correct Answer. b

(67).

A ten year old child with aggressive behavior and poor concentration is brought with presenting complaints of joint pain and
reduced urinary output. Mother gives history of self mutilative behavior, stating that he tends to mutilate his fingers. Which of the
following is likely to be elevated in this child?
a. Blood sugar
b. Serum creatinine
c. Uric acid
d. Blood urea
Solution. (c) Uric acid
Ref Read the text below
Sol:

Self mutilative behavior typically affecting fingers, evidence of joint pain (due to hyperuricemia) And reduced urinary output
together with features of compulsive aggression suggest a diagnosis of Lesch -Nyhan syndrome
Lesch Nyhan syndrome is caused by a complete deficiency of hypoxanthine-guanine phosphoribosyltransferase (HGPRTase).
The excess of purines are catabolized resulting in elevated levels of uricacid.
Your Answer. c
Correct Answer. c

(68).

Heparin is a:
a. Glycoprotein
b. Glycosaminoglycan
c. Lipoprotein
d. Protein
Solution. (b) Glycosaminoglycan
Ref Read the text below
Sol:

Heparin, the anticoagulant is a mucopolysaccharide (glycosaminoglycan).


It binds to a specific cationic site of antithrombin, including a conformational change and promoting its binding to thrombin as
well as to its other substrates. This is the basis for the use of heparin to inhibit coagulation.
Your Answer. b
Correct Answer. b

Copyright 2014 Delhi Academy of Medical Sciences, All Rights Reserved.

28/119

(69).

All are true about serine proteases except:


a. They have serine, histidine, aspartate in their active site
b. Different serine proteases differ in their target cleavage site
c. Renin is a serine protease
d. Trypsin and chymotrypsin are serine proteases
Solution. (c) Renin is a serine protease
Ref Read the text below
Sol:
SERINE PROTEASES
Active site : contains serine, histidine, asparate

Examples are:
Trypsin

Chymotrypsin
Elastase
Thrombin
Your Answer. c
Correct Answer. c

(70).

41 Denaturation of DNA is PCRis carried out by heating to a temperature of:


a. 40C
b. 60C
c. 76C
d. 94C
Solution. (d) 94C
Ref Read the text below
Sol:
POLYMERASE CHAIN PEACTION

It is a test tube method for amplifying a selected DNA sequence

The method can be used to amplify DNA sequences from any source:bacterial, viral, plant or animal

PCR utilizes DNA polymerase to repetitively amplify targeted portions of DNA Requirements

DNA to be amplified

Primer RNA

Deoxynucleotides

DNA polymerase ( Taq polymerase)


PCR: STEPS
1. Primer construction:

It is not necessary to know the nucleotide sequence of the target DNA

But it is necessary to know the nucleotide sequence of short segments on each side of target DNA-flanking sequences
2. Denature the DNA:

The DNA to be amplified is heated to separate the double stranded target DNA into single strands
This is done by heating at 92-96 C for 10 minutes
3. Annealing of primers to ssDNA:

The separated strands are cooled and allowed to anneal to the two primers (one each strand)

Temperature : 45 C for 4 minutes


4. Chain extension:

DNA polymerase and deoxyribonucleoside triphosphates are added to the mixture to initiate the synthesis of two new chains
complementary to the original DNA

Taq polymerase (from thermusaquaticus) is used

At 72 C for variable time


Generally cycles are repeated for 20-30 times

Your Answer. d
Correct Answer. d

Copyright 2014 Delhi Academy of Medical Sciences, All Rights Reserved.

29/119

(71).

All are true about HDL except:


a. Carries cholesterol from peripheral tissues to liver
b. Pre beta HDL is the inactive precursor, which later forms the active HDL
c. It has approteins ApoA, ApoCII, ApoE
d. HDL is the lipoprotein which moves farthest from starting point on electrophoresis
Solution. (b) Pre beta HDL is the inactive precursor, which later forms the active HDL
Ref Read the text below
Sol:
HDL
A. HDL scavenges body cholesterol & blood vessel wall cholesterol to liver. This transport of tissue cholesterol to liver is known
as reverse cholesterol transport.

Class B scavenger receptor B1 (SR-B1) and ATO binding cassette transporter A1 (ABCA1) and G1 (ABCG1) are involved in reverse
cholesterol transport

ABCA1 mediates efflux of cholesterol from peripheral cells

SRB1 mediates uptake of cholesterol from peripheral cells

HDL contains good cholesterol

Pre -HDL is the most potent form of HDL for cholesterol removal from tissues

HDL is synthesized in liver and intestine


C. Apoproteins associated with HDL are ApoA, ApoCII, & ApoE (Arginine rich apolipoprotein)

HDL is the highest density lipoprotein (maximum protein content & least lipid content)
D. HDL has the highest electrophoretic mobility.
Your Answer. d
Correct Answer. b

(72).

In HbM, histidine is replaced by:


a. Valine
b. Serine
c. Lysine
d. Tyrosine
Solution. (d) Tyrosine
Ref Read the text below
Sol:

In hemoglobin M, histidine F8 has been replaced by tyrosine.


The iron of HbM forms a tight ionic complex with the phenolate anion of tyrosine that stabilizes the Fe3+ form.
Your Answer. c
Correct Answer. d

Copyright 2014 Delhi Academy of Medical Sciences, All Rights Reserved.

30/119

(73).

Mitochondrial codons are an exception for the property of universality of codons. For eg: Initiation codon in mammals is AUG,
which codes for methionine. But in mitochondria, methionine is coded by some other codon. Which among the following is that
initiation codon?
a. AAG
b. AUG
c. AUA
d. AGA
Solution. (c) AUA
Ref Read the text below
Sol:

Each codon consists of a sequence of 3 nucleotides

In mammalian cells, AUG codes for methionine (initiation colon)

Stop codons are: UGA, UAG and UAA.


In mitochondria:
AUA codes for methionine (mitochondrial initiation codon)
UGA codes for tryptophan
AGA and AGG serve as chain terminators
In prokaryotes, AUG, the initiation codon codes for Formyl methionine
The Shine-Dalgarno (SD) sequence, is a ribosomal binding site in prokaryotic mRNA, generally located around 8 bases upstream of the
start codon AUG. The Shine-Dalgarno sequence helps recruit the ribosome to the mRNA to initiate protein synthesis by aligning it with
the start codon.
Features of Genetic code:
I.
Degenerate: There are 61 codons for 20 amino acids
I.
This implies multiple codons decode the same amino acid
II.
Unambiguous: Given a specific codon, only a single amino acid is indicated
III.
Non-overlapping
IV.
No punctuation
V.
Universal
Your Answer. c
Correct Answer. c

(74).

Which of the following statements about facilitated diffusion is true?


a. It is a form of active transport
b. It requires a carrier protein
c. Rate of transport is proportionate to the concentration gradient
d. Requires creatine phosphate
Solution. (b) It requires a carrier protein
Ref Read the text below
Sol:
A.Facilitated diffusion is a form of passive transport mediated through carrier.
C. Rate of transport is not always proportionate to the concentration gradient. Rate of transport is proportionate to the concentration
gradient only at low concentrations.
At high concentrations, when the carrier protein becomes saturated, rate of transport becomes constant.
D. Facilitated diffusion does not require a source of energy.
Your Answer. b
Correct Answer. b

Copyright 2014 Delhi Academy of Medical Sciences, All Rights Reserved.

31/119

(75).

All of the following statements are true regarding reversible cell injury except
a. Formation of amorphous densities in the mitochondrial matrix
b. Diminished generation of adenosine triphosphate (ATP)
c. Formation of blebs in the plasma membrane
d. Detachment of ribosomes from the granular endoplasmic reticulum
Solution. (a) Formation of amorphous densities in the mitochondrial matrix
Ref.:Read the text below
Sol :
Formation of Amorphous densities in mitochondrial matrix is a feature of irreversible injury and not reversible injury.
Your Answer. c
Correct Answer. a

(76).

Which of the following organelles plays a pivotal role in apoptosis?


a. Mitochondria
b. Endoplasmic reticulum
c. Nucleus
d. Golgi apparatus
Solution. (a) Mitochondria
Ref.:Read the text below
Sol :
Mitochondria are the most important organelles involved in apoptosis initiation and regulation.
Significant changes can be observed in mitochondrial physiology and morphology during apoptosis.
Your Answer. a
Correct Answer. a

(77).

In apoptosis, Apaf-1 is activated by release of which of the following substances from the mitochondria?
a. Bcl 2
b. Bax
c. Bcl Xl
d. Cytochrome C
Solution. (d) Cytochrome C
Ref.:Read the text below
Sol :
Apoptosis is induced by a cascade of molecular events all of the which culminate in the activation of caspases.
In the cytosol cytochrome C binds to a protein called Apaf I (Apoptosis activating factor 1) and the complex activates caspase 9
Your Answer. a
Correct Answer. d

Copyright 2014 Delhi Academy of Medical Sciences, All Rights Reserved.

32/119

(78).

All of the following statements regarding amyloidosis are true except


a. Multiple myeloma shows AL type deposits
b. Secondary amyloidosis shows AA type deposits
c. Renal amyloidosis commonly presents with hypertension
d. Renal amyloidosis commonly presents with mild proteinuria
Solution. (c) Renal amyloidosis commonly presents with hypertension
Ref.:Robbins - 252
Sol :
Most common presentation is proteinuria.
Hypertension may occur but it is not common, it occurs in 20-50% of the patients.
Your Answer. d
Correct Answer. c

(79).

Amyloid deposits stain positively with all the following except


a. Congo-red
b. Crystal violet
c. Methenamine silver
d. Thioflavin T
Solution. (c) Methenamine silver
Ref.:Robbins - 89
Sol :
Staining Characteristics of Amyloid
Stain on Gross

Stain used Lugols iodine


Appearance : Imparts purple colour, on addition of dilute sulfuric acid turns
blue.

Hemotoxylin & Eosin

Stain used Hemotoxylin & Eosin


Appearance On light microscopy appear as extracellular homogeneous
structure less eosinophillic hyaline material

Metachromatic stains (Rosamine dyes)

Stains used Methyl violet and crystal violet


Appearance Rose pink coloration of amyloid deposits

Congo red and polarized light (All types of amyloid


have affinity for congo red stain)

Stain used Congo red


Appearance Ordinary light Pink
Polarised Apple green birefringence light

Fluoroscent stains

Stain used Thioflavin T


Appearance yellow under ultraviolet light

Immunohistochemistry

Stain used Various antibody stains against the specific antigenic protein
types e.g. Anti
AP, anti AA, anti Lambda, anti kappa

Non specific stains

Stain used : Toluidine blue


Alcian blue
Periodic acid Schiff
Colour imparted : Orthochromatic blue
Blue green colour

Your Answer. b
Correct Answer. c

Copyright 2014 Delhi Academy of Medical Sciences, All Rights Reserved.

33/119

(80).

Single gene disorderis


a. Glycogen storage disease
b. DM
c. HT
d. All of above.
Solution. (a) Glycogen storage disease
Ref.:Robbins - 140
Sol :
Genetic disorder may be of following categories.
Mendelian disorders (single gene disorders)
All mendelian disorders are the result of expressed mutations in single genes of large effect.
Mendelian disorders may be
Autosomal dominant, e.g. Retinoblastoma.
Autosomal recessive, e.gl Glycogen storage disease.
X-linked, e.g. Hemophilia A & B
Diseases with multifactorial inheritance
These diseases are influenced by both genetic and environmental factors.
Examples diabetes and hypertension.
Chromosomal disorders
Result from genomic or chromosomal mutations and are therefore associated with numerical or structural changes in chromosome.
Your Answer. c
Correct Answer. a

(81).

Which of the following chemical mediators of inflammation is an example of a C-X-C or alpha chemokine
a. Lipoxin LXA 4
b. Interleukin IL-8
c. Interleukin IL-6
d. Monocyte Chemoattractant Protein MCP 1
Solution. (b) Interleukin IL-8
Ref.:Read the text below
Sol :
Interleukin-8 (IL-8), growth-related oncogene (GRO) alpha, GRObeta, GROgamma, neutrophil-activating peptide-2 (NAP-2), epithelial
cell-derived neutrophil activating peptide- 78 (ENA-78), and granulocyte chemoattractant protein-2 are potent neutrophil
chemoattractants 40-90% identical in amino acid sequence that comprise a subgroup of human CXC chemokines defined by
the conserved sequence motif glutamic acid-leucine-arginine (ELR).
Neuropeptides
Neuropeptides are secreted by sensory nerves and various leukocytes, and play a role in the initiation and propagation of an
inflammatory response.
Substance P and neurokinin A, belong to a family of tachykinin neuropeptides are produced in CNS & PNS.
Substance P has many biological functions, e.g. pain transmission, regulation of BP, increasing vascular permeability and stimulation
of secretion of endocrine cells.
Nerve fibres containing substance P are prominent in the lung and gastrointestinal tract.
Sensory neurons also produce calcitonin-related gene product, a pro inflammatory molecule, involved in protective host response
of sensing of painful stimuli.
Your Answer. d
Correct Answer. b

Copyright 2014 Delhi Academy of Medical Sciences, All Rights Reserved.

34/119

(82).

Febrile response in CNS is mediated by


a. Bacterial toxin
b. IL-1
c. IL-6
d. Interferon
Solution. (a) Bacterial toxin
Ref.: Readthe text below
Sol :
Pyrogenes
Pyrogenes are substances that cause fever.
Pyrogens may be exogenous or endogenous
Exogenous Bacterial toxins
Endogenous IL-1, TNF IL-6, Interferons, Ciliorys neurotropic factor.
These pyrogenes increase the level of PGE2 in the hypothalamus that elevates the thermoregulatory set point and causes fever.
Your Answer. b
Correct Answer. a

(83).

All of the following conditions may predispose to pulmonary embolism except


a. Protein S deficiency
b. Malignancy
c. Obesity
d. Progesterone therapy
Solution. (d) Progesterone therapy
Ref.:Read the text below
Sol :
Conditions that predisposeto venous thrombosis also increase the risk of pulmonary embolism.
In oral contraceptives deep vein thrombosis is due to estrogen component (not progesterone).
Progesterone only therapy does not cause thrombosis.
All other optionscan cause venous thrombosis and pulmonary embolism.
Your Answer. a
Correct Answer. d

(84).

Patau syndromedue to
a. Trisomy 21
b. Trisomy 18
c. Trisomy 13
d. Trisomy 15
Solution. (c) Trisomy 13
Ref.:Read the text below
Sol :

Trisomies occur when there is a triplet of a specific chromosome instead of the normal number of 2.

In Patau syndrome there is an extra chromosome 13 resulting in multiple and often severe physical and mental
abnormalities.

Some physical characteristics are flattened facial features, malformed and low-set ears, cleft lip and/or palate, prominent
heels, and genital malformations. Often there are problems with the nervous system, including forebrain development, spinal cord
development, mental retardation, and seizures.

Vision and eye problems are common, as well as respiratory and heart defects. Approximately 45% of Patau syndrome
babies die within the first month of life, while the number increases to 70% in the first 6 months.
Your Answer. c
Correct Answer. c

Copyright 2014 Delhi Academy of Medical Sciences, All Rights Reserved.

35/119

(85).

Differential expression of same gene depending on parent of origin is referred to as


a. Genomic imprinting
b. Mosaicism
c. Anticipation
d. Nonpenetrance
Solution. (a) Genomic imprinting
Ref.:Read the text below
Sol :
The phenomenon referred to as genomic imprinting leads to pereferential expression of an allele depending on the parental origin.
Genomic imprinting
Human inherits two copies of each gene, i.e. two alleles, from homologous maternal and paternal chromosomes.
There is no functional difference between the genes derives from mother or the father.
But, with respect to some genes, there are functional differences between the paternal gene and maternal gene.
These differences result from an epigenetic process, called genomic imprinting.
In most cases, genomic imprinting selectively inactivates either the maternal or paternal allele.
Maternal genomic imprinting :
Selective inactivation of maternal allele.
Example is Angelman syndrome.
Paternal genomic imprinting :
Selective inactivation of paternal allele.
Example is Prader Willi syndrome.
Your Answer. a
Correct Answer. a

(86).

A patient presents with respiratory symptoms i.e. cough, hemoptysis and glomerulonephritis. His C-ANCA levels in serum were
found to be raised. The most likely diagnosis is
a. Goodpasteurs syndrome
b. Classic polyarteritis nodosa
c. Wegeners granulomatosis
d. Kawasaki syndrome.
Solution. (c) Wegeners granulomatosis
Ref.:Read the text below
Sol :
Renal and pulmonary symptoms with positive c-ANCA suggest the diagnosis of wegners glomerulonepritis
Clinical syndrome of wegner granulomatosis may be very similar to classical PAN or microscopic polyangitis.
Your Answer. c
Correct Answer. c

Copyright 2014 Delhi Academy of Medical Sciences, All Rights Reserved.

36/119

(87).

Hypersensitivity vasculitisis seen most commonly in


a. Post capillary venules
b. Arterioles
c. Veins
d. Capillaries
Solution. (a) Post capillary venules
Ref.:Read the text below
Sol :
HYPERSENSITIVITY VASCULITIS
In hypersensitivity vasuculitis, postcapillary venules are the most commonly involved vessels, capillaries and arterioles are involved
less frequently.
It is primarily a disease of small vessels.
It is characterized by leukocytoclasis, a term that refers to the nuclear debris remaining from the neutrophils that have infiltrated
in and around the vessels during the acute stages.
Your Answer. b
Correct Answer. a

(88).

Hemorrhagic pericarditisis seen in


a. Uremia
b. TB
c. Neoplasm
d. All
Solution. (d) All
Ref.:Read the text below
Sol :
Hemorrhagic pericarditis
An exudates composed of blood mixed with a fibrinous or supportive effusion accumulate in the pericardial space.
Most common cause is neoplastic involvement of the pericardial space.
Other causes are T.B. Uremia. Cardiac surgery, patients with bleeding diathesis.
Your Answer. d
Correct Answer. d

(89).

In myocardial infarction the infarct acquires hyperemic rim with an yellow centre at
a. 3-7 days
b. 10-20 days
c. 7-14 days
d. 1-2 hours
Solution. (a) 3-7 days
Ref.:Read the text below
Sol :
3 7 days Findings
Gross examination

Hyperemiaat border

Softening yellow-tan center


Histopathology
(light microscopy)

Beginning of disintegration of dead muscle fibers

Necrosis of neutrophils
Beginning of macrophageremoval of dead cells at border
Your Answer. a
Correct Answer. a

Copyright 2014 Delhi Academy of Medical Sciences, All Rights Reserved.

37/119

(90).

The substance that accumulates in cardiae myxoma is


a. Mucopolysaccharides
b. Sialo mucin
c. Gelatin
d. Glycoprotein
Solution. (a) Mucopolysaccharides
Ref.:Read the text below
Sol :
Myxomas are composed of stellate or globular myxoma (lepidic cell, endothelial cells, smooth muscle cells and undifferentiated
cells embedded within abundant acid mucopolysaccharide susbstance.
Your Answer. a
Correct Answer. a

(91).

Lupus anticoagulantsmay cause all of the following except


a. Recurrent abortion
b. False + ve VDRL results
c. Decrease prothrombin time
d. Arterial thrombosis
Solution. (c) Decrease prothrombin time
Ref.: Read the text below
Sol :
Diagnosis of Antiphospholipid syndrome (APS)
At least one clinical criterion and one laboratory criterion must be present for a patient to be classified as having APS.
Clinical criteria :
One or more episodes of arterial, venous or small vessel thrombosis in any tissue or organ confirmed by findings from imaging
studies, Doppler studies or histopathology.
Obstetrical complication : Three or more consecutive abortions before 10 weeks of gestation; or one or more late (> 10 weeks)
abortion, or one or more premature birth due to eclampsia or preeclampsia.
Lab. Criteria :
Presense of one or more of the following on 2 or more occasions at least 12 weeks apart.
Presence of lupus anticoagulant.
Increase prothrombin time
Presence of moderate to high level of anticardioloipin antibody.
Presence of moderate to high level of anti-beta-2 glycoprotein antibodies.
Cardiolipin antibody is used in serological tests for syphilis, so lupus anticoagulant may give false positive test for anticoagulants
syphillis.
Your Answer. c
Correct Answer. c

Copyright 2014 Delhi Academy of Medical Sciences, All Rights Reserved.

38/119

(92).

Platelet are storedin :


a. 20 24 C
b. 4C
c. 2C
d. -20C
Solution. (a) 20 24 C
Ref.:Read the text below
Sol :
Two methods of storage of platelets.
Liquid storage :
Both whole bloodderived and apheresis platelets concentrates may be store for 5 days using following some principles.
Temperature must be 20 24 C
Stroage container must be constructed of a plastic material that allows adequate diffusion of O; to meet cells metabolic needs.
Platelets concentrates must be agitate during storage.
Frozen Storage :
The most widely used method for frozen storage employs controlled rate freezing (1C/min) ; 5% DMSO as a cryoprotective agent.
Your Answer. a
Correct Answer. a

(93).

The most common causative organism for lobar pneumonia is


a. Staphylococcus aureus
b. Streptococcus pyogenes
c. Streptococcus pneumoniae
d. Haemophilus influenzae
Solution. (c) Streptococcus pneumoniae
Ref.:Read the text below
Sol :

The most common organisms which cause lobar pneumonia are Streptococcus pneumoniae, also called pneumococcus,
Haemophilus influenzaeand Moraxella catarrhalis. Mycobacterium tuberculosis, the tubercle bacillus, may also cause lobar pneumonia if
pulmonary tuberculosisis not treated promptly.

The identification of the infectious organism (or other cause) is an important part of modern treatment of pneumonia.
The anatomical patterns of distribution can be associated with certain organisms, and can help in selection of an antibiotic
while waiting for the pathogen to be cultured.
Your Answer. c
Correct Answer. c

Copyright 2014 Delhi Academy of Medical Sciences, All Rights Reserved.

39/119

(94).

All of following features are seen in the viral pneumonia except :


a. Presence of interstitial inflammation
b. Predominance of alveolar exudates
c. Bronchiolitis
d. Multinucleate gaint cells in the bronchiolar wall
Solution. (b) Predominance of alveolar exudates
Ref.:Read the text below
Sol :
Atypical pneumonia
Acute viral pneumonias or interstial pneumonias are sometimes called atypical pneumonias because the typical features of air space
pneumonia are not present in them.
The term atypical denotes
The moderate amount of sputum.
Only moderate elevation of white cell count.
No physical findings of consolidation
Lack of alveolar exudates.
Acute interstitial pneumonias result from infection by agents that are predominantly obligate intracellular pathogens.
Infections with these organisms evokes an acute inflammation that is usually restricted to the interstitium without
involvement of the alveolar spaces.
Your Answer. b
Correct Answer. b

(95).

Serotonin (5-HT) is one of several mediators of the signs and symptoms of carcinoid tumor.When released from this neoplasm, most
of the manifestations of 5-HT are the result of activation of the 5-HT2 receptor. These manifestations are therefore likely to
include which of the following effects?
a. Bronchospasm
b. Constipation
c. Skeletal muscle weakness
d. Tachycardia
Solution. (a) Bronchospasm
Ref:Read the text below
Sol:

5-HT2 receptors mediate smooth muscle contraction and thus cause bronchospasm and diarrhea, not constipation.

Skeletal muscle and cardiac muscle are relatively insensitive to 5-HT.

Peptic ulceration is not mediated in part by 5-HT; the autacoid actually reduces acid secretion.
Your Answer. a
Correct Answer. a

Copyright 2014 Delhi Academy of Medical Sciences, All Rights Reserved.

40/119

(96).

Which of the following drugs is useful in asthma because it stabilizes mast cells?
a. Albuterol
b. Cromolyn
c. Ipratropium
d. Salmeterol
Solution. (b) Cromolyn
Ref:Read the text below
Sol:

Cromolyn is a prophylacticagent for asthma that appears to stabilize mast cells.

Albuterol is a shorter acting beta-2-selective agonist that is used for acute, not prophylactic, therapy.

Ipratropiumis an antimuscarinic drug.

Salmeterolhas a long duration beta-2-agonist action that appears to have an anti-inflammatory action and is useful in prophylaxis of
asthmatic attacks.

It has a slow onset of action that makes it useless in treating acute attacks.
Your Answer. b
Correct Answer. b

(97).

Which of the following mechanisms most accurately describes the blocking action of methimazole?
a. Iodine uptake by the thyroid
b. Organification of iodine in the thyroid
c. Peripheral conversion of T4 to T3
d. Peripheral thyroxine (T4) and T3 receptors
Solution. (b) Organification of iodine in the thyroid
Ref:Read the text below
Sol:

Methimazole (and other thioamides) act primarilyby inhibiting thyroid peroxidase, the critical enzyme in iodine organification.

They also inhibit coupling of iodotyrosines in thyroglobulin.

They do not block iodine uptake by the gland. Block of iodine uptake by the thyroid is a property of antithyroid anions such as
thiocyanate (SCN) and perchlorate (ClO4 ).

Inhibition of peripheral conversion of T4 to T3 is the mechanism of beta blockers and iodine-containing radio-opaque
drugs such as ipodate.

Block of peripheral thyroid receptors is not a mechanism of any currently available drug.
Your Answer. b
Correct Answer. b

(98).

Which of the following antitubercular agents is a strong inducer of hepatic cytochrome P450?
a. Ethambutol
b. Isoniazid
c. Rifampin
d. Streptomycin
Solution. (c) Rifampin
Ref:Read the text below
Sol:

Rifampin selectivelyinhibits bacterial DNA-dependent RNA polymerase. It is very useful in treating mycobacterial infections
since it can penetrate cells and kill intracellular organisms.

It is one of the most potent inducers of cytochrome P450 known, leading to increased hepatic clearance of many other drugs
including the oral anticoagulants, cyclosporine, propranolol, digitoxin, corticosteroids, and oral contraceptives.

Ethambutol is often combined with isoniazid in antitubercular regimens. Clearance is primarily via renal excretion.
Isoniazid is the mostwidely used antitubercular agent. It functions by inhibiting mycolic acid biosynthesis.
Your Answer. c
Correct Answer. c

Copyright 2014 Delhi Academy of Medical Sciences, All Rights Reserved.

41/119

(99).

A 35-year-old woman with hypertension is planning to become pregnant. Which of the following is contraindicated in pregnancy?
a. Clonidine
b. Hydralazine
c. Hydrochlorothiazide
d. Losartan
Solution. (d) Losartan
Ref:Read the text below
Sol:

Losartan causes renal damagein the fetus, and renal impairment in renovascular disease. It is contraindicated in pregnancy.

Clonidine causessome sedation and rebound hypertension when stopped suddenly, but is not contraindicated in pregnancy.

Hydralazine causes a reversibletype of lupus erythematosus.


Hydrochlorothiazide may cause hypokalemia, dilutional hyponatremia, elevated lipids, hyperuricemia, and glucose
intolerance.
Your Answer. d
Correct Answer. d

(100).

A35-year-old opera singer has been treated for hypertension with enalapril. Although his blood pressure has been reduced, he
complains that he is now unable to perform because of a dry cough. Which of the following acts by a similar but not identical
mechanism and is much less likely to cause cough?
a. Captopril
b. Clonidine
c. Losartan
d. Prazosin
Solution. (c) Losartan
Ref:Read the text below
Sol:

Losartan, an AT1 receptor-blocking agent, has effects in hypertension similar to those of ACE inhibitors but causes a much lower
incidence of cough.

Captopril and other ACE inhibitorscause a dry cough in 520% of patients. This may be due to accumulation of bradykinin as a
result of ACE inhibition; ACE also metabolizes bradykinin. The cough disappears with cessation of ACE inhibitor treatment.

Pretreatment with aspirin reduces its frequency and severity in some patients.

Clonidine the centrally acting alpha- 2-receptor agonist, produces sedation and xerostomia but not cough.

The alpha-1-receptor antagonist prazosin produces postural hypotension but not cough.

The betablocker propranolol may produce a variety of side effects including precipitating heart failure and asthma in susceptible
patients.

It does not cause cough. Propranolol has been used by musicians to control palpitations associated with stage fright.
Your Answer. c
Correct Answer. c

Copyright 2014 Delhi Academy of Medical Sciences, All Rights Reserved.

42/119

(101).

Which of the following is the muscle relaxant of choice in renal failure ?


a. Rapacurium
b. Pancuronium
c. Atracurium
d. Rocuronium
Solution. (c) Atracurium
Ref:Read the text below
Sol:
Atracurium

The unique feature of atracurium is inactivation in plasma by spontaneous non enzymatic degradation (Hofmann elimination) in
addition to that by cholinesterases.

Consequently its duration of action is not altered in patients with hepatic / renal insufficiency or hypodynamic circulation.

Hemodynamically it is almost neutral, though histamine release may occur.

Therefore it is the agent of choice in renal failure.


Your Answer. a
Correct Answer. c

(102).

Mebendazole may be used for treatment of all of the following conditions except:
a. Enterobiasis
b. Ascariasis
c. Hook worm disease
d. Hook worm disease
Solution. d) Strongyloidiasis
Ref:Read the text below
Sol:

Characteristics and therapy for commonly encountered nematode infections.


Your Answer. c
Correct Answer. d

(103).

The blood culture from a patient of febrile neutropenia has grown pseudomonas aeruginosa,it was found to be a producer of
extended spectrum beta lactamase enzyme. The best choice of antimicrobial therapy should be:
a. Ceftazidime + Amikacin
b. Aztreonam + Amikacin
c. Cefpirome + Amikacin
d. Imipenem + Amikacin
Solution. d) Imipenem + Amikacin
Ref:Read the text below
Sol:
CARBAPENEMS
Carbapenems are synthetic that differ from the penicillins in that the sulfur atom of the thiazolidine ring has been externalized and
replaced by a carbon atom.
Imipenem is the only drug of this group currently available.
Antibacterial spectrum:
1. Imipenem/cilastatin is the broadest spectrum -lactam antibiotics preparation currently available.
3. The drug plays a role in empiric therapy since it is active against penicillinase-producing grampositive and gram-negative
organisms, anaerobes, an pseudomonas aeruginosa,although other pseudomonas strains are resistant.
Your Answer. a
Correct Answer. d

Copyright 2014 Delhi Academy of Medical Sciences, All Rights Reserved.

43/119

(104).

A patient with cancer received extreme degree of radiation toxicity. Further history revealed that the dose adjustment of a particular
drug was missed during the course of radiotherapy. Which of the following drugs required a dose adjustment in that patient during
radiotherapy in order to prevent radiation toxicity:
a. Vincristine
b. Dactinomycin
c. Cyclophosphamide
d. 6-Mercaptopurine
Solution. (b) Dactinomycin
Ref:Read the text below
Sol:

Your Answer. c
Correct Answer. b

(105).

Gentamycin, obtained from Micromonospora purpura, has become the most commonly usedaminoglycoside for acute infections; but
is ineffective against: a. Klebsiella
b. Strep.pyogenes
c. Proteus
d. Pseudomonas
Solution. (b) Strep.pyogenes
Ref:Read the text below
Sol:

Gentamicin has a broader spectrum of action: effective against Ps. aeruginosa and most strains of Proteus, E. coli, Klebsiella,
Enterobacter, Serratia.

It is ineffective against M.tuberculosis, Strep. pyogenes and Strep. pneumoniae, but inhibits many Strep.faecalis and some
Staph.aureus.
Your Answer. d
Correct Answer. b

(106).

The increased risk of blood clots in women who use certain combined estrogen/progestin birth control pills is well established.
Which of the following best explains this association?
a. Impaired glucose tolerance
b. Increased Factor II, VII, IX, and X levels produced by estrogens
c. Increased plasminogen levels produced by progestins
d. Increased thyroxine-binding globulin
Solution. (b) Increased Factor II, VII, IX, and X levels produced by estrogens
Ref:Read the text below
Sol:

Increased clotting factor synthesis (and decreased antithrombin III synthesis) caused by estrogens are the major factors
responsible for increased incidence of thromboembolic phenomena. Glucose tolerance is impaired in some women by progestins and
would not be expected to affect clotting.

Plasminogen is increased by estrogens, not progestins, but would decrease clotting.

Thyroxine-binding globulin is also increased by estrogen, but would have little effect on clotting.

Triglycerides are increased by estrogens and would have little or no effect on clotting.
Your Answer. b
Correct Answer. b

Copyright 2014 Delhi Academy of Medical Sciences, All Rights Reserved.

44/119

(107).

Which of the following drugs is considered afirst-line agent in the chronic treatment of rheumatoid arthritis and in the
treatment of choriocarcinoma?
a. Cyclosporine
b. Hydroxychloroquine
c. Methotrexate
d. Sulfasalazine
Solution. (c) Methotrexate
Ref:Read the text below
Sol:

Methotrexate in very low dosagehas proven effective in slowing the progression of rheumatoid arthritis and has low toxicity.

It is also one of the most effective (in much higher dosage) antimetabolite cancer chemotherapeutic drugs used in
choriocarcinoma.

Cyclosporine is not used in either condition but is a first-line drug in transplant immunopharmacology.

Hydroxychloroquine is a disease-modifying antirheumatoid arthritis drug, but has no efficacy in the treatment of cancer. It is more
toxic and less effective in rheumatoid arthritis than methotrexate.

Sulfasalazine is also effective in slowing the progression of joint damage in rheumatoid arthritis but has no efficacy in
cancer.
Your Answer. c
Correct Answer. c

(108).

Which of the following laxatives acts osmotically to increase the water content of the stool?
a. Cascara
b. Docusate
c. Glycerine
d. Magnesium hydroxide
Solution. (d) Magnesium hydroxide
Ref:Read the text below
Sol:

Magnesium hydroxideis an osmotic laxative that retains water in the lumen of the colon.

Cascara is a naturally occurring plant product that stimulates the bowel and causes evacuation within 612 hours when given orally.

Docusate is a stool-softeningagentthat acts entirely within the lumen.

Glycerine given by suppository, is a stool softener.


Your Answer. d
Correct Answer. d

(109).

Which of the following drugs binds to, and prevents disassembly of, microtubules?
a. Busulfan
b. Cisplatin
c. Mercaptopurine
d. Paclitaxel
Solution. (d) Paclitaxel
Ref:Read the text below
Sol:

Paclitaxel is a plant alkaloidthat interferes with microtubule disassembly.

Busulfan and cisplatin are alkylatingagents that interfere with DNAmobilization and replication.

Mercaptopurine is an antimetabolitethat interferes with purine synthesis.

Vinblastine and vincristineand vinorelbine are mitotic poisons that prevent cancer cell division by interfering with microtubule
assembly and thus immobilizing the mitotic spindle apparatus.

Their indications and toxicities are quite different from those of paclitaxel.
Your Answer. d
Correct Answer. d

Copyright 2014 Delhi Academy of Medical Sciences, All Rights Reserved.

45/119

(110).

Which of the following drugs blocks the action of angiotensin II at the AT1 receptor?
a. Bradykinin
b. Enalapril
c. Losartan
d. Ondansetron
Solution. (c) Losartan
Ref:Read the text below
Sol:

Losartan and severalother drugs block the action of angiotensin II at its primary cardiovascular target, the AT1 receptor.

They are therefore useful in hypertension and heart failure. Bradykinin (choice A) is a vasodilator peptide.

It is not therapeutically useful because of undesirable side effects.

Enalapril is an ACE inhibitorand reduces the production of angiotensin II from angiotensin I.

Ondansetron is a 5-HT3 receptorantagonist used to prevent chemotherapy-induced and postoperative vomiting.


Your Answer. c
Correct Answer. c

(111).

Which of the following statements describes the major difference between zolpidem and older hypnotics such as barbiturates
and benzodiazepines?
a. Associated with thrombocytopenia
b. Does not act on chloride channels in the CNS
c. Has significant anticonvulsant activity
d. Reduced addiction liability
Solution. (d) Reduced addiction liability
Ref:Read the text below
Sol:

Zolpidem has greatly reduced addiction liability compared to barbiturates and benzodiazepines even though it does act on
chloride channels.

Zolpidem does not havemajor anticonvulsant actions, a longer duration of action, or thrombocytopenic toxicity.
Your Answer. d
Correct Answer. d

(112).

Which of the following drugs is used in cerebral palsy to reduce skeletal muscle spasticity by an action on cholinergic nerve endings?
a. Baclofen
b. Botulinum toxin
c. Dantrolene
d. Diazepam
Solution. (b) Botulinum toxin
Ref:Read the text below
Sol:

Botulinum toxin actson cholinergic nerve endings (including skeletal muscle motor nerve endings) to reduce acetylcholine release.

Baclofen acts in the CNS to activate GABAB receptors; some sedation may result.

Dantroleneacts within the skeletal muscle cell to reduce calcium release from the sarcoplasmic reticulum; reduction of muscle
spasm results without central nervous system depression.

Diazepam facilitates GABAergic transmission in the brain and spinal cord; it does not act on cholinergic transmission.
Your Answer. a
Correct Answer. b

Copyright 2014 Delhi Academy of Medical Sciences, All Rights Reserved.

46/119

(113).

Which of the following drugs is most likely to cause parkinsonian adverse effects?
a. Clozapine
b. Fluphenazine
c. Haloperidol
d. Olanzapine
Solution. (c) Haloperidol
Ref:Read the text below
Sol:

Haloperidol is a verypotent and efficacious antipsychotic but also manifests the highest incidence of extrapyramidal effects.

Clozapine, olanzapine, and quetiapineare newer, atypical antipsychotic agents with much lower incidence of extrapyramidal
toxicity.
Your Answer. c
Correct Answer. c

(114).

Apatient has been treated for bipolar disorder for several months. Which of the following is a common adverse effect of lithium
carbonate?
a. Dry mouth
b. Hyperthyroidism
c. Leukopenia
d. Nephrogenic diabetes insipidus
Solution. (d) Nephrogenic diabetes insipidus
Ref:Read the text below
Sol:

Nephrogenic diabetes insipidus is a common adverse effect of lithium at therapeutic dosage.

Dry mouth and other anticholinergic toxicities are common with tricyclic antidepressants and some phenothiazines.

Lithium may induce hypothyroidism, but not hyperthyroidism

Similarly, lithium increases rather than decreases white blood cell count
Your Answer. a
Correct Answer. d

(115).

If a person is not heard for.years by his relatives or friends, he is presumed to be dead


a. 3
b. 7
c. 10
d. 12
Solution. (b) 7
Ref:Read the text below
Sol:
Currently, the law generally assumes a person is dead if, after seven years:

There has been no evidence that they still live.

The people most likely to have heard from them have had no contact.

Inquiries made of that person have had no success


Your Answer. b
Correct Answer. b

Copyright 2014 Delhi Academy of Medical Sciences, All Rights Reserved.

47/119

(116).

Which of the following constitutes Grievous injury


a. Incised wound over the scalp
b. Lacerated wound over the scalp
c. Fracture of radius
d. Hospitalization for 15 days
Solution. (c) Fracture of radius
Ref:Read the text below
Sol:
Grievous hurt.-- The following kinds of hurt only are designated as" grievous":
First.- Emasculation.

Secondly.- Permanent privation of the sight of either eye.

Thirdly.- Permanent privation of the hearing of either ear.

Fourthly.- Privation of any member or joint.

Fifthly.- Destruction or permanent impairing of the powers of any member or joint.

Sixthly.- Permanent disfiguration of the head or face.

Seventhly.- Fracture or dislocation of a bone or tooth.

Eighthly.- Any hurt which endangers life or which causes the sufferer to be during the space of twenty days in severe bodily pain, or
unable to follow his ordinary pursuits.
Your Answer. c
Correct Answer. c

(117).

How many points of comparison should be present for fingerprints to match:


a. 8
b. 6
c. 12
d. Varies according to the country
Solution. (d) Varies according to the country
Ref:Read the text below
Sol:

Holland 12 points

Germany 8 points

USA no basis for deciding the minimum no. of points

UK 16 points

India 8 points
Your Answer. b
Correct Answer. d

(118).

Commonest fractureof skull is:


a. Communited fracture
b. Signature fracture
c. Ponds fracture
d. Ring fracture
Solution. (b) Signature fracture
Ref: Read the text below
Sol:

Commonest skull fracture is linear fracture/ Fissure fracture. However, this is not given in our options.

Depressed fracture/ signature fracture is the second most common fracture of skull.
Your Answer. a
Correct Answer. b

Copyright 2014 Delhi Academy of Medical Sciences, All Rights Reserved.

48/119

(119).

Fish tailing of margins in stab wound is seen with:


a. Single edged knife
b. Double edged knife
c. Bayonet
d. None
Solution. (a) Single edged knife
Ref:Read the text below
Sol:

Stab by a single edged weapon causes fish tailing wound

Stab by a double edged weapon causes spindle shaped wound.


A Bayonet is a sharp weapon (usually double edged) placed in front of gun for close combat.
Your Answer. b
Correct Answer. a

(120).

Gun powder residueson forearms can be detected by:


a.
b.
c.
d.
a. Benzidine test
b. Barberios test
c. Dermal Nitrate test
d. Hydrostatic test
Solution. (c) Dermal Nitrate test
Ref: Read the text below
Sol:

Benzidine test screening test for blood

Barberios test chemical test for detecting seminal stain

Dermal Nitrate test test for detecting gunpowder residues


Hydrostatic test test for detecting whether a child was live born or not.
Your Answer. b
Correct Answer. c

(121).

Gun powder residueson forearms can be detected by:


a. Benzidine test
b. Barberios test
c. Dermal Nitrate test
d. Hydrostatic test
Solution. (c) Dermal Nitrate test
Ref: Read the text below
Sol:

Benzidine test screening test for blood

Barberios test chemical test for detecting seminal stain

Dermal Nitrate test test for detecting gunpowder residues


Hydrostatic test test for detecting whether a child was live born or not.
Your Answer. b
Correct Answer. c

Copyright 2014 Delhi Academy of Medical Sciences, All Rights Reserved.

49/119

(122).

What causes post mortem luminescence?


a. Dhatura
b. Mercury
c. Armillaria
d. Oleander
Solution. (c) Armillaria
Ref: Read the text below
Sol:
Causes for post mortem luminescence
1. Armillaria millea/ Ramsbottom
2. Photobacterium fischerii
Your Answer. d
Correct Answer. c

(123).

Phossy jaw is seen in which poisoning ?


a. Mercury
b. Yellow phosphorous
c. Red phosphorus
d. Tetanus
Solution. (c) Red phosphorus
Ref:Read the text below
Sol:

Phossy jawis seen in chronic poisoning with the fumes of Red phosphorus, seen in the industries where its use is rampant like
matchbox industry and incense making..
Your Answer. c
Correct Answer. c

(124).

Which sexual perversion is punishable


a. masochism
b. Voyeurism
c. Tranvestism
d. Fetishism
Solution. (b) Voyeurism
Ref:Read the text below
Sol:

In 2013, Indian Parliament made amendments to the Indian Penal Code, introducing voyeurism as a criminal offence.

A man committing the offence of voyeurism would be liable for imprisonment not less than one year and which may extend up to
three years for the first offence, and shall also be liable to fine and for any subsequent conviction would be liable for imprisonment for
not less than three year and which may extend up to seven years and with fine.

Amended section 354 C IPC. (3 to 7 years of imprisonment)


Your Answer. b
Correct Answer. b

Copyright 2014 Delhi Academy of Medical Sciences, All Rights Reserved.

50/119

(125).

Dowry deathis covered under which section of IPC


a. 304- A
b. 304- B
c. 176
d. 302
Solution. (b) 304- B
Ref:Read the text below
Sol:
This Section of the Indian Penal Code was inserted by a 1986 amendment. The wording of the law states:
Section 304B. Dowry death
(1) Where the death of a woman is caused by any burns or bodily injury or occurs otherwise than under normal circumstances within
seven years of her marriage and it is shown that soon before her death she was subjected to cruelty or harassment by her husband or any
relative of her husband for, or in connection with, any demand for dowry, such death shall be called "dowry death" and such husband or
relative shall be deemed to have caused her death.
Explanation:-For the purpose of this sub-section, "dowry" shall have the same meaning as in section 2 of the Dowry Prohibition Act, 1961
( 28 of 1961).
(2) Whoever commits dowry death shall be punished with imprisonment for a term which shall not be less than seven years but which
may extend to imprisonment for life
Your Answer. b
Correct Answer. b

(126).

The WHO has set the SARS containment period at:


a. 10 days
b. 20 days
c. 28 days
d. 30 days
Solution. (b) 20 days
Ref Read the text below
Sol:

The mortality rate of SARS is higher than that of non-H5N1 influenza strains or other common respiratory tract infections.

The overall mortality rate of SARS has been approximately 10%. According to the CDC and the WHO, the death rate among
individuals older than 65 years exceeds 50%.

The WHO has set the SARS containment period at 20 days. If no new cases of SARS are reported in a given area over a 20-day
period, given the relatively short incubation period of the disease, the WHO considers SARS infections in that area to be
contained.
Your Answer. b
Correct Answer. b

(127).

The timeelapsed between exposure to a pathogenicorganism, a chemicalor radiation, and when symptomsand signs are first apparent is
a. Secondary attack rate
b. Incubation period
c. Latency
d. Serial interval
Solution. (b) Incubation period
Ref Read the text below
Sol:

Incubation periodis the timeelapsed between exposure to a pathogenicorganism, a chemicalor radiation, and when symptomsand
signs are first apparent. The period may be as short as minutes to as long as thirty years in the case of variant Creutzfeldt-Jakob disease.
While Latent or Latency period may be synonymous, a distinction is sometimes made between Incubation period, the period between
infection and clinical onset of the disease, and Latent period, the time from infection to infectiousness. Which is shorter depends on the
disease
Your Answer. b
Correct Answer. b

Copyright 2014 Delhi Academy of Medical Sciences, All Rights Reserved.

51/119

(128).

For the community diagnosis of trachoma, WHO recommends that follicular and intense trachoma inflammation should be assessed in:
a. Women aged 15-45 years
b. Population of 10-28 years
c. Children aged 0-10 years
d. Population above 25 years irrespective of sex
Solution. (c) Children aged 0-10 years
Ref: Read the text below
Sol:

According to WHO, prevalence of more than 5% severe and moderate trachoma in children under 10 years is an indication of
mass or blanket treatment consisting of twice daily application of ointmenttetracycline 1% or ointment erythromycin to all
children for 5 consecutive days each month or oncedaily for 10 days per month for 6 months or for 60 consecutive days.
Your Answer. d
Correct Answer. c

(129).

Which of the following is an example of Disability limitation in poliomyelitis


a. Reducing occurance of polio by immunization
b. Arranging of schooling of child suffering from PRPP
c. Resting affecting limbs in neutral position
d. Providing calipers for walking
Solution. (c) Resting affecting limbs in neutral position
Ref: Read the text below
Sol:

CONCEPT OF PREVENTION
No risk factors :
Primordial prevention(prevention OF EMERGENCE of risk factors)
Appearance of risk factors: Primary prevention Health promotion & Specific protection
Appearance of disease : Secondary prevention Early diagnosis & Disease treatment
Late pathogenesis phase: Tertiary prevention Disability limitation & Rehabilitation
Your Answer. d
Correct Answer. c

(130).

In a certain population, there were 4050 births in the last one year. There were 50 still births. 50 infants died within 7 days where as 150
died within 28 days. What is the NMR
a.
b.
c.
d.
a. 50
b. 49.6
c. 37.03
d. 69%
Solution. (c) 37.03
Ref: Read the text below
Sol:

Neonatal deathsare the deaths occurring during the neonatal period, commencing at birth and ending 28 completed days after
birth. Neonatal mortality rate is the number of neonatal deaths in a given year per 1000 live births in that year

This can be tabulated as:

NMR=Number of deaths of children under 28 days of age in a year1000/ Total live births in the same year.

Here, NMR=1501000/4050= 37.03


Your Answer. b
Correct Answer. c

Copyright 2014 Delhi Academy of Medical Sciences, All Rights Reserved.

52/119

(131).

Mortality experiences are taken into consideration when defining :a. General fertility rate
b. .Total fertility rate
c. Gross reproduction rate
d. Net reproduction rate (NRR)
Solution. (d) Net reproduction rate (NRR)
Ref: Read the text below
Sol:

NRR is defined as, the number of daughters a newborn girl will bear during her life time assuming fixed age specific fertility
and mortality rates.

It is the most significant indicator of fertility. NRR = 1 is alsoknown as replacement rate and coincides approximately to the two
child norm.
In a declared policy, Indiaaims to achieve NRR = 1 by 2006. In order to achieve NRR = 1, 60% of the eligible couples should
beeffectively practicing family planning (couple protection rate)
Your Answer. c
Correct Answer. d

(132).

Isolation period for mumps is


a. Until fever subsides
b. Until pain subsides
c. Until swelling subsides
d. Until headache subsides
Solution. (c) Until swelling subsides
Ref Read the text below
Sol:
Mode of transmission of mumps

Transmissionoccurs through via respiratory aerosols and respiratory droplet spread or by direct contact with contaminated saliva.

Mumps is communicable from six to seven days before to nine days after the onset of parotitis.

Asymptomatic and inapparent cases can also be infectious.


Your Answer. a
Correct Answer. c

(133).

Considering transmission probabilities(tp), p01 is:


a. Transmission probabilities from unvaccinated infective to unvaccinated susceptible
b. Transmission probabilities from vaccinated infective to unvaccinated susceptible
c. Transmission probabilities from unvaccinated infective to vaccinated susceptible
d. Transmission probabilities from vaccinated infective to vaccinated susceptible
Solution. (b) Transmission probabilities from vaccinated infective to unvaccinated susceptible
Ref Read the text below
Sol:
TRANSMISSION PROBABILITIES(tp):

p00: tp from unvaccinated infective to unvaccinated susceptible

p01: tp from vaccinated infective to unvaccinated susceptible

p10: tp from unvaccinated infective to vaccinated susceptible

p11: tp from vaccinated infective to vaccinated susceptible


Your Answer. a
Correct Answer. b

Copyright 2014 Delhi Academy of Medical Sciences, All Rights Reserved.

53/119

(134).

Infection that is transmissible under natural conditions from vertebrate animals to man is:
a. Zoonosis
b. Epizootic
c. Enzotic
d. Both b and c
Solution. (a) Zoonosis
Ref Read the text below
Sol:

Zoonosisis an infection that is transmissible under natural conditions from vertebrate animals to man, e.g. rabies, plague, bovine
tuberculosis..

An epizotic is an outbreak (epidemic) of disease in an animal population, e.g. rift valley fever.

An Enzotic is an endemic occurring in animals, e.g. bovine TB.


Your Answer. a
Correct Answer. a

(135).

The disease is constantly present at high incidence and/or prevalence rate and affects all age groups equally is:
a. Hyperendemic
b. Holoendemic
c. Exotic diseases
d. Epidemic
Solution. (a) Hyperendemic
Ref Read the text below
Sol:

The term hyperendemic expresses that the disease is constantly present at high incidence and/or prevalence rate and affects all
age groups equally.

The term holoendemic expresses a high level of infection beginning early in life and affecting most of the child population,
leading to a state of equilibrium such that the adult population shows evidence of the disease much less commonly than do the children
(e.g. malaria)

Exotic diseasesare those which are imported into a country in which they do not otherwise occur, as for example, rabies in the UK.

An epidemic usually affecting a large proportion of the population, occuring over a wide geographic area such as a section of a
nation, the entire nation, a continent or the world, e.g. Influenza pandemics.
Your Answer. a
Correct Answer. a

(136).

Select incorrect statement regarding AFB isolation:


a. Indicated for patients with pulmonary tuberculosis who have a positive sputum smear.
b. Specifications include use of a private room with special ventilation and closed door.
c. Gowns are used to prevent gross contamination of clothing.
d. Gloves are indicated
Solution. (d) Gloves are indicated
Ref Read the text below
Sol:
TUBERCULOSIS ISOLATION(AFB ISOLATION)

For patients with pulmonary tuberculosis who have a positive sputum smear or a chest x-ray that strongly suggests active
tuberculosis.

Specifications include use of a private room with special ventilation and closed door. In addition to the basic
requirements, respirator-type masks are used by those entering the room.

Gowns areused to prevent gross contamination of clothing.

Gloves are not indicated.


Your Answer. c
Correct Answer. d

Copyright 2014 Delhi Academy of Medical Sciences, All Rights Reserved.

54/119

(137).

Denominator of positive predictive valueis


a. True positive + False positive
b. True positive + False negative
c. False positive + True negative
d. False positive + False negative
Solution. (a) True positive + False positive
Ref Read the text below
Sol:
Sensitivity and specificity are terms used to describe the value of tests. This calculator will determine these values based on
true positives, false positives, true negatives, and false negatives.
Disease Present

No Disease

Positive test

Negative test

Sensitivity= a/(a+c)
Specificity = d/(b+d)
Positive Predictive Value = a/(a+b)
Negative Predictive Value = d/(c+d)
Your Answer. a
Correct Answer. a

(138).

In the context of epidemiology, the following are important criteria for making casual inferences except.
a. Strength of association
b. Consistency of association
c. Coherence of association
d. Predictive value
Solution. (d) Predictive value
Ref Read the text below
Sol:
PREDICTIVE VALUE:
The predictive value of a test is a measure (%) of the times that the value (positive or negative) is the true value, i.e. the percent of all
positive tests that are true positives is the Positive Predictive Value.
__TP___ X 100 = Predictive Value of a Positive Result (%)
TP + FP
__TN___ X 100 = Predictive Value Negative Result (%)
Your Answer. b
Correct Answer. d

Copyright 2014 Delhi Academy of Medical Sciences, All Rights Reserved.

55/119

(139).

All of the following statements about plague are wrong, except


a. Domestic rat is the main reservoir
b. Bubonic is the most common variety
c. The causative bacillus can survive up to 10 years in the soil of rodent burrows
d. The incubation period for pneumonic plague is one to two weeks
Solution. (b) Bubonic is the most common variety
Ref: Read the text below
Sol:
Plague:
Vector: -X-chenopis
Both sexesX.index >1 is significant; should drop to 0

Partially blocked flea more effective than Blocked flea in


transmitting disease
Most common: - bubonic
Transmissionbite of infected rat flea
Chemoprophylaxis Tetracycline
Treatment streptomycin x 7-10 days.
Vaccination formalin killed ; 2 doses s/c immunity in 5-7 days, lasts 6 months. Ineffective during epidemics.
Area free of plague 3 month last sign in wild rodent
1 month last sign in domestic rodent
Twice the IP
Your Answer. d
Correct Answer. b

(140).

Which if the following conditions does not have a national screening/ control program
a. Dibetes Mellitus
b. Dental caries
c. Refractive errors
d. Carcinoma cervix
Solution. (b) Dental caries
Ref: Read the text below
Sol:
There is no national program in India for dental caries or oral health
The program for other diseases are:
i.
National program for control of blindness (NPCB)
ii.
National program for prevention and control of cancer, diabetes, cardiovascular disease and stroke (NPCDCS)
Your Answer. a
Correct Answer. b

Copyright 2014 Delhi Academy of Medical Sciences, All Rights Reserved.

56/119

(141).

A village is divided into 5 relevant subgroups for the purpose of survey. Individuals from each subgroups are then selected
randomly. This type of sampling is:
a. Simple random
b. Stratified
c. Cluster
d. Systematic random
Solution. (b) Stratified
Ref: Read the text below
Sol:

The village is divided into strata and then individuals are selected from each strata randomly.

Thus the sampling is stratified random sampling


Your Answer. b
Correct Answer. b

(142).

An investigator observes that 5 independent risk factors influence the occurrence of disease. Which of the following test should be
applied to assess the relationship between these 5 risk factors and the occurrence of disease
a. ANOVA
b. Multiple linear regression
c. Multiple logistic regression
d. One way analysis of variance
Solution. (c) Multiple logistic regression
Ref: Read the text below
Sol:

5 independent risk factorsfor the disease have already been observed.

The statistical test to assess the relationship between multiple independent risk factors and outcome is multiple logistic
regression analysis
Your Answer. b
Correct Answer. c

Copyright 2014 Delhi Academy of Medical Sciences, All Rights Reserved.

57/119

(143).

The systematic distortion of retrospective studies that can be eliminated by a prospective design is
a. Confounding
b. Effect modification
c. Recall bias
d. Measurement bias
Solution. (c) Recall bias
Ref: Read the text below
Sol:
Recall bias is a type of type of information bias that occurs when there are differences in how exposure groups or disease
groups remember certain information.
Other Biases are:
Several types of bias can occur.
1. Selection bias occurs when the case and controls selected for the study are not representative of the general population, hence the
results cant be generalized (loss of external validity).
2. Measurement bias Occurs while data collection and occurs when the subjects cant recall the events properly or the investigator is
biased in collecting data because of prior knowledge of the association between the risk factor and the disease, e.g. if the investigator is
studying the relationship between smoking and lung cancer he might take a more detailed questioning of the cases and not the controls
and thus exaggerate the relationship.
3. Confounding bias Occurs when the disease under study is associated with another factor apart from the risk factor under study. In
such a scenario it might be difficult to interpret the results, e.g. if we are studying the association between smoking and CHD; it is
possible that the cases also take alcohol, thus confounding the results. This bias can be minimized by matching.
Statistical procedures like regression analysis also help in detecting potential confounders.
4. Berkesonian Bias Differing rates of admissions to hospitals cause bias.
5. Surveillance bias: If the population is measured ove r a period of time, the dis ascertainment may be better in monitored population,
leading to erroneous estimate of OR/RR.
Your Answer. c
Correct Answer. c

(144).

Measles vaccineat a primary health centre should be stored at which temperature?


a. -200C
b. 00C
c. +20C to 80C
d. Any temperature not exceeding room temperature
Solution. (c) +20C to 80C
Ref Read the text below
Sol:
Range

Vaccines

Most sensitive
Least sensitive

BCG(after reconstitution)
OPV
Measles
Hepatitis B
DPT
DT
BCG(before reconstitution)
TT(Tetanus toxoid)

Vaccine damaged by freezing

Vaccines that can be frozen without harm

DPT
DT
TT
Hepatitis B

BCG(before reconstitution)
OPV
Measles(before reconstitution)

Storage Temperature:

All vaccines are to be retained at temperatures between +2 to +8 C.

A break in the cold chain is indicated if temperature rises above +8 C or falls below +2C in case of ILR and other
refrigerators.
Your Answer. b
Correct Answer. c

Copyright 2014 Delhi Academy of Medical Sciences, All Rights Reserved.

58/119

(145).

In STD cases, a patient is asked to name other persons is his socio sexual group, then they are investigated. This is a example of
a. Screening
b. Contact tracing
c. Mass screening
d. High risk screening
Solution. (b) Contact tracing
Ref Read the text below
Sol:

In epidemiology, contact tracing is the identification and diagnosis of persons who may have come into contact with an infected
person.

For sexually transmitted diseases, this is generally limited to sexual partners but for highly virulent diseases such
as Ebola and tuberculosis, a thorough contact tracing would require information regarding casual contacts.
Some AIDS activists have argued that contact tracing is counter-productivein that it would lead persons to avoid seeking medical
treatment for fear that it would breach their right to privacy.
Your Answer. b
Correct Answer. b

(146).

The lingual tonsil are found.


a. Tip of toungue
b. Near submandibular gland
c. Middle of the tongue
d. Pharyngeal tongue
Solution. (d) Pharyngeal tongue
Ref:Read the text below
Sol:

The lingual tonsils are rounded masses of lymphatic tissuethat cover the posterior region of the tongue.

They are located on the dorsal surface at the base of the tongue. Their lymphatic tissue are dense and nodular, their surface is
covered with stratified squamous epitheliumwhich invaginates as a single cryptinto each lingual tonsil.

They are partially surrounded by connective tissue placing them in the group of Partially Encapsulated Lymphatic Organs, tonsils,
the only one of its kind.

They have associated mucousglands which are drained by ducts directly into the single tonsillar crypt.
Your Answer. d
Correct Answer. d

(147).

Laryngocele arises as herniation of laryngealmucosa through the following membrane :


a. Thyrohyoid
b. Criothyroid
c. Cricotracheal
d. Crisosternal
Solution. (a) Thyrohyoid
Ref: Turner 10/e, p 168
Sol :
LARYNGOCELE
It is an air-filled cystic swelling due to dilatation of the saccule.
It may be
Internal which is confined within the larynx and presents as distension of false cord and aryepiglotic fold.
External in which distended saccule herniates through the thyrohoid membrane and presents as a reducible swelling in the neck,
which increases in size on coughing or performing valsalva.
Combined or mixed in which both internal and external components are seen.
Laryngocoel is supposed to arise from raised transglottic air pressure as in trumpet players, glass blowers or weight
lifters.
Your Answer. b
Correct Answer. a

Copyright 2014 Delhi Academy of Medical Sciences, All Rights Reserved.

59/119

(148).

Lateral sinus thrombosis is associated with all except


a. Gresisinger sign
b. Gradenigo sign
c. Lily-Crowe sign
d. Tobey Ayer test
Solution. (b) Gradenigo sign
Ref: Readthe text below
Sol:
Lateral sinus thrombophlebitis:
Symptoms:
Fever: Picket fence fever/ Hectic fever/due to septic emboli in blood
Progressive anaemia and emaciation Otalgia Neck pain with mastoid tenderness
Signs:
Pallor:due to the hemolytic nature of the infected organism
Griesingers sign: Pitting oedema over the occipital region (due to mastoid emissary vein thrombosis)
Tobey-Ayer test/ Quekenstedt test: Tests the change in the C.S.F following compression of either jugular veins
Crowe-Beck Test:Pressure on jugular vein of the healthy side produces engorgement of the retinal and supraorbital veins.
Your Answer. d
Correct Answer. b

(149).

Regarding laryngomalacia :
a. Most common cause of stridor in adult
b. Inspiratory stridor
c. Requires immediate surgery
d. Stridor worsens on laying in prone position
Solution. (B) Inspiratory stridor
Ref: Dhingra - 314
Sol :
Laryngomalacia : or congenital laryngeal stridor or congenital laryngeal stridor
Most common congenital anomaly of larynx.
Most common condition causing inspiratory stridor at or shortly after birth.
In most cases, it is asymptomtic.
M:F=1:1
There is abnormal flaccidity of laryngeal cartilage. Stridor occurs as a result of prolapsed of supraglottic structures into the
laryngeal inlet on inspiration.
Your Answer. b
Correct Answer. b

Copyright 2014 Delhi Academy of Medical Sciences, All Rights Reserved.

60/119

(150).

Gelles testis for


a. Otosclerosis
b. NIHL
c. Sensorineural deafness
d. None
Solution. (a) Otosclerosis
Ref: Readthe text below
Sol:
Gelle's test:
The air pressure in the EAM is altered by a Siegle's speculum leading to increased stiffness of the ossicular chain. In the
normal patient or in sensorineural deafness bone conducted sound appears to be decreased in intensity. Whereas in stapes fixation
no alteration occurs.
Bing test:
Increased loudness for bone conducted sound less than 2 kHz, occurs in the normal or sensorineural deafness when the EAM is
occluded without increasing the pressure ( As the masking effect of air conducting sound is removed). There is no change in
conductive deafness.
Your Answer. a
Correct Answer. a

(151).

Which of the following bones do not contribute the nasal septum :


a. Sphenoid
b. Lacrimal
c. Palatine
d. Ethmoid
Solution. (b) Lacrimal
Ref: Readthe text below
Sol:
The fleshy external end of the nasal septum is sometimes also called columella.
The nasal septum is composed of five structures:

perpendicular plate of ethmoid bone

vomer bone

cartilage of the septum

crest of the maxillary bone

crest of the palatine bone


Your Answer. b
Correct Answer. b

(152).

Bone conductionis decreased in


a. Ossicular necrosis
b. Ossicular disruption
c. Tympannic perforation
d. Cochlear disease
Solution. (d) Cochlear disease
Ref:Read the text below
Sol:

Sensorineural hearing loss(SNHL) is a type of hearing lossin which the root cause lies in the vestibulocochlear nerve(cranial
nerveVIII), the inner ear, or central processing centers of the brain. Sensorineural hearing loss can be mild, moderate, or severe,
including total deafness.

Bone conduction can be taken as a marker for Sensorineural hearing loss


Your Answer. d
Correct Answer. d

Copyright 2014 Delhi Academy of Medical Sciences, All Rights Reserved.

61/119

(153).

Suprahyoid laryngeal malignancydrain into


a. Pre tracheal
b. Upper jugular
c. Sub mental
d. Sub mandibular
Solution. (b) Upper jugular
Ref:Read the text below
Sol:
Supraglottis:
a. Sites: Suprahyoid epiglottis(suprahyoid and infrahyoid), aryepiglottic folds, arytenoids, false cords
b. Inferior border: horizontal plane passing through lateral margin of the ventricle
c. Lymphatics: Rich network, pass through thyrohyoid membrane into subdigastric, midjugular, and lower jugular nodes
Your Answer. b
Correct Answer. b

(154).

Rhinitis siccais most commonly seen


a. Anterior nares
b. Posterior nares
c. Supreme meatus
d. Superior meatus
Solution. (a) Anterior nares
Ref:Read the text below
Sol:

Ozena, which is also called rhinitissicca or atrophic rhinitis, is a rare disorder of the nasal passages. It occurs most often in arid
regions such as India, Egypt and the Middle East as well as in many other developing nations.
Rhinitis siccais most commonly seen on anterior nares.
Your Answer. a
Correct Answer. a

(155).

Most common site of laryngeal papillomatosis is


a. Anterior commissure
b. Posterior commissure
c. Subglottis
d. Glottis
Solution. (a) Anterior commissure
Ref:Read the text below
Sol:

Laryngeal papillomatosis, also known as recurrent respiratory papillomatosis or glottal papillomatosis, is a rare medical
condition(2 per 100,000 adults and 4.5 per 100,000 children), caused by a HPVinfection of the throat.

Laryngeal papillomatosis causes assorted tumorsor papillomasto develop over a period. Without treatment it is potentially fatal as
uncontrolled growths could obstruct the airway.

Laryngeal papillomatosis is caused by HPV types 6 and 11, in which benign tumors form on the larynx or other areas of the
respiratory tract

Most common site of laryngeal papillomatosis is Anterior commissure


Your Answer. a
Correct Answer. a

Copyright 2014 Delhi Academy of Medical Sciences, All Rights Reserved.

62/119

(156).

All of the following are diagnostic signs of cervical sympathetic trunk injury except :

a. Ptosis
b. Dilated pupil
c. Miosis
d. Anhidrosis
Solution. (b) Dilated pupil.
Ref Read the text below
Sol:

The superior tarsal muscle is innervated by fibers traveling through the cervical sympathetic trunk and the internal carotid
plexus.

If the trunk is interrupted, the muscle is paralyzed, causing drooping of the eyelid (ptosis). This is one of the signs of cervical
sympathetic trunk injury.

The affected pupil is smaller than the pupil of the opposite eye. It does not dilate when the pupil is shaded (miosis).

The face is dry (anhidrosis), red, and warm.


Your Answer. b
Correct Answer. b

(157).

Lebers optic neuropathy :


a. Typically presents in the fourth decade of life
b. Males do not transmit the disease
c. Is inherited in autosomal X-Link fashion
d. The optic disc is pale early in the disease
Solution. (b) Males do not transmit the disease
Ref.:Read the text below
Sol :
Lebers Optic Neuropathy
A mitochondrial inherited disease.
Bilateral loss of central visionwhich is severed and painless.
Typically in the second decade of life.
Classic early picture shows a triad of circumpapillary telangiectatic microangiopathy, pseudoedema of the disc and absent
fluorescein staining.
Your Answer. b
Correct Answer. b

Copyright 2014 Delhi Academy of Medical Sciences, All Rights Reserved.

63/119

(158).

In acute APMPPE(Acute Posterior Multifocal Placoid Pigment Epitheliopathy(APMPPE)):


all are true except :
a. Bilateral in the majority of cases
b. Hypofluorescence in early stage of fluorescein angiography
c. Spontaneous recovery is common
d. 80% of patients have exudative vasculitis
Solution. (d) 80% of patients have exudative vasculitis
Ref.:Read the text below
Sol :
APMPPE(Acute Posterior Multifocal Placoid Pigment Epitheliopathy(APMPPE)):
o Typically affects healthy young adults who presents with sudden onset central or paracentral visual loss after a flue like illness.
o Scattered, patchy creamy lesions at the level of the retinal pigment epithelium layers.
o The lesions fade after one to two weeks leaving behind granular pigmentary changes.
o Fluorescein angiography shows early blockage of choroidal circulation by these lesions but in the late phase the lesions
show late staining.
o Other findings : Uveitis, serous retinal detachment, cerebralvasculitis, cerebrospinal fluid pleocytosis, headache, hearing loss
and tinnitus.
o Spontaneous resolution is common and systemic steroid has not been shown to be useful.
Your Answer. c
Correct Answer. d

(159).

Which conjunctivitis is least likely to occur bilaterally?


a. Allergic
b. Viral
c. Bacterial
d. Vernal
Solution. (c) Bacterial.
Ref Read the text below
Sol:
Allergies are likely to affect both eyes and present with itching and watering.
Vernal is a type of seasonal allergy you see in young boys.
Viral conjunctivitisusually starts in one eye, but hops to the other eye as it is very contagious.
Bacterial conjunctivitiscan occur bilaterally, but of the available choices is most likely to occur in just one eye.
Your Answer. c
Correct Answer. c

(160).

A 75-year-old man develops unilateral, painless loss of vision associated with pallor of the optic disc, a cherry-red fovea, and
bloodless arterioles. These findings are most consistent with which of the following?
a. Central retinal vein occulusion
b. Acute glaucoma
c. Acute anterior uveitis
d. Central retinal artery occlusion
Solution. : (d)Central retinal artery occlusion
Reference Read the text below
Sol:

The patient has central retinal artery occlusion. This is characterized by a sudden, complete, painless loss of vision in one eye,
most commonly n an elderly patient.

Retinal examination reveals pallor of the optic disc; edema of the retina; a cherry-red fovea;bloodless, constricted
arterioles; and a boxcar segmentation of blood in the retinal veins.

Surgical decompression of the anterior chamber in less than 1 hourafter occlusion prevents permanent damage to the eye.
Your Answer. d
Correct Answer. d

Copyright 2014 Delhi Academy of Medical Sciences, All Rights Reserved.

64/119

(161).

Keratomalacia is associatedwith which of the following infections? Herpes simplex


a. Herpes simplex
b. Varicella zoster
c. Measles
d. Diphtheria
Solution. (c) Measles
Ref.:Read the text below
Sol :
Keratomalaciais often precipitated by an acute systemic illness such as measles, pneumonia, or severe diarrhea.
Keratomalacia is common in developingcountries and affects poorly nourished children who are deficient in Vitamin A, often early
in the first year of life.
The cornea becomes dull and insensitive, finally the whole tissue undergoes necrosis and seems to melt away
(keratomalacia)
Your Answer. c
Correct Answer. c

(162).

What is the treatment of choice for congenital ptosis?


a. Levator resection
b. Fasanella Servat procedure
c. Modified wheller procedure
d. Bick procedure
Solution. (a) Levator resection
Ref.:Read the text below
Sol :
Treatment of children with mild to moderate congenital ptosis is usually by levator resection during the pre-school years.
PTOSIS :
Plosis can be classified into myogenic, neurogenic, aponeurotic, mechanical, traumatic, and pseudoptosis, based on
etiology.
The most common cause for congenital ptosis is myogenic, due to improper development of the levator muscle.
In adults the most common type is involutional ptosis due to aponeurotic dehiscence
Your Answer. b
Correct Answer. a

Copyright 2014 Delhi Academy of Medical Sciences, All Rights Reserved.

65/119

(163).

Which of the following is an inert metal that can be left alone in eye?
a. Iron
b. Copper
c. Lead
d. Nickel
Solution. (c) Lead
Ref.:Read the text below
Sol :
INTRAOCULAR FOREIGN BODIES
They can be classified into the following types.
Frequently produce severe inflammatory reactions
Magnetic : Iron, steel, tin
Non magnetic : Copper and vegetable matter
Typically produce mild inflammatory reactions
Magnetic : Nickel
Nonmagnetic : Aluminium, mercury, zinc, vegetable matter.
Inert foreign bodies
Carbon, coal, glass, gold, lead, stone
Your Answer. d
Correct Answer. c

(164).

Which of the following results in bitemporal hemianopia?


a. Craniopharyngioma
b. Optic nerve glioma
c. Posterior cerebral artery occlusion
d. AION
Solution. (a) Craniopharyngioma
Ref.:Read the text below
Sol :
Tumours of pituitary body arethe most common cause of bitemporal hemianopia, but suprasellar tumours particularly
craniopharygioma and suprasellar meningioma also require consideration.
Your Answer. a
Correct Answer. a

Copyright 2014 Delhi Academy of Medical Sciences, All Rights Reserved.

66/119

(165).

A 73-year-old man with a history of hypertension complains of a 10-min episode of left-sided weakness and slurred speech. On
further questioning, he relates three brief episodes in the last month of sudden impairment of vision affecting the right eye. His
examination now is normal.The episodes of visual loss are most likely related to
a. Retinal vein thrombosis
b. Central retinal artery ischemia
c. Posterior cerebral artery ischemia
d. Middle cerebral artery ischemia
Solution. (b)Central retinal artery ischemia
Reference Read the text below
Sol:

The presumed mechanism of transient monocular blindness in carotid artery disease is embolism to the central retinal
artery or one of its branches.

Although classic teaching has emphasized the role that cholesterol emboli play in causing this blindness, it has been noted that
cholesterol emboli (Hollenhorst plaques) may be seen on fundoscopic examination even of asymptomatic individuals.

Retinal vein thrombosismay produce a rapidly progressive loss of vision with hemorrhages in the retina, but would not be
associated with the TIAs described here.

Although both posterior and middle cerebral artery ischemia can cause visual loss, they would not be expected to cause the
monocular blindness described here.

Posterior ciliary artery ischemiacan cause ischemic optic neuropathy, but this is usually acute, painless, and not associated with
preceding transient monocular blindness or TIAs.
Your Answer. a
Correct Answer. b

(166).

Use of Digoxin is contraindicated in WPW syndrome because


a. It can precipitate asystole
b. It can precipitate ventricular fibrillation
c. It causes complete AV block
d. They are more susceptible to digoxin toxicity
Solution. (b) It can precipitate ventricular fibrillation
Ref Read the text below
Sol:
Digoxin shortens the refractory potential of the accessory pathway in WPW and thus increases the ventricular rate
predisposing them to ventricular fibrillation (VF)
WPW SYNDROME

Normally, the only conducting pathway between atria and ventricles is the AV node. In individuals with WPW syndrome, there is an
aberrant muscular or nodal tissue connection (Bundle of Kent) between atria and ventricles.

The aberrant pathway either connects left atrium with left ventricle or right atrium with right ventricle.

Bundle of Kent conducts more rapidly than the slowly conducting AV node and thus, one of the ventricles is excited early.
Your Answer. b
Correct Answer. b

(167).

Average normal LES pressureat rest:


a. 10 mm Hg
b. 15 mm Hg
c. 40 mm Hg
d. 30 mm Hg
Solution. (b) 15 mm Hg
Ref Read the text below
Sol:
Normal LES pressure is 10-30 mm of Hg.
Your Answer. d
Correct Answer. b

Copyright 2014 Delhi Academy of Medical Sciences, All Rights Reserved.

67/119

(168).

The most common neoplasm associated with achalasia:


a. Esophageal squamous cell carcinoma
b. Gastric squamous cell Carcinoma
c. Gastric adenocarcinoma
d. Gastric lymphoma
Solution. (c) Gastric adenocarcinoma
Ref Read the text below
Sol:

Tumor infiltration especially carcinoma in gastric fundus is the most common malignant cause of achalasia.
Your Answer. b
Correct Answer. c

(169).

Type of peptic ulcer associated with highest recurrence rate following proximal gastric vagotomy is:
a. Post-pyloric ulcer
b. Pre-pyloric ulcer
c. Intra-pyloric ulcer
d. Post-bulbar ulcer
Solution. (b) Pre-pyloric ulcer
Ref Read the text below
Sol:
Highly selective vagotomy is procedure of choice in elective surgery except in situation where ulcer recurrence rates are high.
Your Answer. b
Correct Answer. b

(170).

According to JRSGC, the morphological classification of EGC, an excavated lesion involving mucosa and submucosa belong to:
a. Type I
b. Type II
c. Type III
d. Type IV
Solution. (c) Type III
Ref Read the text below
Sol:

Type O: Superficial, Flat tumor with or without minimal elevation or depression

Type I poly tumor

Type II a-elevated, b-flat, c-depressed

Type III excavated.


Your Answer. b
Correct Answer. c

Copyright 2014 Delhi Academy of Medical Sciences, All Rights Reserved.

68/119

(171).

DLCO is a very sensitive test for interstitial lung diseases, in which it is decreased. Which among the following conditions cause
an increase in DLCO?
a. Pulmonary fibrosis
b. Pulmonary artery hypertension
c. Alveolar hemorrhage
d. ILD
Solution. (c) Alveolar hemorrhage
Ref Read the text below
Sol:
DLCO :
Diffusion capacity (or Transfer factor) of lung Carbon monoxide

Normal in asthma

Reduced in all restrictive lung diseases and pulmonary HT


Increased in alveolar hemorrhage like Goodpasture syndrome and Congestive heart failure
Your Answer. b
Correct Answer. c

(172).

Gastric mucous gel thickness is not decreased by:


a. Gastric distension
b. Pepsin
c. NSAIDs
d. N-Acetyl cysteine
Solution. (a) Gastric distension
Ref Read the text below
Sol:
Gastric mucous gel thickness is affected by

Pepsin

NSAIDs

N-Acetyl cysteine
Your Answer. d
Correct Answer. a

(173).

The BODE index, a simple multidimensional grading system, is better than the FEV 1 at predicting the risk of death from any cause
and from respiratory causes among patients with COPD.Which of the following is not the component of BODE index?
a. BMI
b. Oxygen requirement
c. Dyspnoea
d. Exercise performance
Solution. (b) Oxygen requirement
Ref Read the text below
Sol:
BODE index includes
1. BMI
2. Airflow Obstruction
3. Dyspnoea
4. Exercise performance
Your Answer. c
Correct Answer. b

Copyright 2014 Delhi Academy of Medical Sciences, All Rights Reserved.

69/119

(174).

Which form of anemia most commonly occurs in postgastrectomy patients?


a. Iron deficiency
b. Vitamin B12 deficiency
c. Folate deficiency
d. Hemolytic anemia
Solution. (a) Iron deficiency
Ref Read the text below
Sol:

Iron deficiency is a commoncause of anemia in postgastrectomy patients.

This condition is related to reduce postoperative acid secretion and a decreased ability to dissociate ferric iron from food.

Folate and vitamin B12 deficiencies can also develops in these patients, but these deficiencies are much less common.
Your Answer. b
Correct Answer. a

(175).

Non-surgical management of peptic ulcer perforationincludes each of the followingexcept:


a. Nasogastric suction
b. .Intravenous H2 receptor antagonists
c. Antibiotics
d. Octreotide
Solution. (d) Octreotide
Ref Read the text below
Sol:

Poor surgical candidatesand patients with a negative result from a gastrografin swallow can be managed conservatively if there
are no sign of generalized peritonitis.
There is no evidence suggesting that octreotide is of any benefit in the management of these cases.
Your Answer. d
Correct Answer. d

(176).

Flapping tremorsoccurs in all except :


a. Uremic encephalopathy
b. Parkinsonism
c. Hepatic encephalopathy
d. CO2 poisoning
Solution. (b) Parkinsonism
Ref: Read the text below
Sol :
Asterixis(also called the flapping tremor, or liver flap) or wing beating tremor
Tremor is of 2-3 second cycle
Seen in uremic, hepatic, metabolic encephalopathy,C02 retention ,Alcohol intoxication
Some Important tremors :
Intention tremor: cerebellar dysfunction
Resting tremoror pin rolling tremor : Parkinsonism
Essential tremor(sometimes called benign essential tremor) is the most common type of tremor
Orthostatic tremoris characterized by fast (> 12 Hz) rhythmic muscle contractions that occur in the legs and trunk immediately after
standing seen along with essential tremor.
Physiologic tremor occurs in every normal individual and has no clinical significance.
Your Answer. b
Correct Answer. b

Copyright 2014 Delhi Academy of Medical Sciences, All Rights Reserved.

70/119

(177).

Mid diastolic murmurwith pre systolic accentuation is seen in?


a. Aortic stenosis
b. MVP
c. Mitral regurgitation
d. Mitral stenosis
Solution. (d) Mitral stenosis
Ref: Read the text below
Sol :
The firstheart sound (S1) is usually accentuated and slightly delayed. The pulmonic component of the second heart sound (P2) also is
often accentuated, and the two components of the second heart sound (S2) are closely split.
The opening snap (OS) of the mitral valve is most readily audible in expiration at, or just medial to the cardiac apex. This sound
generally follows the sound of aortic valve closure (A2) by 0.05 0.12 s. The time interval between A2 and OS varies inversely with
the severity of the MS.
The OS is followed by a low pitched, rumbling, diastolic murmur, heard best at the apex with the patient in the left lateral
recumbent position.
Your Answer. d
Correct Answer. d

(178).

Low erythropoietin levelis seen in?


a. Aplastic anemic
b. Renal failure
c. Obesity
d. Hematoma
Solution. (b) Renal failure
Ref: Read the text below
Sol :
ERYTHROPOIETIN:
Sources of erythropoietin: 85% from kidney and 15% from liver and brain. (Interstitial cells in the peritubular capillary bed of
the kidneys and by perivenous hepatocytes in the liver)
Stimulus for erythropoietin secretionis : Hypoxia (most potent), Cobalt salts and Androgens
Secretion of the hormoneis facilitated by the alkalosis and catecholamines
Function: For red cell production erythropoietin (EPO) is the regulatory hormone. EPO is required for the maintenance of committed
erythroid progenitor cells that, in the absence of the hormone, undergo programmed cell death (apoptosis).
Your Answer. b
Correct Answer. b

(179).

In hemochromatosis, loss of libido and atrophy of testis occurs due to:


a. Deposition of iron in testis
b. Deposition of iron in pitituary
c. Deposition of iron in the thalamus
d. Reduced blood flow to testis
Solution. Deposition of iron in pitituary
Ref.:Read the text below
Sol :
Hypogonadism occursin both sexes : can antedate other clinical features.
Manifestations include loss of libido, impotence, amenorrhea, testicular atrophy, gynecomastia, and sparse body hair.
These changes are primarily the result of decreased production of gonadotropins due to impairment of hypothalamicpituitary function by iron deposition in pituitary
Your Answer. b
Correct Answer. b

Copyright 2014 Delhi Academy of Medical Sciences, All Rights Reserved.

71/119

(180).

Most common organism implicated in Spontaneous Bacterial

Peritonitis in adults is :

a. E. Coli
b. Staph. Aureus
c. Strep. Viridians
d. Enterococcus
Solution. (a) E. Coli
Ref.:Read the text below
Sol :
Spontaneous Bacterial Peritonitis (SBP)
Common and severe complication of ascites characterized by spontaneous infection of ascetic fluid without an intraabdominal source.
Can occur in upto 30% of individuals with cirrhosis and ascites severe enough for hospitalization
Mechanism Bacterial translocation with gut flora traversing the intestine into mesenteric lymph nodes leading to bacteremia and
seeding into ascetic fluid.
Most common organism implicated is E. Coli
Your Answer. a
Correct Answer. a

(181).

Early loss of bladder controlseen in?


a. AMLS
b. Conus medullaris
c. Caudal equine
d. Guillain barre syndrome
Solution. (b) Conus medullaris
Ref: Read the text below
Sol :
Even in the late stages of the illness, sensory, bowel and bladder, and cognitive functions are preserved.
Even when there is severe brainstem disease, ocular motility is spared until the very late stages of the illness
About conus medullaris :
The distinctive conus syndrome consists of bilateral saddle anesthesia (S3-S5), prominent bladder and bowel dysfunction (urinary
retention and incontinence with lax anal tone), and impotence.
Thebulbocavernosus (S2-S4) and anal (S4-S5) reflexes are absent
Your Answer. b
Correct Answer. b

(182).

Burst EEG patternis seen in?


a. Herpes simplex encephalitis
b. Absent seizures
c. Myoclonic epilepsy
d. SSPE
Solution. (b) Absent seizures
Ref: Read the text below
Sol :
EEG findings :
Herpes simplex encephalitis: periodic focal temporal lobe spikes on a background of slow or low-amplitude (flattened)
Absence seizures: The electrophysiologic hallmark of typical absence seizures is a generalized, symmetric, 3-Hz spike-and-wave
discharge that begins and ends suddenly, superimposed on a normal EEG background.
Periods of spike-and-wave discharges lasting more than a few seconds usually correlate with clinical signs, but the EEG often shows
many more brief bursts of abnormal cortical activity than were suspected clinically.
Hyperventilation tends to provoke these electrographic discharges and even the seizures themselves and is routinely used
when recording the EEG.
Your Answer. a
Correct Answer. b

Copyright 2014 Delhi Academy of Medical Sciences, All Rights Reserved.

72/119

(183).

Which of the following tests is most sensitive for detecting early diabetic nephropathy?
a. Serum creatinines
b. Creatinine clearance
c. Microalbuminuria
d. Ultra sonography
Solution. (c) Microalbuminuria
Ref: Read the text below
Sol :
The first clinically detectable sign of diabetic nephropathy is microalbuminuria.
The most sensitive method to detect microalbuminuria is by radioimmunoassay
The renal biopsy is the only test of the effect of diabetes on the kidney that is more sensitive than microalbuminuria.
Your Answer. c
Correct Answer. c

(184).

Most common symptom of Henoch Schnolein purpura is?


a. Intussception
b. Purpura
c. Edema
d. Vomiting
Solution. (b) Purpura
Ref: Read the text below
Sol :
The frequency of involvement is : Skin Rash > Arthritis > GIT > Renal
Your Answer. d
Correct Answer. b

(185).

Which drug is usually not preferred in treatment of HIV with TB?


a. Efavirenz
b. Lamivudine
c. Stavudine
d. Nevirapine
Solution. (d) Nevirapine
Ref: Read the textbelow
Sol :
FIRST LINE ART IN HIV-TB IN INDIA
Regimen II

Zidovudine + Lamividine + Efavirenz

Preferred for patients on ANTI-TB Drugs

Regimen II a

Stavudine + Lamuvidine + Efavirenz

Preferred for patients on ANTI-TB Drugs and Hb <9gm/dI

Your Answer. c
Correct Answer. d

Copyright 2014 Delhi Academy of Medical Sciences, All Rights Reserved.

73/119

(186).

Alternative parenteral drug for chloropuine resistantmalaria is?


a. Mefloquine
b. Sulfamethoxazole
c. Artemesim
d. Dihydroartemesin
Solution. (c) Artemesim
Ref: Read the text below
Sol :
For treatment of chloroquine resistant malaria :
Artmesins combined with other partner madications (ACT)
Amodiaquine
Mefloquine
Doxycycline
Quinine
Your Answer. c
Correct Answer. c

(187).

Poor prognosis in Maddreys index is above?


a. 24
b. 28
c. 32
d. 36
Solution. (c) 32
Ref: Read the text below
Sol :
Modified Maddreys discriminant function was originally described by Maddrey and Boitnott to predict prognosis in
alcoholic hepatitis.
It is calculated by a simple formula : (4.6 x (PT test control) ) + S. Bilirubin in mg/dl)
Prospective studies have shown that, it is useful in predicting short term prognosis especially mortality within 30 days. A value more
than 32 implies poor outcome with one month mortality ranging between 35% to 45%. It also indicates need for corticosteroid
therapy.
To calculate Maddrey discriminantfunction using SI units micromol/I (i.e. not US) divide bilirubin value by 17.
A specific model called LILLES model was designed to identify patients who even at a score of 32 are not responsive to
corticosteroids.
Your Answer. c
Correct Answer. c

Copyright 2014 Delhi Academy of Medical Sciences, All Rights Reserved.

74/119

(188).

Best method to diagnose typhoidin the first week is?


a. Blood culture
b. Bone marrow culture
c. Urine culture
d. Stool culture
Solution. (b) Bone marrow culture
Ref: Read the text below
Sol :
DIAGNOSIS OF TYPHOID

1st week

Both blood culture and bone marrow culture show high sensitivity, however, blood cultures are affected by
prior intake of antibiotics, but bone marrow cultures are more specific and sensitive and not affected by
antibiotics, hence making them a better diagnostic test than blood culture in first week of typhoid infection.
Sensitivity of blood culture : 60-90%
Sensitivity of bone marrow culture : >95%

2nd week

Widal test

3rd and 4th week

Stool and urine culture. Sensitivity > 70%.


Obsolete now and rarely used.

Your Answer. a
Correct Answer. b

(189).

Investigation of choice for diagnosis of Epilepsy is?


a. EEG
b. CT scan
c. MRI
d. CSF investigations
Solution. (a) EEG
Ref: Read the text below
Sol :
All patients who have a possible seizure disorder should be evaluated with an EEG as soon as possible.
The EEG is most useful in evaluating patients with suspected epilepsy. The presence of electrographic seizure activity i.e., of
abnormal, repetitive, rhythmic activity having an abrupt onset and termination and a characteristic evolution clearly
establishes the diagnosis.
Your Answer. a
Correct Answer. a

Copyright 2014 Delhi Academy of Medical Sciences, All Rights Reserved.

75/119

(190).

Which among the following is not a feature of Hemobilia?


a. Malena
b. Jaundice
c. Fever
d. Biliary Colic
Solution. (c) Fever
Ref.:Read the text below
Sol :
Hemobilia is definedas bleeding into the biliary tree from an abnormal communication between a blood vessel and bile duct.
The most common causes of hemobilia in modern times are iatrogenic trauma, accidental trauma, gall stones, tumors,
inflammatory disorders, and vascular disorders.
Quinkes triad of upper abdominal pain, upper gastrointestinal haemorrhage and jaundice is classical but only present in 22%
cases.
The symptoms and signs of major hemobilia are melena (90% of cases), hematemesis (60% of cases), biliary colic (70% of cases), and
jaundice (60% of cases).
Your Answer. d
Correct Answer. c

(191).

Xanthogranulomatous pyelonephritisassociated stones are due to

infection with?

a. Proteus
b. E coli
c. Pseudomonas
d. Klebsiella
Solution. (a) Proteus
Ref.:Read the text below
Sol :
Xanthogranulomatous pyelonephritis is a chronic condition of the kidney resulting in diffuse renal destruction, a grossly
enlarged and nonfunctioning kidney associated with nephrolithiasis.
It is thought to start with obstruction (stone or sloughed papilla) followed by infection (most commonly proteus)
Your Answer. c
Correct Answer. a

(192).

Supero-lateral boundaryof axillary dissection is?


a. Clavi-pectoral fascia
b. Brachial plexus
c. Axillary artery
d. Axillary vein
Solution. (d) Axillary vein
Ref.:Read the text below
Sol :
Axillary node clearance can be defined as clearing the axillary contents bounded by the axillary skin laterally, latissimus dorsi,
teres major and subscapularis posteriorly, the lower border of the axillary vein superiorly, pectoralis muscles anteriorly, and the chest
wall medially.
The levels of axillary nodes are anatomically defined as level one (inferolateral to pectoralis minor), level two (posterior to pectoralis
minor) and level three (superomedial to pectoralis minor).
In reality, these nodes are in continuity with each other but the concept of axillary node levels is useful when discussing the extent
of axillary node surgery
Your Answer. d
Correct Answer. d

Copyright 2014 Delhi Academy of Medical Sciences, All Rights Reserved.

76/119

(193).

Distal gastrectomy without vagotomy has traditionally been the procedure of choice for: a. Type I gastric ulcer.
b. Type II gastric ulcer.
c. Type III gastric ulcer.
d. Hiatal hernia
Solution. (a) Type I gastric ulcer.
Ref: Sch8- 962-963.
Sol:
Distal Gastrectomy

Distal gastrectomy without vagotomy (usually about a 50% gastrectomy to include the ulcer) has traditionally been the
procedure of choice for type I gastric ulcer.

Reconstruction may be done as a Billroth I (preferable) or Billroth II.


Truncal vagotomy is added for type II and III gastric ulcers, or if the patient is believed to be at increased risk for recurrent ulcer,
and should be considered if Billroth II reconstruction is contemplated.
Your Answer. a
Correct Answer. a

(194).

Procedure of choice for bleeding gastric ulcer is : a. Distal gastric resection.


b. Vagotomy and drainage.
c. Vagotomy and drainage with oversewing of the ulcer.
d. Highly-selective vagotomy
Solution. (a) Distal gastric resection.
Ref: Sch8- 967.
Sol:
Bleeding Gastric Ulcer

Bleeding gastric ulcers tend to occur in older and/or medically complicated patients, and this fact tends to increase the
operativerisk.

Patients with gastric ulcer bleeding who are most likely to require surgery have bled more than 6 units and have presented in
shock.

Endoscopically, their ulcers tend to be on the lesser curvature with the usual stigmata of recent hemorrhage.

Distal gastric resection to include the bleeding ulcer is the procedure of choice for bleeding gastric ulcer.
Second best is vagotomy and drainage with oversewingand biopsy of the ulcer.
Your Answer. d
Correct Answer. a

Copyright 2014 Delhi Academy of Medical Sciences, All Rights Reserved.

77/119

(195).

The most common complication of biliopancreatic diversion is : a. Anemia.


b. Protein-calorie malnutrition.
c. Dumping syndrome
d. Hypocalcemia
Solution. (a) Anemia.
Ref: Read the text below.
Sol:
BILIOPANCREATIC DIVERSION
Mechanism of Action

The procedure combines gastric restriction with an intestinal malabsorptive procedure.

A 50- to 100-cm common absorptive alimentary channel is created proximal to the ileocecal valve; digestion and absorption are
limited to this segment of bowel.
Indications

This procedure is primarily indicated for the superobese or for those who have failed restrictive bariatric procedures.
Contraindications

Patients with anemia, hypocalcemia, and osteoporosis, and those who are not motivated to comply with stringent postoperative
supplementation regimens may not be appropriate for this procedure.
Complications

The most common morbidities include anemia (30%), protein-calorie malnutrition (20%), dumping syndrome,and marginal
ulceration (10%).

The duodenal switch modification is associated with a lower ulceration rate (1%) and a lower incidence of dumping
syndrome.
Other complications include vitamin B12deficiency, hypocalcemia, fat-soluble vitamin deficiencies, osteoporosis,night blindness, and
prolongation of prothrombin time.
Your Answer. b
Correct Answer. a

(196).

Choose the incorrect statement regarding Campylobacters jejuni ?


a. Campylobacters are motile, curved, oxidasepositive, gram-negative rods
b. C. jejuni grows well only on enriched media under microaerophilic conditions
c. The cells have polar flagella.
d. It break down carbohydrates,uses metabolic intermediates for energy.
Solution. (d) It break down carbohydrates,uses metabolic intermediates for energy.
Ref: Medical microbiology by C.George ray 379.
Sol:
Campylobacters jejuni

Campylobacters are motile, curved, oxidase-positive, Gram-negative rods similar in morphology to vibrios.

The cells have polar flagella and are often are attached at their ends giving pairs S shapes or a seagull appearance.

C. jejuni grows well only on enriched media under microaerophilic conditions. That is, it requires oxygen at reduced tension (5
10%), presumably due to vulnerability of some of its enzyme systems to superoxides.

Growth usually requires 2 to 4 days, sometimes as much as a week.

C. jejuni has the structural components found in other Gram-negative bacteria (eg, outer membrane,LPS).
In contrast to the vibrios, it does not break down carbohydrates, but uses amino acids and metabolic intermediates for energy.
Your Answer. c
Correct Answer. d

Copyright 2014 Delhi Academy of Medical Sciences, All Rights Reserved.

78/119

(197).

Which of the following conditions are associated with the development of esophageal carcinoma?
a. Caustic esophageal stricture
b. Achalasia of the esophagus
c. Plummer-Vinson syndrome
d. All of above.
Solution. (d) All of above.
Ref - Read the text below
Sol:

Chronic irritation of the esophageal mucosa by a variety of noxious stimuli (alcohol, tobacco, hot foods and liquids) eventually
may lead to the development of esophageal carcinoma.

A variety of other esophageal lesions have a recognized premalignant nature.

The patient who survives the initial injury long enough to develop a caustic esophageal stricture has a 1000-fold increased
risk of developing carcinoma compared with the normal population. Ten to 12% of patients with achalasia of the esophagus
who are observed 15 years or more develop esophageal carcinoma.

This is thought to be related to the irritating effects of the fermenting intraesophageal contents on the adjacent esophageal mucosa.
Plummer-Vinson syndrome is a premalignant esophageal condition. Patients with this syndrome are typically elderly women
who have cervical dysphasia and iron deficiency anemia.

About 10% of patients will develop squamous cell carcinoma of the hypopharynx, oral cavity or esophagus. Finally, there have been
isolated reports of esophageal carcinomas found incidentally within esophageal diverticula, presumably as the result of the irritating
effects on the mucosa of stagnant, putrefying food within the pouch.
Esophageal diverticula are therefore regarded as premalignant esophageal lesions although this occurrence is extremely
rare.
Your Answer. d
Correct Answer. d

(198).

Testicular cancer is common in


a. Ectopic testis
b. Undescended abdominal testis
c. Atrophic testis
d. Anteverted testis
Solution. (b) Undescended abdominal testis
Ref: Read the text below
Sol :
The exact cause of testicular cancer is unknown. There is no link between vasectomyand testicular cancer. Factors that may increase a
man's risk for testicular cancer include:

Abnormal testicle development

History of testicular cancer

History of undescended testicle

Klinefelter syndrome
Other possible causes include exposure to certain chemicals and HIVinfection. A family history of testicular cancer may also
increase risk.
Testicular cancer is the most common form of cancer in men between the ages of 15 and 35. It can occur in older men, and
rarely, in younger boys.
Your Answer. b
Correct Answer. b

Copyright 2014 Delhi Academy of Medical Sciences, All Rights Reserved.

79/119

(199).

The first investigation of choice in a patient with suspected subarachnoid haemorrhage should be:
a. Non-contrast computed tomography
b. CSF examination.
c. Magnetic resonance imaging (MRI).
d. Contrast-enhanced computed tomography.
Solution. (a) Non-contrast computed tomography
Ref: Read the text below.
Sol:

Intracranial hemorrhage is often discovered on non contrast CT imaging of the brain during the acute evaluation of stroke.
CT is more sensitive than routine MRI for acute blood. The location of hemorrhage narrows the differential diagnosis to a few
entities.
Your Answer. b
Correct Answer. a

(200).

Which is the most common complication of pancreas division ?


a. Obstructive jaundice
b. Duodenal obstruction
c. Recurrent acute pancreatitis
d. Peptic ulcer
Solution. (c) Recurrent acute pancreatitis
Ref: Sabiston, Textbook of Surgery- 1591
Sol :
Pancreas division:
Is the most common congenital anomaly involving the pancreas and occurs in up to 1% of individuals, is rarely the cause of
pancreatitis.
Pancreas division has also been described as a cause of obstructive pancreatitis.
Incomplete fusion of the two pancreatic anlagencreates pancreas division and predisposes to recurrent pancreatitis.
Your Answer. a
Correct Answer. c

(201).

Which one of the following procedures is contraindicated in the treatment of hypospadias ?


a. Chorde correction
b. Circumcision
c. Urethroplasty
d. Meatotomy
Solution. (b) Circumcision
Ref: Read the text below
Sol :
Hypospadias :
Incidence

Most common congenital malformation of urethra


1 in every 350 males

Variety

Glandular most common variety


Coronal external meatus at corona glandis
Penile variety chordae is prominent feature
Penoscrotal
Perineal associated with bilateral undescended testis

Treatment

No treatment required in glandular variety


Circumcision should be avoided until hypospadias has been repaired as procedures make use of prepuceal skin

Your Answer. d
Correct Answer. b

Copyright 2014 Delhi Academy of Medical Sciences, All Rights Reserved.

80/119

(202).

An air fluid level with dilated oesophagus and bird beak appearance in a barium swallow is diagnostic of which one of the
following ?
a. Achalasia cardia
b. Carcinoma oesophagus
c. Hiatus hernia
d. Barretts oesophagus
Solution. (a) Achalasia cardia
Ref: Bailey and Loves short Practice of Surgery- 883,
Sol :
Radiological features

Examples

Birds beak appearance of esophagus on barium meal

Achalasia cardia

Rat tail appearance of distal esophagus

Carcinoma of distal esophagus

Curling esophagus

Diffuse esophageal spasm

Cork screw appearance of esophagus on barium meal Diffuse esophageal spasm


Nut craker esophagus

Diffuse esophageal spasm

Your Answer. b
Correct Answer. a

(203).

What is the ideal treatment of choledochal cyst ?


a. Conservative management
b. Cystojejunostomy
c. Cystogastrostomy
d. Cyst excision and biliary enteric reconstruction
Solution. (d) Cyst excision and biliary enteric reconstruction
Ref: Harrisons Principles of Internal Medicine- 1888
Sol :
Triad of choledochalcyst (in one-third of patients)
a. Abdominal pain
b. Jaundice
c. Abdominal mass
Treatment excision of the cystand biliary enteric anastomosis
Your Answer. d
Correct Answer. d

Copyright 2014 Delhi Academy of Medical Sciences, All Rights Reserved.

81/119

(204).

Gall stones in childrenis caused by all except:


a. Prematurity
b. Obesity
c. Sickle cell anemia
d. Leptospira interrogans infection
Solution. (d) Leptospira interrogans infection
Ref.:Read the text below
Sol :
Conditions Associated with Cholelithiasis:
Chronic hemolytic disease (sickle cell anemia, spherocytosis)
Obesity Ileal resection of disease
Cystic fibrosis
Chronic liver disease
Crohns disease
Prolonged parenteral nutrition
Prematurity with complicated medical or surgical course
Prolonged fasting or rapid weight reduction
Treatment of childhood cancer
Abdominal surgery
Pregnancy
Your Answer. a
Correct Answer. d

(205).

Blood born spreadis a feature of?


a. Carcinoma
b. Sarcoma
c. Dysplasia
d. Metaplasia
Solution. (b) Sarcoma
Ref.:Read the text below
Sol :
Hematogenous spread is typical of sarcomas and Lymphatic spread is typical of carcinomas
However the above is not a rule and there are sarcomas that spread through lymphatics. The sarcomas that spread through lymplatics
are.
Synovial cell sarcoma
Angiosarcoma
Malignant fibrous Histocytoma
Epithelial sarcoma
Clear cell sarcoma
Lymphosarcoma
Rhabdomyosarcoma
Your Answer. b
Correct Answer. b

Copyright 2014 Delhi Academy of Medical Sciences, All Rights Reserved.

82/119

(206).

Potato nodesare feature of?


a. Sarcoidosis
b. Tuberculosis
c. Carcinoid
d. Lymphoma
Solution. (a) Sarcoidosis
Ref.:Read the text below
Sol :
Sarcoidosis is a systemicdisease characterized by non-caseating granulomas in multiple organs.
In 90% of cases, symmetrical massive bilateral hilar lymphadenopathy clear or the cardiac border (potato nodes) or lung involvement
is present and can be revealed by chest x-ray or Trans bronchial biopsy.
Your Answer. a
Correct Answer. a

(207).

Pascals law is used in which technique of hernia repair?


a. Lichtensteins mesh repair
b. Stoppas preperitoneal repair
c. Bassinis repair
d. Darning rapair.
Solution. (b) Stoppas preperitoneal repair
Ref.:Read the text below
Sol :
In stoppas preperitoneal repair the expanding intra-abdominal pressure holds the mesh in place without suture fixation.
According to Pascals law any additional pressure exerted upon an enclosed fluid mass is transmitted equally in all directions.
Use of a large mesh extending 3-5 cm beyond the edge of the defect on all sides utilizes Pascals Law.
Your Answer. b
Correct Answer. b

(208).

Which of the following is not a sign of appendicitis?


a. Rovsings sign
b. Murphys sign
c. Obturator sign
d. Psoas sign
Solution. (b) Murphys sign
Ref.:Read the text below
Sol :
Peritoneal irritationcan be elicited on physical examination by the findings of voluntary and involuntary guarding, percussion, or
rebound tenderness.
Dunphys sign: Any movement, including coughing, may cause increased pain.
Revsings sign :Pain in the right lower quadrant during palpation of the left lower quadrant.
Your Answer. b
Correct Answer. b

Copyright 2014 Delhi Academy of Medical Sciences, All Rights Reserved.

83/119

(209).

Interstitial cystitisis also known as?


a. Eosinophilic systitis
b. Radiation cystitis
c. Hunners cystitis
d. Tubercular cystitis
Solution. (c) Hunners cystitis
Ref.:Read the text below
Sol :
Interstitial cystitis (Hunners Ulcer, Submucous Fibrosis)
Definition of Bladder Pain Syndrome/Painful Bladder syndrome/Interstitial cystitis syndrome : An unpleasant sensation (pain,
pressure, discomfort) perceived to be related to the urinary bladder, associated with lower urinary tract symptoms of more than 6
weeks duration, in the absence of infection or other identifiable causes.
Your Answer. a
Correct Answer. c

(210).

TB epididymoorchitisinvolves which part of epididymis?


a. Upper pole
b. Lower pole
c. Head
d. Body
Solution. (b) Lower pole
Ref.: Readthe text below
Sol :
In c/c tuberculous epididymoorchitis, typically, there is a firm discrete swelling of the lower pole of the epididymis.
Your Answer. a
Correct Answer. b

(211).

Which of the following is NOT an indication of MOHs micrographic surgery?


a. Small BCC (lesion < 2cm)
b. Recurrent SCC
c. Morpheaform BCC
d. Metatypical BCC
Solution. (a) Small BCC (lesion < 2cm)
Ref.:Read the text below
Sol :
MOHs micrographic surgery is a method to excise skin cancer under microscopic control.
MOHs surgery may be considered in a large lesion (> 2 cm) as there is direct correlation between tumor size and recurrence.
MOHS MICROGRAPHIC SURGERY
Involves excision of skin cancer under microscopic control
Advantages
Accurate histological margin evaluation
Minimum recurrence rates
Cure rates for BCC and SCC are the highest with MOHS, significantly better than the rates for standard excision or any other accepted
method.
Maximum conservation of surrounding normal tissue.
Your Answer. b
Correct Answer. a

Copyright 2014 Delhi Academy of Medical Sciences, All Rights Reserved.

84/119

(212).

Best approachfor carcinoma middle third of esophagus is :


a. Mckeown approach
b. Orringer approach
c. Nissens approach
d. Ivor Lewis approach
Solution. (d) Ivor Lewis approach
Ref.:Read the text below
Sol :
Surgical Approaches for Ca Esophagus
Upper third: Treated mainly by radiotherapy.
If early and operable, Mckeown three phased anastomosis is done.
Procedure : At the beginning, laparotomy is done to mobilize stomach.
Then esophagus is mobilized through thoracotomy. Then esophagus with growth is resected by neck approach and anastomosis
completed.
Middle third: Ivor Lewis operation (Lewis Tanner two phased esophagectomy) Stomach is mobilized through a laparotomy. Then a
thoracotomy is done; resection and anastomosis is done in the thorax.
Your Answer. c
Correct Answer. d

(213).

Parathyroid insufficiencymanifests after thyroidectomy after :


a. 2-5 days
b. 2-5 weeks
c. 3 months
d. 6 months
Solution. (a) 2-5 days
Ref.:Read the text below
Sol :
Most cases present 2-5 days after operation.
Very rarely, the onset may be delayed for 2-3 weeks.
Your Answer. a
Correct Answer. a

(214).

Maximum risk of invasive breast carcinoma is seen with?


a. Complex Fibroadenoma
b. Sclerosing Adenosis
c. Intraductal papilloma
d. Atypical ductal hyperplasia
Solution. (d) Atypical ductal hyperplasia
Ref.:Read the text below
Sol :
Annual risk for development of breast cancer in a woman with lobular carcinoma insitu (LCIS) is slightly less than 1% per year,
and with either atypical ductal hyperplasia (ADH) or atypical blobular hyperplasia (ALH) it is between 0.5% and 1% per year (NOTE :
Annual risk and relative risk provided in the chart are different)
Your Answer. d
Correct Answer. d

Copyright 2014 Delhi Academy of Medical Sciences, All Rights Reserved.

85/119

(215).

Biophysical profileis composed of all except :


a. .Amniotic fluid volume
b. NST
c. Fetal breathing movement
d. Placental maturity
Solution. (d) Placental maturity
Ref.:Read the text below
Sol :
Biophysical Profile
Ultrasound is used to perform a BPP.
The BPP looks at five categories and gives a score of either 0 or 2 each.
Thecategories include :
Amniotic fluid volume.
Fetal tone.
Fetal activity
Fetal breathing movements, and
NST.
A BPP of 8/10 or better is reassuring.
Your Answer. d
Correct Answer. d

(216).

Wave length of USGused in obstetrics :


a. 1.0 2.0 Mhz
b. 5.0-7.5 Mhz
c. 3.5 5.0 Mhz
d. 7.5 5.10 Mhz
Solution. (c) 3.5 5.0 Mhz
Ref.:Read the text below
Sol :
Obstetrical and Gynecological ultrasound examination should be performed by 3.5 MHz and 5.0 MHz frequency transducers.
Transvaginal and Transrectal examinations are performed by 5 MHz and 7.5 MHz frequency transducers.
Breast, thyroid and scrotal ultrasound examinations should be performed by 7.5 MHz and 10.0 MHz transducers.
Your Answer. c
Correct Answer. c

(217).

Which of the following is an example for transplacental carcinogenesis?


a. Vaginal Ca
b. Pancreatic Ca
c. Colon Ca
d. Breast cancer
Solution. (a) Vaginal Ca
Ref.:Shaws Textbook of Gynaecology- 52
Sol :
Adenocarcinoma (clear cell carcinoma) was seen in the vagina in children whose mothers took dethylstilbesterol for threatened
abortion.
Your Answer. c
Correct Answer. a

Copyright 2014 Delhi Academy of Medical Sciences, All Rights Reserved.

86/119

(218).

The most common cause of thrombocytopenia in pregnant mother is:


a. Benign gestational thrombocytopenia
b. Preeclampsia
c. Immune thrombocytopenia
d. Abruptio placenta
Solution. (a) Benign gestational thrombocytopenia
Ref:Read the text below
Sol:

Thrombocytopenia occurs commonly during pregnancy.

The majority of cases are benigngestational thrombocytopenias, but the differential diagnosis should include immune
thrombocytopenia and preeclampsia.

Maternal thrombocytopenia may also be caused by catastrophic obstetric events such as retention of a dead fetus, sepsis,
abruption placenta, and amniotic fluid embolism.
Your Answer. a
Correct Answer. a

(219).

18 year old girl presentswith amenorrhea, anorexia, weight loss and with milk discharge from the nipples. What is thelikely
diagnosis?
a. . HIV
b. Occult carcinoma
c. Anorexia nervosa
d. Hypothyroidism
Solution. (c) Anorexia nervosa
Ref:Read the text below
Sol:

In the above options, hypothyroidism is ruled out as it leads on to weight gain and menorrhagia commonly, though
galactorrhoea can be present.

Though HIV can be associated with anorexia and weight loss,Harrison says, HIV does not seem to have a significant effect on
menstrual cycle outside setting of advanced disease.

Also, galactorrhoea has not been mentioned as a consequence of HIV anywhere. Occult carcinoma, can cause anorexia and
weight loss.
Your Answer. b
Correct Answer. c

(220).

Progesterone only pillscarry the risk of :


a. Hypertension
b. Venous embolism
c. rregular bleeding
d. Ectopic pregnancy
Solution. (c) Irregular bleeding
Ref.:Text book of Gynaecology, - 233.
Sol :
Irregular bleeding p/v is the most common side effect of POP (20%)
Advantages with minipill are :
No increase in
Coagulability
Glucose tolerance
B.P.
Cardiovascular complications
Do not interfere with lactation
Your Answer. d
Correct Answer. c

Copyright 2014 Delhi Academy of Medical Sciences, All Rights Reserved.

87/119

(221).

The ideal treatment for metastatic choriocarcinoma in the lungs in a young women is :
a. Chemotherapy
b. Surgery with radiation
c. Surgery
d. Wait and watch
Solution. (a) Chemotherapy
Ref.:Shaws - 258
Sol :
Since gestational choriocarcinoma(which arises from a hydatidiform mole) contains paternal DNA (and thus paternal antigens), it is
exquisitely sensitive to chemotherapy. The cure rate, even for metastatic gestational choriocarcinoma, is around 90-95%.
At present, treatment with single-agent methotrexateis recommended for low-risk disease, while intense combination regimens including
EMACO (etoposide, methotrexate, actinomycin D, cyclosphosphamideand vincristine(Oncovin) are recommended for intermediate or
high-risk disease.
Your Answer. a
Correct Answer. a

(222).

All of the following are part of manning criteria, except :


a. .Nonstress test
b. Fetal body movement
c. Vibroacoustic movement
d. Fetal respiratory movement
Solution. (c) Vibroacoustic movement
Ref: Dutta - 115
Sol:
Manning criteria : Has 5 components.
The features of normal score are :1. Presence of fetal tone
2. Presence of at least 3 discrete gross fetal movements with in 30 min.e-Test Series in this e-ea
3. Presence of at least one episode of fetal breathing movements in 30 min.
4. Adequate amniotic fluid volume.
5. Reactive Nonstress test This is a continuous electronic monitoring of the fetal heart rate
along with recording of fetal movements. There is an observed association of FHR acceleration
with fetal movements which when present indicate healthy fetus.
The test is said to be reactive when two or more accelerations of more than 15 beats per minutes
above the baseline and longer than 15 seconds in duration are present in 10-20 minutes observations.
Your Answer. c
Correct Answer. c

Copyright 2014 Delhi Academy of Medical Sciences, All Rights Reserved.

88/119

(223).

Antiprogesterone compound RU-486 if effective for inducing abortion if the duration of pregnancy is:
a. 63 days
b. 72 days
c. 88 days
d. 120 days
Solution. (a) 63 days
Ref:Read the text below
Sol:
RU-486 i.e. Mifepristone, is an analogue of the progestin norethindrone which has strong affinity for progesterone receptors
but acts as an antagonist of progesterone.
Given alone, the during was moderately useful in causing abortion of early pregnancy, however the combination of mifepristone
with analogue of prostaglandin E, i.e. misoprostol is very effective.
Your Answer. d
Correct Answer. a

(224).

In a 30-year-old patient presenting at 38 weeks gestation with increased reflexes and hypertension and pedal edema all of the
following can be seen except:
a. SGOT/SGPT increased
b. Increased platelets
c. Increased uric acid levels
d. Increased creatine levels
Solution. (b) Increased platelets
Ref.:Read the text below
Sol :
Variable
Diastolic blood pressure
Convulsions
Blindness
Headaches
Visual symptoms
Oliguria
Upper abdominal pain
Fetal distress
Fetal growth retardation
Intravascular hemolysis
Thrombocytopenia
Blood urea nitrogen (BUN) creatinine, uric acid
levels
Sercum glutamicoxaloacetic transaminase (SGOT),
serum glutamic pyruvic transaminase (SGPT),
lactate dehydrogenase (LDH)

Mild
90-10 mmH
Absent
Absent
Minimal
Minimal
Absent
Absent
Absent
Absent
Absent
Absent
Normal
Normal

Moderate

Severe

100-110 mmHg
Absent
Absent
Mild
Mild
Absent
Absent
Absent
Absent
Absent
Absent
Mildly elevated

> 110 mmHg


Present
Present
Marked, persistent
Market, Persistent
Present
Present
Present
Present
Present
Present
Markedly elevated

Mildly elevated

Markedly elevated

Your Answer. b
Correct Answer. b

Copyright 2014 Delhi Academy of Medical Sciences, All Rights Reserved.

89/119

(225).

Endometrial hyperplasiais often associated with :


a. Cystic terratoma
b. Endodermal tumor
c. Polycystic ovarian syndrome
d. Sertoli leydig cell tumor
Solution. (c) Polycystic ovarian syndrome
Ref.:Read the textbelow
Sol :
Patients at risk for endometrial hyperplasia, like those at risk for endometrial carcinoma, are also at risk for unopposed estrogen
exposure.
This includes women with polycystic ovarian syndrome and estrogen-producing tumors, such as granulose-theca cell tumors.
Your Answer. c
Correct Answer. c

(226).

Ectopic pregnancyis not common in :


a. IUCD use
b. Adenomyosis
c. Endometriosis
d. Tuberculosis
Solution. (b) Adenomyosis
Ref.:Read the text below
Sol :
Risk factors for ectopic pregnancy.
History of sexually transmitted infections or PID (genital TB causes adhesions)
Prior ectopic pregnancy.
Previous tubal surgery.
Prior pelvic or abdominal surgery resulting in adhesions.
Endometriosis.
Current use of exogenous hormones including progesterone or estrogen.
In vitro fertilization and other assisted reproduction DES-exposed patients with congenital abnormalities.
Congenital abnormalities of the fallopian tubes.
Use of an IUCD for birth control.
Your Answer. a
Correct Answer. b

(227).

Among 100 healthy, fertile couples, approximately how many will become pregnant within 1 month if they have regular intercourse?
a. 15
b. 20
c. 35
d. 45
Solution. (b) 20
Ref:Read the text below
Sol:

The fecundability, or monthly probability of pregnancy, is 20% among fertile couples.


Your Answer. b
Correct Answer. b

Copyright 2014 Delhi Academy of Medical Sciences, All Rights Reserved.

90/119

(228).

A 24-year-old woman, gravida 3, para 1, spontaneous abortions 1, presents with irregular vaginal bleeding. She is found to be
pregnant and her serum hCG is 3500 mIU/mL. She has a past medical history significant for diabetes mellitus and mild asthma.
Her BP = 103/68, P = 88, and T = 98.8. Transvaginal ultrasound reveals a uterus with no gestational sac present and a 2-cm right
adnexal mass. The least invasive treatment of choice is:
a. Expectant management
b. Methotrexate
c. Laparoscopic salpingostomy
d. Laparoscopic salpingectomy
Solution. (b) Methotrexate
Ref:Read the text below
Sol:

This patient has an ectopic pregnancy that is amenable to treatment with methotrexate because she has no contraindications
(hCG not high, less than 3-cm adnexal mass, and mild asthma and diabetes mellitus are not contraindications).

Although methotrexate and linear salpingosto-my have comparable rates of tubal patency and fertility, methotrexate is the least
invasive.

Expectant management of a growing ectopic pregnancy is not appropriate given possibility of rupture and hemorrhage, which can
be catastrophic.

Salpingectomy is not preferred to salpingostomy in someone who desires future fertility. Laparotomy is not indicated in this
patient, who is hemodynamically stable.
Your Answer. b
Correct Answer. b

(229).

Periconceptional use of the following agent leads to reduced incidence of neural tube defects:
a. Folic acid
b. Iron
c. Calcium
d. Vitamin A
Solution. (a) Folic acid
Ref:Read the text below
Sol:
Folic acid has been shown to effectively reduce the risk of neural tube defects (NTDs) A daily 4 mg dose is recommended for
patients who have had a previous pregnancy affected by neural tube defects.
It should be started at least 1 month (ideally 3 months) prior to pregnancy and continued through the first 6-12 weeks of
pregnancy.
Your Answer. a
Correct Answer. a

(230).

Minimum level of HCG detected by radiolimmunoassay is:


a. 1.5 3.5IU/ml
b. 0.5 1 IU/ml
c. 0.02 0.5 IU/ml
d. 0.001 IU/ml
Solution. (D) 0.001 IU/ml
Ref:Read the text below
Sol:
Test
Sensitivity
Radioimmunoassay
( subunit)
Immunoradiometric assay
(IRMA) (Serum)

Time taken
0.002
0.05mlU/ml

Positive on
3-4 hours
30 min.
conception

25th day of cycle


8 days after

Your Answer. b
Correct Answer. d

Copyright 2014 Delhi Academy of Medical Sciences, All Rights Reserved.

91/119

(231).

Denominator is
a. Relation of the different parts of the fetus with one another
b. Part of the presentation which oversees the internal OS
c. Boney fixed point on the presenting part
d. Part of the fetus that occupies the lower pole of the uterus
Solution. (c) Boney fixed point on the presenting part
Ref.:Read the text below
Sol :
Denominator is an arbitrary bony fixed point on the presenting part which comes in relation with the various quadrants of the maternal
pelvis.
The following are the denominators of the different presentations.
Occiput in vertex.
Mentum in face.
Frontal eminence in brow.
Sacrum in breech, and
Acromion in shoulder.
Your Answer. c
Correct Answer. c

(232).

Abruptio placenta is characterized by all except :


a. .Tenderness over the uterus
b. Profuse vaginal bleeding
c. Hypotension
d. Absence of fetal heart sound
Solution. (c) Hypotension
Ref.:Dutta Obs. - 257
Sol :
Abruptio placenta
Symptoms :
Most common Vaginal bleeding (It can be profuse).
Severe and constant abdominal pain
Sign:
Pallor
Uterine tenderness
Uterus will be larger.
Difficulty in palpating fetal parts.
Your Answer. d
Correct Answer. c

Copyright 2014 Delhi Academy of Medical Sciences, All Rights Reserved.

92/119

(233).

All of the following are used in the treatment of postpartum hemorrhage except:
a. .Misoprostol
b. Mifepristone
c. Carboprost
d. Methyl ergometrine
Solution. (b) Mifepristone
Ref.:Dutta Obs. - 416
Sol :
Medical Management : Drugs used for management of PPH are :
Oxytocin
Ergometrine or methergine(care should be taken in hypertensive patients).
15 methyl PGF 2 : can be given both I/M and intramurally into the uterine musculature and prepeated after 15 mins, for a maximum
of 3 doses.
Prostaglandins may cause diarrhoea, vomiting, tachycardia and pyrexia. PGF-2 can also produce severe bronchospasm
(Contraindicated in asthmatic patients).
Misoprostol (PGE1)can be inserted rectally or vaginally upto a maximum of 1000ug.)
Your Answer. b
Correct Answer. b

(234).

A case of 35 week pregnancy with hydramnios and marked respiratory distress is best treated by :
a. Intravenous frusemide
b. Saline infusion
c. Amniocentesis
d. Artificial rupture of membranes
Solution. (c) Amniocentesis
Ref.:Dutta Obs 6/e, p 216
Sol :
The patient has marked respiratory distress (i.e. it is a severe polyhydramnios and requires treatment) and gestational age is 35
weeks (i.e., fetal maturity is not yet achieved).
So our aim should be to relieve the distress of patient in hope of continuing the pregnancy till at least 37 weeks.
This can be achieved by : Amniocentesis or Use of Indomethacin
Amniocentesis
The main aim of amniocentesis is to relieve maternal distress with the added advantage that lung maturity of fetus can be predicted by
Lecithin/Sphingomyelin ratio in the expressed fluid.
Slow decompression is done at the rate of 500 ml/hr.
Maximum fluid removed is 1.5 2 litres.
Your Answer. d
Correct Answer. c

Copyright 2014 Delhi Academy of Medical Sciences, All Rights Reserved.

93/119

(235).

Most common type of twin pregnancy is :


a. Vertex + transverse
b. Both vertex
c. Vertex + breech
d. Both breech
Solution. (b) Both vertex
Ref.:Dutta Obs. 6/e, p. 205
Sol :
In twins most common lie of both the fetus at term is longitudinal
Rarest lie is both the twins transverse.
Presentations :
Both vertex (Most common) 60%
Vertex (1st) Breech (IInd) 20%
Breech (1st) Vertex (IInd) 10%
Both Breech 8-10%
Note : Interlocking of twins is a rare complication seen in twins with 1st Breech presentation and IInd vertex presentation.
Your Answer. b
Correct Answer. b

(236).

Placental alkaline phosphataseis secreted by :


a. Endogenous sinus tumour
b. Choriocarcinoma
c. Granulosa cell tumour
d. Dysgerminoma
Solution. (d) Dysgerminoma
Ref.:Read the text below
Sol :
Epithelial non mucinous ovarian ca

CA 125

Mucinous cystadenoma

CEA, CA 19-9

Endogenous sinus tumour antitrypsin

AFP, Alpha 1

Embryonal cell carcinoma

HCG, AFP

Choriocarcinoma

HCG

Dysgerminoma

LDH, Placental alkaline phosphatase

Granulosa cell tumour

Inhibin

Your Answer. a
Correct Answer. d

Copyright 2014 Delhi Academy of Medical Sciences, All Rights Reserved.

94/119

(237).

Indication of amnioinfusion includes all except:


a. Oligohydramnios
b. Suspected renal anomalies
c. To facilitate labour
d. In case of fetal distress
Solution. (c) To facilitate labour
Reference: Dutta Obs. - 614;
Sol :
Amnioinfusion is the technique to increase the intrauterine fluid volume with normal saline (500 ml).
Indications are :

Treatment of variable or prolonged deceleration (i.e. fetal distress).

Prophylaxis for cases of known oligohydramnios as with prolonged rupture of membrane.


In an attempt to dilute or wash out thick meconium.
Besides the above mentioned therapeutic it can be used for diagnosis of : Renal agenesis PROM (Premature rupture of membrane.
Your Answer. a
Correct Answer. c

(238).

To say twin discordance the differences in the two twins should be :


a. 15% with the larger twin as index
b. 15% with the smaller twin as index
c. 25% with the larger twin as index
d. 25% with the smaller twin as index
Solution. (c) 25% with the larger twin as index
Reference: Williams Obs. 22/e, p 932 233
Sol :

Unequal sizes of twin fetuses with a difference of 25% (larger twin being used as the index) is called as Discordant growth.

It is a sing of pathological growth restriction in the smaller fetus.


Your Answer. a
Correct Answer. c

(239).

In superfecundation which of the following is seen:


a. Fertillization of 2 ova released at same time, by sperms released at intercourse on 2 different occasions.
b. Fertilization of 2 ova released at same time by sperms released at single intercourse
c. Both
d. None
Solution. (a) Fertillization of 2 ova released at same time, by sperms released at intercourse on 2 different occasions
Reference: Dutta Obs. 6/e, p 205
Sol :
Superfecundation

It is the fertilization of two different ova released in the same cycle, by separate acts of coitus within a short period of time.
Superfetation

It is the fertilization of two ova released in different menstrual cycles. This is theoretically possible until the decidual space is
obliterated by 12 weeks of pregnancy.
Your Answer. b
Correct Answer. a

Copyright 2014 Delhi Academy of Medical Sciences, All Rights Reserved.

95/119

(240).

Portal hypertension in childrenin India is commonly due to


a. Indian childhood cirrhosis
b. Extrahepatic portal venous obstruction
c. Idiopathic portal hypertension
d. Hepatic out flow tract obstruction
Solution. (b) Extrahepatic portal venous obstruction
Ref.:Read the text below
Sol :
Portal hypertensionis defined by elevation of portal venous pressure to valves above 10-12 mm Hg.
A review of pediatric report suggests that in children extrahepatic portal hypertension (EHPH) is at least 50% more prevalent as a
cause of portal hypertension than intrahepatic portal hypertension.
Extrahepatic portal hyperatension mostly occurs due to venous obstruction.
Bleeding from esophageal varies as a result of portal hypertension is the commonest cause of upper GI bleeding in children.
Your Answer. b
Correct Answer. b

(241).

Regarding Wilsons disease, the true statement is :


a. Hemolytic anemia
b. Autosomal dominant inheritance
c. Decreased urinary copper excretion
d. Normal hepatic copper level.
Solution. (a) Hemolytic anemia
Ref.:Read the text below
Sol :
Hemolytic anemiais a feature of Wilsons disease. Hemolytic anemia occurs because large amounts of copper derived from
hepatocellular necrosis are released into the bloodstream.
Wilsondisease :
An autosomal recessive trait
Caused by mutations in the ATP7B gene in chromosome 13
ATP7B protein deficiencyimpairs biliary copper excretion, resulting in copper toxicity affecting the liver and brain.
Hepatic features: may present as hepatitis, cirrhosis or as hepatic decompensation.
Neurological features.
Triad of dystonia, incoordianation, and tremor
Loss of emotional control (temper tantrums, crying bouts), depression, hyperactivity, or loss of sexual inhibition.
No sensory involvement or muscular weakness
Associated with recurrent abortions, fanconis syndrome, renal tubular acidosis, nephrolithiasis, cholelithiasis.
Eye findings: Sunflower cataract, Kayhser Fleisher ring.
Your Answer. a
Correct Answer. a

(242).

Which of the following is a marker of hypoxic ischaemic encephalopathy ?


a. Serotonin
b. Neuron specific enolase
c. Epinephrine
d. Dopamine
Solution. (b) Neuron specific enolase
Ref.:Read the text below
Sol :
Neuron-specific enolase is a market of the severity and outcome of hypoxic ischemic encephalopathy.
Your Answer. a
Correct Answer. b

Copyright 2014 Delhi Academy of Medical Sciences, All Rights Reserved.

96/119

(243).

The most common cause of pancreatic insufficiency in childhood is:


a. Chronic pancreatitis
b. Congenital lipase deficiency
c. Cystic fibrosis
d. Shwachman-Diamond syndrome
Solution. (c) Cystic fibrosis
Ref: Read the text below
Sol :

In the developed countries, cystic fibrosis is the most common etiology of pancreatic insufficiency.

This is present in more than 90% of children with cystic fibrosis and is usually present at birth.

In developing countries, malnutrition is the most common cause of pancreatic insufficiency.


Your Answer. c
Correct Answer. c

(244).

A 15-month-old child presents with a high fever, and an intense, red rash on the cheeks with circumoral pallor. What is the most
likely etiology of this febrile exanthem?
a. Enterovirus 71
b. Adenovirus
c. Parvovirus B19
d. Rubeola Virus
Solution. (c) Parvovirus B19
Ref: Read the text below
Sol:

The clinical manifestations of human parvovirus B-19 include erythema infectiosum (healthy child), polyarthropathy syndrome
(adults especially women), chronic anemia/pure red cell aplasia (immunocompromised hosts), transient aplastic crisis (sickle cell
patients), and hydrops fetalis/congenital anemia (fetus). Erythema infectiosum is most commonly diagnosed and easily
recognized.

Adistinctive rash featuring a slapped cheek appearance is noted that is often associated with circumoral pallor.

Coxsackievirus A16 and enterovirus 71 are causesof hand-foot-and-mouth syndrome.

Enterovirus 71 is also associated with encephalitis. Adenovirus is not associated with a characteristic exanthem, however it may
cause conjunctivitis.

Rubeola virus is the causativeagent of measles, and generally presents with a triad of cough, coryza, and conjunctivitis, as well as
Koplik spots and rash.
Your Answer. c
Correct Answer. c

(245).

A term female is born by spontaneous vaginal delivery to a primiparous woman who received two doses of meperidine 30 min
and 2 hr prior to an abrupt delivery. The baby is apneic and limp. The most important, immediate management is to:
a. Administer naloxone in the umbilical vein
b. Perform bag-mask ventilation
c. Administer naloxone in the endotracheal tube
d. Begin chest compressions
Solution. (b) Perform bag-mask ventilation
Ref: Read the text below
Sol:

Apnea from any cause is treated with securing a patent airway and instituting ventilation.

If bag mask ventilation is ineffective, endotracheal intubation should be performed.


Naloxone (Narcan) can be given onlyafter the baby is ventilated.
Your Answer. c
Correct Answer. b

Copyright 2014 Delhi Academy of Medical Sciences, All Rights Reserved.

97/119

(246).

Proteinuria >1-2 g/24 his known as?


a. Transient Proteinuria
b. Sustained proteinuria
c. Benign Proteinuria
d. Malignant Proteinuria
Solution. (b) Sustained proteinuria
Ref.:Harrisons - Table 283-1
Sol :
Sustained proteinuria > 1.2/ g/24h is also commonly associated with glomerular disease. Patients often will not know they have
proteinuria unless they become edematous or notice foaming urine on voiding.
This latter class of proteinuria is non sustained, generally <1 g/24 h, and is sometimes called functional or transient proteinuria.
Fever, exercise, obesity. Sleep apnea, emotional stress, and congestive heart failure can explain transient proteinuria.
Your Answer. c
Correct Answer. b

(247).

Which sinus will be absent in a newborn?


a. Frontal sinus
b. Maxillary
c. Anterior ethmoidal
d. Posterior ethmoidal
Solution. (a) Frontal sinus
Ref.:Nelsons - 48
Sol :
Both the ethmoidal and maxillary sinuses are present at birth, but only the ethmoidal sinuses are pneumatized.
The maxillary sinuses are not pneumatized until 4 year of age.
The sphenoidal sinuses are present by 5 years of age, whereas the frontal sinuses begin development at age 7-8 year and are not
completely developed until adolescence.
Your Answer. a
Correct Answer. a

(248).

A 5-yr-old girl is evaluated forsevere mental retardation, microcephaly, hand-wringing movements, poor growth and weight
gain, and seizures. Genetic analysis indicates a mutation in a transcription factor called MeCP2 that normally functions to
silence transcription of numerous genes. This finding indicates that she has which of the following disorders?
a. Juvenile Huntington disease
b. Metachromatic leukodystrophy
c. Rett syndrome
d. Adrenoleukodystrophy
Solution. (c) Rett syndrome
Ref: Read the text below
Sol :

Rett syndrome is a neurodevelopmental disorderof the grey matter of the brain that almost exclusively affects females.

The clinical features include small hands and feet and a deceleration of the rate of head growth (including microcephaly in
some).

Repetitive hand movements, such as wringing and/or repeatedly putting hands into the mouth, are also noted. People with Rett
syndrome are prone to gastrointestinal disorders and up to 80% have seizures.

They typically have no verbal skills, and about 50% ofindividuals affected are not ambulatory. Scoliosis, growth failure, and
constipation are very common and can be problematic.
Your Answer. c
Correct Answer. c

Copyright 2014 Delhi Academy of Medical Sciences, All Rights Reserved.

98/119

(249).

Steroid Resistant Nephrotic syndrome is associated with which of the following gene:
a. NPHSI
b. PAX
c. HOX II
d. NPHS2
Solution. (d) NPHS2
Ref:Read the text below
Sol :

Your Answer. b
Correct Answer. d

(250).

A 5-mo-old infant with gross motor delay, hypotonia, and cardiomegaly has a creatine kinase level of 860 IU/L. Of the
following, which is the most likely diagnosis?
a. Spinal muscular atrophy
b. Hypothyroidism
c. Prader-Willi syndrome
d. Pompe disease
Solution. (d) Pompe disease
Ref:Read the text below
Sol :

Pompe disease, also referred to as GSD type II oracid maltase deficiency, is caused by a deficiency of acid -1,4-glucosidase
(acid maltase), an enzyme responsible for the degradation of glycogen in lysosomes.

This enzyme defect results in lysosomal glycogenaccumulation in multiple tissues and cell types, with cardiac, skeletal, and
smooth muscle cells being the most seriously affected. The disease is characterized by accumulation of glycogen in lysosomes, as
opposed to its accumulation in cytoplasm in the other glycogenoses.

Pompe disease is an autosomal recessive disorder. Clinical trials of enzyme replacement therapy (ERT) have been
promising and ERT withmyozyme is available for treatment of Pompe disease. Recombinant acid -glucosidase is capable of improving
cardiac and skeletal muscle functions
Your Answer. d
Correct Answer. d

Copyright 2014 Delhi Academy of Medical Sciences, All Rights Reserved.

99/119

(251).

Pulmonary plethorais not seen in :


a. VSD
b. TOF
c. Truncus arteriosus
d. TAPVC
Solution. (b) TOF
Ref.:Nelsons - 48
Sol :
Cyanotic heart disease with increased pulmonary blood flow : (a) Double outlet right ventricle (DORV) with sub pulmonary VSD
(Taussig Bing anomal)
Total anomalous pulmonary venous return (TAPVR)
Transposition of the great arteries (TGA)
Single ventricle (without PS)
Persistent truncus arteriosus type I
Cyantic heart disease with decreased pulmonary blood flow :
Tetralogy of Fallot (TOF)
Ebsteins anomaly
Pulmonary atresia
Tricuspid atresia
TGA and PS
Persistent truncus arteriosus (Type II or III)
Single ventricle and PS
Your Answer. b
Correct Answer. b

(252).

In a neonate with the skin of soles and palms showing yellow discoloration, the approximate level of serum bilirubin (mg/dl)
will be :
a. 4-8 mg/dL
b. 8-16 mg/dl
c. 11-18 mg/dL
d. > 15 mg/dL
Solution. (d) > 15 mg/dL
Ref.:Read the text below
Sol :
Area Yellow discoloration of body in jaundice Range of serum bilirubin (mg%)
Head and neck

48

Upper trunk

5- 12

Lower trunk and thigh

8 16

Arms and lower limbs

11- 18

Palms and soles

More than 15

Your Answer. d
Correct Answer. d

Copyright 2014 Delhi Academy of Medical Sciences, All Rights Reserved.

100/119

(253).

Which rule is for estimation of child age from the crown rump length?
a. Haase rule
b. Mc Naghtens rule
c. Locards principle
d. Durhams rule
Solution. (a) Haase rule
Ref.:Read the text below
Sol :
Haases rule is for calculating the age of fetus from its length in cm.
1st 5 lunar months = square of length in cm.
2nd 5 lunar months = length in cm/5.
Your Answer. d
Correct Answer. a

(254).

A 3 year old child presents with recurrent stridor, what is the most probable diagnosis?
a. Laryngomalacia
b. Laryngotracheobronchitis
c. Foreign body aspiration
d. Laryngeal edema
Solution. : (b) Laryngotracheobronchitis
Ref.:Read the text below
Sol :
Most common cause of stridoroverall is laryngotracheobronchitis
Most common cause of stridor in children is laryngotracheobronchitis
Most common of congenital stridor is laryngomalacia
Most common cause of stridor in newborn is laryngomalacia
Your Answer. b
Correct Answer. b

(255).

A 45 day old infant developed icterus and two days later symptoms and signs of acute liver failure appeared. Child was found to
be positive for HbsAg. The mother was also HBsAg carrier. The mothers hepatitis B serological profile is likely to be
a. HBsAg positive only
b. HBsAg and HBeAg positivity
c. HBsAg and anti HBe antibody positivity.
d. Mother infected with mutant HBV
Solution. (b) HBsAg and HBeAg positivity
Ref.:Read the text below
Sol :
Vertical transmissiondepends upon the maternal HBeAg status.
HBsAg carrier mother who are HBeAg positive almost invariably (>90%) transmit hepatitis B, infection to their off springs where as
HBsAg carrier mother with anti HBe rarely (10-15%) infect their offsprings.
Your Answer. b
Correct Answer. b

Copyright 2014 Delhi Academy of Medical Sciences, All Rights Reserved.

101/119

(256).

All are features of neonatal necrotizing enterocolitis except


a. Abdominal distention
b. Increased bowel sounds
c. Metabolic acidosis
d. Pneumoperitoneium
Solution. (b) Increased bowel sounds
Ref.:Read the text below
Sol :
Clinical manifestations of necrotizing enterocolitis
May be described in 3 stages
Stage 1 Suspect Unstable temperature, apnoea, bradycardia, lethargy, abdominal distension, vomiting.
Stage 2 Definite Above signs associated with diminished bowel sounds with or without tenderness. In more severe cases metabolic
acidosis and mild thrombocytopenia.
Stage 3- Advanced Above signs associated with low BP, bradycardia apnoea, acidosis, disseminated intravascular coagulation, and
even anuria. There are sign of peritoritis with abdominal wall edema.
X-ray Finding
Stage 2 Pneumatosis intestinalis and dilatation of intestine.
State 3 Pneumoperitonium.
Your Answer. b
Correct Answer. b

(257).

The age of rumination in children is?


a. 3 to 12 months
b. 1 to 4 years
c. 3 to 5 years
d. 4 to 10 years
Solution. (a) 3 to 12 months
Ref.:Nelson paediatrics 18th/pg. 113
Sol :
Rumination disorder :
The hallmark of rumination disorder is weight loss or failure to gain at the xpected level because of repeated regurgitation of food
without nausea or associated gas trointestinal illness.
This rare disorder occurs more commonly in males and usually appears at 3-14 mo of age. It is potentially fatal; some reports indicated
that up to 1/4th of affected children die. There are psychogenic and self-stimulating ruminators.
Your Answer. b
Correct Answer. a

(258).

The milestone of object constancy is achieved at ?


a. 6 months
b. 9 months
c. 12 months
d. 15 months
Solution. (b) 9 months
Ref.:Nelson - 47
Sol :
A major milestone is the achievement at about 9 mo of object permanence (constancy), the understanding that objects continue
to exlst, even when not seen.
At 4-7 mo of age, infants look down for a yarn ball that has been dropped, but quickly give up if it is not seen. With object constancy,
infants persist in searching, finding objects hidden under a cloth or behind the examiners back, Peek-a-boo brings unlimited pleasure as
the child magically brings back the other player. Events seem to occur as a result of the childs own activities.
Your Answer. d
Correct Answer. b

Copyright 2014 Delhi Academy of Medical Sciences, All Rights Reserved.

102/119

(259).

All of the following are essential features of attention deficit hyperactivity disease (ADHD) except :
a. Lack of concentration
b. Impulsivity
c. Mental retardation
d. Hyperactivity
Solution. (c) Mental retardation
Ref.:Read the text below
Sol :
Inattention, hyperactivity, disruptive behavior and impulsivity are common in ADHD.

Academic difficulties are also frequent.

The symptoms are especially difficult to define because it is hard to draw a line at where normal levels of inattention, hyperactivity,
and impulsivity end and clinically significant levels requiring intervention begin.
To be diagnosed with ADHD, symptoms must be observed in two different settings for six months or more and to a degree that
is greater than other children of the same age
Your Answer. c
Correct Answer. c

(260).

Pseudo isomorphic response is seen in


a. Vitiligo
b. Psoriasis
c. Lichen planus
d. Warts
Solution. (d) Warts
Ref:Read the text below
Sol :

True isomorphism (Koebners) is seen in vitiligo, psoriasis and lichen planus while pseudo isomorphism is seen in
autoinoculation in warts and molluscum contagiosum.
Your Answer. d
Correct Answer. d

(261).

Choose the correct sentence


a. Crystalline miliaria is due to obstruction of most superficial part of eccrine sweat duct
b. Fordyce disease is associated with apocrine glands
c. Palms and soles have eccrine glands solely controlled by thermal stimuli
d. Sebaceous glands are mostly free but sometimes may be associated with hair follicles
Solution. (a) Crystalline miliaria is due to obstruction of most superficial part of eccrine
sweat duct
Ref:Read the text below
Sol :

Blockage of eccrine glandsproduces miliaria.

Most superficial type produces crystalline miliaria while mid level block produces miliaria rubra (prickly heat) and deep blocks
produce miliaria profunda.

Sebaceous glandsare always associated with hair follicles except free/ectopic glands like Fordyce disease or granules.
Eccrine sweatglands of palms and soles differ from other glands by responding to emotional stimuli too.
Your Answer. d
Correct Answer. a

Copyright 2014 Delhi Academy of Medical Sciences, All Rights Reserved.

103/119

(262).

Fish handlers disease is


a. Swimming pool granuloma
b. Fish tank granuloma
c. Erysipeloid
d. Sporotrichosis
Solution. (c) Erysipeloid
Ref:Read the text below
Sol :

Fish handlers diseaseis erysipeloid caused by Erysipelothrix spp.

Fish tank granuloma and swimming pool granuloma are cutaneous atypical mycobacteriosis caused by M. marinum.

Sporotrichosis is known as rose gardeners disease.


Your Answer. c
Correct Answer. c

(263).

Which is a bacterial infection


a. Pyoderma gangrenosum
b. Impetigo herpetiformis
c. Pitted keratolysis
d. Mycosis fungoides
Solution. (c) Pitted keratolysis
Ref:Read the text below
Sol :

Pitted keratolysis is a coryneform infection.

Mycosis fungoides is a cancer.

Impetigo herpetiformis is pustular psoriasis of pregnancy and pyoderma gangrenosum is a neutrophilic ulceration associated with
ulcerative colitis etc.

Pitted keratolysis presents with pits or holes on the soles.


Pitted keratolysis caused by Coryneforms like Kytococci (micrococci), corynebacterium sedantarius etc
Your Answer. a
Correct Answer. c

(264).

Granular degeneration is seen in all except


a. Lichen planus
b. Verrucous epidermal nevus
c. Bullous ichthyosiform erythroderma
d. Vorners palmoplantar keratoderma
Solution. (a) Lichen planus
Ref:Read the text below
Sol :

Granular degenerationis vacuolization of stratum granulosum.

It is seen in half of cases of verrucous epidermal nevus.

It is also seen in Vorners palmoplantar keratoderma and bullous ichthyosiform erythroderma.

Lichen planushas vacuolization of basal layer cells and hyper granulosis.


Your Answer. c
Correct Answer. a

Copyright 2014 Delhi Academy of Medical Sciences, All Rights Reserved.

104/119

(265).

Systemic steroids are preferred treatment for all except


a. Impetigo herpetiformis
b. Pemphigus vulgaris
c. Type I lepra reaction
d. Widespread plaque psoriasis
Solution. (d) Widespread plaque psoriasis
Ref:Read the text below
Sol :

Plaque psoriasisshould be treated with topical steroids if limited, phototherapy if moderate and methotrexate or retinoids if
extensive.

Pustular psoriasisif generalized are treated with retinoids or methotrexate.

But in pregnancy if pustular psoriasis is severe (impetigo herpetiformis) it is treated with oral steroids.
Pemphigus vulgaris, type I lepra reaction are also treated with oral steroids
Your Answer. d
Correct Answer. d

(266).

Most common cause of common warts?


a. HPV 1
b. HPV 2
c. HPV 3
d. HPV 4
Solution. (b) HPV 2
Ref:Read the text below
Sol :

Common warts are most commonly cuased by HPV2.


Your Answer. c
Correct Answer. b

(267).

Treatment of reactional neuritis in the acute phase of Type I lepra reaction involves:
a. Active exercises
b. Passive exercises
c. Rest of the affected limb in neutral position
d. Surgical exploration
Solution. (c) Rest of the affected limb in neutral position
Ref:Read the text below
Sol :

Lepra reactionsespecially type I reactions are complicated by neuritis.

Acute neuritis requires, resting of the affected limb, oral steroids and continued MDT.
Your Answer. b
Correct Answer. c

Copyright 2014 Delhi Academy of Medical Sciences, All Rights Reserved.

105/119

(268).

Scombroid fish food poisoning caused by proteus spp. mainly producing?


a. Serotonin
b. Bradykinin
c. Catecholamines
d. Histamine
Solution. (d) Histamine.
Ref:Read the text below
Sol :

Food-associated flushingmay be caused by capsaicin (red pepper), sodium nitrate, or alcohol.

Alcohol may produce flushingin patients using topical calcineurin inhibitors.

Sulfites are found in wine, dried fruit, prepared foods, and fresh grapes and potatoes.

Ciguatera or scombroid fish poisoningis a form of histamine-related food poisoning, caused by histamine within the flesh of the
fish
Your Answer. a
Correct Answer. d

(269).

Which of the following is not associated withatopic dermatitis


a. Posterior subcapsular cataract
b. Anterior subcapsular cataract
c. Keratoconus
d. Hertoghes sign
Solution. (a) Posterior subcapsular cataract
Ref:Read the text below
Sol :

Many eye/periorbital changes are associated with atopic dermatitis including:

Anterior subcapsular cataract

Keratoconus

Periorbital pallor (headlight sign)

Periorbital darkness (allergic shiners)

Lateral madarosis (Hertoghes sign

Accessory and deep infraorbital lines (Dennies lines/ Dennie Morgan folds).
Your Answer. a
Correct Answer. a

(270).

Opioid effects on respiratory rate and rhythminclude all the following except
a. Decreased respiratory rate
b. Irregular respiratory rate
c. Cheyne-Stokes breathing patterns
d. Increased respiratory pauses
Solution. (c) Cheyne-Stokes breathing patterms
Ref:Read the text below.
Sol:
A Cheyne-Stokes respiratory pattern consists of periods of increasing hyperventilation alternating with apnea. The rate of
ventilation increases markedly and then declines until apnea occurs.
This pattern repeats in a rhythmic fashion.
Cheyne-Stokes respiration usually occurs after a cerebral anoxic event and reflects brain damage. Opioids do not cause this respiratory
pattern.
Your Answer. d
Correct Answer. c

Copyright 2014 Delhi Academy of Medical Sciences, All Rights Reserved.

106/119

(271).

CT scan is least accurate for diagnosis of:


a. 1 cm size aneurysm in an artery
b. 1 cm size lymph node in thyroid carcinoma
c. 1 cm size mass in tail of pancreas
d. 1 cm size gall stone
Solution. (d) 1 cm size gall stone
Ref: Read the text below
Sol:

Gall stones arepoorly evaluated on CT.


This is in contrast to urinary stones: for which NCCT abdomen is the gold standard.
Your Answer. d
Correct Answer. d

(272).

Investigation of choice for vascular ring around airway


a. PET
b. CT
c. MRI
d. CATHETER DIRECTED ANGIOGRAPHY
Solution. (b) CT
Ref: Read the text below
Sol:

Patients, especially infants or young children, with recurrent respiratory symptoms such as chronic cough, stridor and wheeze,
should be examined for the possible presence of congenital vascular rings.

Contrast-enhanced CT can clearly show the anatomy of vascular rings. Contrast-enhanced thoracic CT with 3D reconstruction may
allow accurate diagnosis and clearly show compression of the tracheoesophageal tract.

For several decades, the diagnosis of vascularrings has mainly relied on chest X-rays and barium swallow. Recently, it has been
shown that CT had replaced barium swallow as the diagnostic procedure of choice for vascular ring evaluation.

Although angiography has been used to further delineate arch anatomy, it is now rarely performed due to the availability of
noninvasive techniques and due to problems like overlapping of structures.
Your Answer. b
Correct Answer. b

(273).

Investigation of choice for meningeal carcinomatosis in CNS


a. Gd enhanced MRI
b. PET
c. SPECT
d. CT scan
Solution. (a) Gd enhanced MRI
Ref: Read the text below
Sol:

Leptomeningeal carcinomatosis refers to diffuseseeding of the leptomeninges by tumor metastases.

LC occurs in an estimated 20% of patients diagnosed with cancer and is most commonly found in breast carcinoma, lung carcinoma,
and melanoma in adults and hematogenous malignancies and primitive neuroectodermal tumor (PNET) in children.

Contrast-enhanced MRI of the brain and spine is the imaging modalityof choice because of its safety, excellent contrast
resolution, and multiplanar abilities.
Your Answer. a
Correct Answer. a

Copyright 2014 Delhi Academy of Medical Sciences, All Rights Reserved.

107/119

(274).

Distal acoustic enhancement on ultrasound is seen in :


a. Air containing lesion
b. Fluid containing area
c. Calculi
d. Bone
Solution. (b) Fluid containing area
Ref: Read the text below
Sol:
1. Acoustic shadowing behind calcified objects- In bones calcification, calculi

2.

Acoustic enhancement- Acoustic enhancement of fluid filled spaces, Cysts,bladder.

Your Answer. b
Correct Answer. b

(275).

A 37-year-old woman complains of headache and blurry vision that have been present for a year and are slowly getting worse.
As part of her evaluation an MRI is obtained and shown below:
What is the most likely diagnosis in this patient?
a. Brain abscess
b. Glioblastoma
c. Low-grade astrocytoma
d. Meningioma
Solution. (d) Meningioma
Ref: Read the text below
Sol:

This figure illustrates a mass attached to the meninges with a dural tail. Other dural tumors may appear this way, but of the
options listed, the meningioma is by far the most likely to appear this way.

Meningiomas derive from the cells that give rise to the arachnoid granulations. They are usually benign and attached to the dura.
They rarely invade the brain.

They are more frequent in women than men and have a peak incidence in middle age. Total surgical resection of a
meningioma is curative. Low-gradeastrocytoma and high-grade astrocytoma (glioblastoma) often infiltrate into adjacent brain and
rarely have the clear margins seen in this figure.

Oligodendroma comprise ~15% of all gliomas and show calcificationin roughly 30% of cases. They have a more benign course
and are more responsive than other gliomas to cytotoxic therapy.

For low-grade oligodendromas, the median survival is 78 years. Brain abscess will have distinctive ring-enhancing features
with a capsule, often have mass effect, and will have evidence of inflammation on MRI scanning.
Your Answer. d
Correct Answer. d

Copyright 2014 Delhi Academy of Medical Sciences, All Rights Reserved.

108/119

(276).

Missed IUCDcan be best recognized by


a. Standing X-ray abdomen
b. Ultrasound
c. Barium meal study
d. CT scan
Solution. (b) Ultrasound
Reference: (Ref. Dutta, Gynacology, 6th ed., 541)
Sol:

Ultrasonography can be detect the IUCD eitherwithin the uterine cavity or in the peritoneal cavity (if perforated).
It is preferred over radiography.
Your Answer. b
Correct Answer. b

(277).

Best imaging modality in patientswith breast implants is


a. MRI scan
b. CT scan
c. Mammography
d. Radionuclide scan
Solution. (a) MRI scan
Reference: (Ref. Diagnostic Radiology, MSK & Breast imaging 2nd ed. 548)
Sol:

MRI is preferable modality in assessment of breast with implants.

Mammographic evaluation of the augmented breasts (i.e. breasts with silicon implant) is challenging, since breast implants
obscure significant part of breast tissue & also reduce the effect of compression.

Associated scarring can also make mammographic interpretation difficult.


MRI has evolved as the modality of choice for diagnosing implant complication as well as detection of primary or recurrent breast
cancer in these patients.
Your Answer. a
Correct Answer. a

(278).

Radiation exposureduring infancy has been linked to which one of the following carcinomas?
a. Breast
b. Melanoma
c. Thyroid
d. Lung
Solution. (c) Thyroid
Reference: (Ref. Radioactive Isotopes in Medicine and Biology, 2nd ed., 135)
Sol:

Secondary cancersare the cancers which occur due to exposure to radiation. Ca thyroid (papillary type) is a classical example.

A number of cases of cancer of thyroid have been reported in children who had some years previously received X-ray treatment for
disorders like thymus enlargement, infected tonsils, etc. these treatments were given in early infancy.
However, it has now been practically discontinued. However, radiation exposure during childhood, especially infancy, for some reason or
other has been linked to further risk of development of thyroid cancer.
Your Answer. c
Correct Answer. c

Copyright 2014 Delhi Academy of Medical Sciences, All Rights Reserved.

109/119

(279).

Restricted Diffusion onDWI in MRI is seen in all the followingEXCEPT


a. Brain Abscess
b. Epidermoid Cyst
c. Acute Infarct
d. Arachnoid Cyst
Solution. (d) Arachnoid Cyst
Ref: Read the text below
Sol:

The classic arachnoid cyst has no identifiable internal architecture and does not enhance. The cyst typically has the same
signal intensity as CSF at all sequences. The most difficult lesion to distinguish from the arachnoid cyst is an epidermoid cyst. Arachnoid
cysts typically suppress completely on FLAIR images and do not restrict on diffusion-weighted images. Arachnoid cysts displace
adjacent arteries and cranial nerves rather than engulf them, as epidermoid cysts often do.
Proton density-weighted and then fluid-attenuated inversionrecovery (FLAIR) images were first used to differentiate epidermoids
from arachnoid cysts. These sequences demonstrate epidermoids as being hyperintense relative to CSF. Diffusion-weighted imaging can
now be used to differentiate these entities, because epidermoids have markedly restricted diffusion and, therefore, high signal intensity
on the diffusion-weighted trace images. The free water in arachnoid cysts has low signal intensity.
Your Answer. d
Correct Answer. d

(280).

Initial investigation of choice in post cholecystectomy biliary stricture:


a. USG ABDOMEN
b. ERCP
c. CT
d. MRCP
Solution. (d) MRCP
Ref: Read the text below
Sol:

Initial investigation on post cholecystectomy Jaundice is USG

Initial investigation of choice in post cholecystectomy biliary stricture:MRCP

Best investigation of choice in post cholecystectomy biliary stricture: ERCP


Your Answer. b
Correct Answer. d

(281).

Who is considered "Father of Modern Psychiatry"?


a. S. Freud
b. J. Reil
c. E. Erikson
d. P. Pinnel
Solution. (d) P. Pinnel
Ref:Read the text below
Sol :

He is father of modern psychiatry.

He was first to unchain mental patients, prior to him, psychiatric patients were like animals kept caged in a zoo.
Your Answer. a
Correct Answer. d

Copyright 2014 Delhi Academy of Medical Sciences, All Rights Reserved.

110/119

(282).

Number of chapters in ICD-10?


a. 19
b. 20
c. 21
d. 22
Solution. (c) 21
Ref:Read the text below
Sol :

ICD-10 having 21 chapters was endorsed by the Forty-third World Health Assembly in May 1990 and came into use in WHO Member
States as from 1994.

The 11th revision of the classification has already started and will continue until 2017.
Your Answer. c
Correct Answer. c

(283).

Lactate provocationtest is used in


a. Panic disorder
b. Generalised anxiety disorder
c. Phobias
d. Somatoform disorders
Solution. (a) Panic disorder
Ref:Read the text below
Sol :
This is an old test, not used these days, but these types of things are expected in a multiple session paper, where they are always
looking for new one liners, you have never heard before exams, to irritate you!!
Kaplan pg 261:Up to 72 percent of patients with panic disorder have a panic attack when administered IV injection of sodium lactate.
Therefore, lactate provocation is used to confirm a diagnosis of panic disorder. Lactate provocation has also been used to trigger
flashbacks in patients with posttraumatic stress disorder.
Your Answer. a
Correct Answer. a

Copyright 2014 Delhi Academy of Medical Sciences, All Rights Reserved.

111/119

(284).

An adoloscent whose chronological age was 18y, mental age was corresponding to 12y, ; according to binet formula for I.Q.
calculation, he should be (as per DSM-IV TR grading)
a. Mild mental retarded
b. Moderate mental retarded
c. Borderline intelligence
d. Dull Normal intelligence
Solution. (d) Dull Normal intelligence
Ref:Read the text below
Sol :
There is a catch here. You all know how to calculate I.Q.
But answer is not 66.6 I.Q. (Mild)
Pg 178 kaplan states : (Because it is impossible to measure age-associated changes in intellectual power after the age of 15 with
available intelligence tests, the highest divisor in the IQ formula is 15.)
So, for this case
I.Q. is 12/15 * 100 = 80.
Remember, DSM IV grading:
Profound mental retard(MR)

Below 20 or 25

Severe MR

20-25 to 35-40

Moderate MR

35-40 to 50-55

Mild MR

50-55 to about 70

Borderline

70-79

Dull normal

80 to 90

Normal

90 to 110

Bright normal

110 to 120

Superior

120 to 130

Very superior

130 and above

Your Answer. b
Correct Answer. d

(285).

8 yr old student c/o no interest in studies 4 last 6 months , has frequent quarrels with parents and has frequent headaches. Most
appropriate clinical approach is:
a. Leave as normal adolescent problem
b. Rule out depression
c. Rule out migraine
d. Rule out oppositional defiant disorder
Solution. (b) Rule out depression
Ref:Read the text below
Sol :

Irritability & somatic complaints (headache) along with school refusal (decreased interest in studies) are very commonly seen
in childhood depression...

Hence we need to first rule out depression.


Your Answer. d
Correct Answer. b

Copyright 2014 Delhi Academy of Medical Sciences, All Rights Reserved.

112/119

(286).

Anorexia nervosa can be differentiated from bullimia nervosa by:


a. Intense fear of weight gain
b. Disturbance of body image
c. Adolescent age
d. Peculiar pattern of food handling
Solution. (d) Peculiar pattern of food handling
Ref:Read the text below
Sol :

Patients with anorexia nervosa exhibit peculiar behavior about food. They hide food all over the house and frequently carry large
quantities of candies in their pockets and purses.

While eating meals, they try to dispose of food in their napkins or hide it in their pockets.

They cut their meat into very small pieces and spend a great deal of time rearranging thepieces on their plates.

If the patients are confronted with their peculiar behavior, they often deny that their behavior is unusual or flatly refuse to
discuss it. (Kaplan pg 729)
Your Answer. d
Correct Answer. d

(287).

The following differentiates "binge eating disorder" from bullimia nervosa?


a. Recurrent episodes of binge eating
b. A sense of lack of control over eating during the episode
c. Marked distress regarding binge eating
d. Not associated with use of inappropriate compensatory behaviour
Solution. (d) Not associated with use of inappropriate compensatory behaviour
Ref: Read the text below
Sol :
Rest all features are similar in both these disorders.
Table 23.2-3 DSM-IV-TR Research Criteria for Binge-Eating Disorder
A. Recurrent episodes of binge eating. An episode of binge eating is characterized by both of the following:
1. eating, in a discrete period of time (e.g., within any 2-hour period), an amount of food that is definitely larger than what most people
would eat in a similar period of time under similar circumstances
2. a sense of lack of control over eating during the episode (e.g., a feeling that one cannot stop eating or control what or how much one
is eating)
B. The binge-eating episodes are associated with three (or more) of the following:
1. eating much more rapidly than normal
2. eating until feeling uncomfortably full
3. eating large amounts of food when not feeling physically hungry
4. eating alone because of being embarrassed by how much one is eating
5. feeling disgusted with oneself, depressed, or very guilty after overeating
C. Marked distress regarding binge eating is present.
D. The binge eating occurs, on average, at least 2 days a week for 6 months.
E. The binge eating is not associated with the regular use of inappropriate compensatory behaviors (e.g., purging, fasting,
excessive exercise) and does not occur exclusively during the course of anorexia nervosa or bulimia nervosa.
Your Answer. d
Correct Answer. d

Copyright 2014 Delhi Academy of Medical Sciences, All Rights Reserved.

113/119

(288).

All of the following terminologies have been replaced by alternative names in dsm-5 except?
a. Dysthymia
b. Somatoform Disorder
c. Dementia
d. Trichotillomania
Solution. (d) Trichotillomania
Ref:Read the text below
Sol :

Dysthymia chronic depression

Dementia neurocognitive disorder

Somatoform- somatic symptom disorder

(refer dams psychiatry appendix for more changes in dsm-5)


Your Answer. c
Correct Answer. d

(289).

The following person shown in the figure, can maintain this posture for a long period of time. What is the most effective first line
treatment for this patient.

a. Intravenous lorazepam
b. Injectable first generation antipsychotics
c. Oral second generation antipsychotics
d. Electroconvulsive therapy
Solution. (a) Intravenous lorazepam
Ref:Read the text below
Sol :
Sign shown in pic is waxy flexibility; seen in catatonia; first line is iv lorazepam; most effective is ECT.
Your Answer. b
Correct Answer. a

(290).

All are true regarding epidemiology of mood disorder except:


a. The mean age of onset of major depressive disorder is 40 years
b. BPAD-1 is more prevalent in women than men
c. Rapid cyclers are more common in women than men
d. Depression is more common in rural areas
Solution. (b) BPAD-1 is more prevalent in women than men
Ref:Read the text below
Sol :

Bipolar I disorder has an equal prevalence among men and women

Rest all statements are true regarding epidemiology of mood disorders.


Your Answer. a
Correct Answer. d

Copyright 2014 Delhi Academy of Medical Sciences, All Rights Reserved.

114/119

(291).

All are true about "recurrent depression" except:


a. To call depression as "recurrent", minimum no of episodes required are 3
b. 2 episodes of depression must be separated by an intervening normal period of at least 2 months
c. Mood stablizers proven to be very effective in recurrent depression is lamotrigine
d. Above 3 statements are all true and none is false
Solution. (a) To call depression as "recurrent", minimum no of episodes required are 3
Ref:Read the text below
Sol :

For calling depression as "recurrent", minimum no of episodes required are 2 and these 2 episodes of depression must be
separated by an intervening normal period of at least 2 months.
Your Answer. c
Correct Answer. a

(292).

A feature of Hansens Disease :


a. Extension of both MP and IP joints
b. Extension of MP and flexion of JP joints
c. Extension of IP and flexion of MP joins
d. Extension of wrist and flexion of MP joints
Solution. (b) Extension of MP and flexion of JP joints
Ref.:Read the text below
Sol :
Claw Hand is a characteristic hand deformity of Hansens which involves hyperextension of MP joints and flexion of IP joints.
Wrist Drop due to radial nerve palsy occurs infrequently.
Your Answer. b
Correct Answer. b

(293).

Cause of Medial leminiscus injury is


a. Adduction and external rotation with flexion of knee
b. Rotation on extended knee
c. Abduction and external rotation with flexion of knee
d. Abduction and internal rotation with flexion of knee
Solution. (a) Adduction and external rotation with flexion of knee
Ref.:Read the text below
Sol :
Mechanism of injury to the Meniscus
Abduction external rotationviolence, on a flexed weight bearing knee, causes a tear in the medial meniscus.
During football, it occurs when the player, standing on one leg which is slightly flexed at the knee, turns to tackle the ball with the
other leg.
The lateral meniscus is damaged by the opposite violence i.e. internal rotation and abduction violence of the tibia on a semi
flexed weight bearing knee.
Your Answer. c
Correct Answer. a

Copyright 2014 Delhi Academy of Medical Sciences, All Rights Reserved.

115/119

(294).

Not a complication of supracondylar fracture:


a. Median nerve injury
b. Non union
c. Cubitus varus
d. Volkmans ischemic contracture
Solution. (b) Non union
Ref.:Read the text below
Sol :
Supracondylar Humerus Fracture
Supracondylar humerus fracturesare the most common elbow fractures in children, accounting for 60-80% of pediatric elbow
fractures.
These injuries are associated with a high rate of complications and can be challenging to manage.
Your Answer. b
Correct Answer. b

(295).

Schmorls node signifies :


a. Trauma
b. Hyperparathyroidism
c. Osteomalacia
d. All the above
Solution. (d) All the above
Ref.:Read the text below
Sol :
Schmorls nodes are defined as herniation of the intervertebral disc through the vertebral end plate.
A German pathologist, Christian G. Schmort in 1927, first described them.
These lesions are believed to be associated with trauma, especially in the thoracic and lumbar vertebrae.
Schmorls nodesmay be caused by numerous factors; trauma; hyperparathyroidism; osteoporosis; scheuermanns disease;
osteomalacia; infectins and neoplasm. Trauma in adolescent athletes may be responsible for symptomatic schmorls nodes.
It is generally agreed that trauma likely precipitates the actual formation of the schmorls node, but a possible congenital origin of
a vertebral end-plate defect has also been suggested.
A dorso-lumbar kyphosis is a distinct clinical entity and may be associated with a Schmorls nnode and should be considered as a
possible etiology for low back pain in the active adolescent.
Your Answer. d
Correct Answer. d

(296).

During callus formation, cell proliferation and differentiation begin to produce:


a. Osteoblasts and chondroblasts in the granulation tissue.
b. Osteoclasts in the granulation tissue.
c. Osteocalcin
d. Osteolaminin
Solution. (a) Osteoblasts and chondroblasts in the granulation tissue.
Ref.:Read the text below
Sol :
During callus formation,cell proliferation and differentiation begin to produce osteoblasts and chondroblasts in the granulation
tissue.
The osteoblasts and chondroblasts synthesize the extracellular orgtanic matrices of woven bone and cartilage respectively, and them
the newly formed bone is mineralized. This stage requires 4-16 weeks.
Your Answer. a
Correct Answer. a

Copyright 2014 Delhi Academy of Medical Sciences, All Rights Reserved.

116/119

(297).

True about the Muller AO comprehensive classification of Fractures is all except :


a. A multifragmentary fracture is one that has several breaks in the bone, creating more than 3 fragments.
b. Simple fractures are spiral, oblique, or transverse
c. The complex multifragmentary fracture is a segmental fracture or one in which there is no contact between the proximal and distal
fragments without the bone shortening.
d. Wedge fractures are either spiral (low energy) or bending (high energy) and allow the proximal and distal fracture fragments to
contact each other.
Solution. (a) A multifragmentary fracture is one that has several breaks in the bone, creating more than 3 fragments.
Ref.:Read the text below
Sol :
The Muller AO comprehensive classification of fractures provides a standardized description of fracture patterns making
communication more precise and understandable.
A multifragmentary fracture is one that has several breaks in the bone, creating more than 2 fragments.
Wedge fractures are either spiral (low energy) or bending (high energy) and allow the proximal and distal fracture fragments to
contact each other.
The complex multifragmentary fracture is a segmental fracture or oone in which there is no contact between the proximal and
distal fragments without the bone shortening.
Your Answer. c
Correct Answer. a

(298).

Gustilo classification of open fracturesis all except :


a. In Type I is wound is shorter than 1 cm, clean and generally is caused by a fracture fragment piercing the skin.
b. In type II the wound is longer than 1 cm, not contaminated and without major soft tissue damage or defect
c. In Type III the wound is longer than 1 cm, with significant soft tissue disruption.
d. In Type IV the wound is with open fracture associated with an arterial injury.
Solution. (D) In Type IV the wound is with open fracture associated with an arterial injury.
Ref.:Read the text below
Sol :
Gustilo et al described a classification of open fractures involving 3 types with subtypes.
Type I : The wound is shorter than 1 cm. it is clean and generally is caused by a fracture fragment piercing the skin (ie, inside out
injury). This is a low energy injury.
Type II : The wound is longer than 1 cm. It is not contaminated and without major soft tissue damage or defect. This is also a low
energy injury.
Type III : The wound is longer than 1 cm, with significant soft tissue disruption. The mechanism often involves high energy trauma
resulting in a severely unstable fracture with varying degrees of fragmentation.
Your Answer. c
Correct Answer. d

Copyright 2014 Delhi Academy of Medical Sciences, All Rights Reserved.

117/119

(299).

Posterior polymorphous dystrophy:


a. Causes corectopia
b. Is inherited in an autosomal recessive fashion
c. Causes blindness in over 90% of sufferers
d. Can be treated with lamellar corneal grafts
Solution. (a) Causes corectopia
Ref.:Read the text below
Sol :
Posterior Polymorphous Dystrophy
A bilateral dominantly inherited dystrophy.
Vesicular polymorphous deposits with clear halos in Descemets membrane.
Usually asymptomatic, rarely endothelium decompensation requiring penetrating corneal graft.
The abnormal endothelium may extend into the trabecular meshwork and iris.
Glaucoma can occur as a result of trabecular meshwork involvement iris involvement can lead to corectopia and ectopia simulating
iridocorneal endothelial syndrome except that the latter is unilateral.
Your Answer. b
Correct Answer. a

(300).

True statements about ankylosing spondylitis include all except :


a. HLA B27 is fund in 90% of sufferers
b. Uveitis is found in 15 to 20% of sufferers
c. The conditions is commoner in female
d. The radiological changes can occur in the spine before symptoms.
Solution. (c) The conditions is commoner in female
Ref.:Read the text below
Sol :
Ankylosing Spondylitis
Inflammatory disordersof spinal joints.
90% have the HLA-B27 halotype.
Systemic features include peripheral arthritis, uveitis, aortic valve incompetence and chronic inflammatory bowel disease.
The conditions is commoner and more severe in males than females.
In the spine, the inflammation begins at the site where ligaments are attached to vertebral bone (the enthuses), however, signs may
occur before the patients complain of any symptoms.
Your Answer. c
Correct Answer. c

Test Answer
1.(d)

2.(a)

3.(a)

4.(a)

5.(d)

6.(c)

7.(b)

8.(c)

9.(d)

10.(b)

11.(b)

12.(a)

13.(a)

14.(c)

15.(a)

16.(a)

17.(b)

18.(a)

19.(a)

20.(a)

21.(a)

22.(d)

23.(c)

24.(d)

25.(d)

26.(a)

27.(b)

28.(c)

29.(a)

30.(d)

31.(a)

32.(c)

33.(c)

34.(a)

35.(d)

36.(b)

37.(a)

38.(b)

39.(a)

40.(d)

41.(b)

42.(a)

43.(b)

44.(a)

45.(d)

46.(c)

47.(b)

48.(b)

49.(a)

50.(d)

51.(d)

52.(a)

53.(d)

54.(a)

55.(c)

56.(a)

57.(b)

58.(b)

59.(c)

60.(b)

61.(d)

62.(a)

63.(a)

64.(c)

65.(d)

66.(b)

67.(c)

68.(b)

69.(c)

70.(d)

71.(b)

72.(d)

73.(c)

74.(b)

75.(a)

76.(a)

77.(d)

78.(c)

79.(c)

80.(a)

81.(b)

82.(a)

83.(d)

84.(c)

85.(a)

86.(c)

87.(a)

88.(d)

89.(a)

90.(a)

91.(c)

92.(a)

93.(c)

94.(b)

95.(a)

96.(b)

97.(b)

98.(c)

99.(d)

100.(c)

101.(c)

102.(d)

103.(d)

104.(b)

105.(b)

106.(b)

107.(c)

108.(d)

109.(d)

110.(c)

111.(d)

112.(b)

113.(c)

114.(d)

115.(b)

116.(c)

117.(d)

118.(b)

119.(a)

120.(c)

121.(c)

122.(c)

123.(c)

124.(b)

125.(b)

126.(b)

127.(b)

128.(c)

129.(c)

130.(c)

131.(d)

132.(c)

133.(b)

134.(a)

135.(a)

136.(d)

137.(a)

138.(d)

139.(b)

140.(b)

Copyright 2014 Delhi Academy of Medical Sciences, All Rights Reserved.

118/119

141.(b)

142.(c)

143.(c)

144.(c)

145.(b)

146.(d)

147.(a)

148.(b)

149.(b)

150.(a)

151.(b)

152.(d)

153.(b)

154.(a)

155.(a)

156.(b)

157.(b)

158.(d)

159.(c)

160.(d)

161.(c)

162.(a)

163.(c)

164.(a)

165.(b)

166.(b)

167.(b)

168.(c)

169.(b)

170.(c)

171.(c)

172.(a)

173.(b)

174.(a)

175.(d)

176.(b)

177.(d)

178.(b)

179.(b)

180.(a)

181.(b)

182.(b)

183.(c)

184.(b)

185.(d)

186.(c)

187.(c)

188.(b)

189.(a)

190.(c)

191.(a)

192.(d)

193.(a)

194.(a)

195.(a)

196.(d)

197.(d)

198.(b)

199.(a)

200.(c)

201.(b)

202.(a)

203.(d)

204.(d)

205.(b)

206.(a)

207.(b)

208.(b)

209.(c)

210.(b)

211.(a)

212.(d)

213.(a)

214.(d)

215.(d)

216.(c)

217.(a)

218.(a)

219.(c)

220.(c)

221.(a)

222.(c)

223.(a)

224.(b)

225.(c)

226.(b)

227.(b)

228.(b)

229.(a)

230.(d)

231.(c)

232.(c)

233.(b)

234.(c)

235.(b)

236.(d)

237.(c)

238.(c)

239.(a)

240.(b)

241.(a)

242.(b)

243.(c)

244.(c)

245.(b)

246.(b)

247.(a)

248.(c)

249.(d)

250.(d)

251.(b)

252.(d)

253.(a)

254.(b)

255.(b)

256.(b)

257.(a)

258.(b)

259.(c)

260.(d)

261.(a)

262.(c)

263.(c)

264.(a)

265.(d)

266.(b)

267.(c)

268.(d)

269.(a)

270.(c)

271.(d)

272.(b)

273.(a)

274.(b)

275.(d)

276.(b)

277.(a)

278.(c)

279.(d)

280.(d)

281.(d)

282.(c)

283.(a)

284.(d)

285.(b)

286.(d)

287.(d)

288.(d)

289.(a)

290.(d)

291.(a)

292.(b)

293.(a)

294.(b)

295.(d)

296.(a)

297.(a)

298.(d)

299.(a)

300.(c)

Copyright 2014 Delhi Academy of Medical Sciences, All Rights Reserved.

119/119

Potrebbero piacerti anche